Download as pdf or txt
Download as pdf or txt
You are on page 1of 199

'EPGYPYW&'7GSVMRK+YMHIPMRIW

Copyright © 1998 by College Entrance Examination Board. All rights reserved.


Advanced Placement Program and AP are registered trademarks of the College Entrance Examination Board.
'EPGYPYW&'7GSVMRK+YMHIPMRIW

Copyright © 1998 by College Entrance Examination Board. All rights reserved.


Advanced Placement Program and AP are registered trademarks of the College Entrance Examination Board.
'EPGYPYW&'7GSVMRK+YMHIPMRIW

Copyright © 1998 by College Entrance Examination Board. All rights reserved.


Advanced Placement Program and AP are registered trademarks of the College Entrance Examination Board.
'EPGYPYW&'7GSVMRK+YMHIPMRIW

Copyright © 1998 by College Entrance Examination Board. All rights reserved.


Advanced Placement Program and AP are registered trademarks of the College Entrance Examination Board.
'EPGYPYW&'7GSVMRK+YMHIPMRIW

Copyright © 1998 by College Entrance Examination Board. All rights reserved.


Advanced Placement Program and AP are registered trademarks of the College Entrance Examination Board.
'EPGYPYW&'7GSVMRK+YMHIPMRIW

Copyright © 1998 by College Entrance Examination Board. All rights reserved.


Advanced Placement Program and AP are registered trademarks of the College Entrance Examination Board.
BC{1 1999
1. A particle moves in the xy{plane so that its position at any time t, 0  t  , is given by
t2
x(t) = ; ln(1 + t) and y (t) = 3 sin t.
2
(a) Sketch the path of the particle in the xy{plane below. Indicate the direction of motion along the
path.
(b) At what time t, 0  t  , does x(t) attain its minimum value? What is the position (x(t); y(t)) of
the particle at this time?
(c) At what time t, 0 < t < , is the particle on the y{axis? Find the speed and the acceleration vector
of the particle at this time.
(a) y
( 1: graph
2
1: direction

1
x
O 1

1 =0 8
(b) x (t) = t ; 1 +
0

t
>
< 1: x (t) = 0
0

3 1: solution for t
t2 + t ; 1 = 0 >
: 1: position
p
t=
; 1 + 5 or t = 0:618 in [0; ]
2
x(0:618) = ;0:290 y (0:618) = 1:738

2 8
(c) x(t) = t2 ; ln(1 + t) = 0 >
> 1: x(t) = 0
>
< 1: solution for t
t = 1:285 or 1:286 4
1 > 1: speed
x (t) = t ;
0
y (t) = 3 cos t
0 >
: 1: acceleration vector
1+t
p
speed = (x (1:286))2 + (y (1:286))2 = 1:196
0 0

x (t) = 1 +
00
1 y (t) = ;3 sin t
00

(1 + t)2
acceleration vector = <x (1:286); y (1:286) >
00 00

= < 1:191; ;2:879 >


AB{2 / BC{2 1999
2. The shaded region, R, is bounded by the graph of y = x2 and the line y
y x2
y = 4, as shown in the gure above.
(a) Find the area of R.
y 4
(b) Find the volume of the solid generated by revolving R about the
x{axis.
(c) There exists a number k, k > 4, such that when R is revolved about
the line y = k, the resulting solid has the same volume as the solid in O
x
part (b). Write, but do not solve, an equation involving an integral
expression that can be used to nd the value of k.

Z 2
(
(a) Area = (4 ; x2 ) dx 1: integral
;2 2
Z 2
1: answer
=2 (4 ; x2 ) dx
0
 2
x3
= 2 4x ; 3
0

= 32
3 = 10:666 or 10:667

Z 2  8
(b) Volume =  42 ; (x2 )2 dx 1: limits and constant
>
>
;2 <

Z 2
3 1: integrand
>
>
= 2 (16 ; x4 ) dx :
1: answer
0
 5 2
= 2 16x ; x5
0

= 256
5 = 160:849 or 160:850

Z 2
(k ; x2 )2 ; (k ; 4)2 dx = 256 
  8
(c)  1: limits and constant
>
;2 5 >
>
>
<
4
2: integrand
>
> < ;1 > each error
>
>
:
1: equation
AB{3 / BC{3 1999
3. The rate at which water ows out of a pipe, in gallons per hour, is t ()
R t

given by a di erentiable function of time . The table above


R t (hours) (gallons per hour)
shows the rate as measured every 3 hours for a 24{hour period. 0 9.6
(a) Use a midpoint Riemann sum with 4 subdivisions of equal
Z 24 3 10.4
length to approximate ( ) . Using correct units, explain
R t dt
6 10.8
0
the meaning of your answer in terms of water ow. 9 11.2
12 11.4
(b) Is there some time , 0 24, such that ( ) = 0? Justify
t < t < R
0
t
15 11.3
your answer.
18 10.7
(c) The rate of water ow ( ) can be approximated by
R t
21 10.2
1 ;768 + 23 ; 2 . Use ( ) to approximate the
( ) = 79
Q t t t Q t 24 9.6
average rate of water ow during the 24{hour time period.
Indicate units of measure.

Z 24
8
(a) ()
R t dt  6[ (3) + (9) + (15) + (21)]
R R R R >
>
<
1:(3) + (9) + (15) + (21)
R R R R
0
= 6[10 4 + 11 2 + 11 3 + 10 2]
: : : : 3 1: answer
>
>
= 258.6 gallons :
1: explanation
This is an approximation to the total ow in
gallons of water from the pipe in the 24{hour
period.

(
(b) Yes; 1: answer
2
Since (0) = (24) = 9 6, the Mean Value
R R : 1: MVT or equivalent
Theorem guarantees that there is a , 0 24, t < t <

such that ( ) = 0.
R
0
t

8
(c) Average rate of ow >
>
<
1:
limits and average value constant
 average value of ( ) Q t
3 1: ( ) as integrand
Q t
>
>
Z 24 :
1 1 (768 + 23 ; 2 ) 1: answer
= 24 79 t t dt
0

= 10 785 gal/hr or 10.784 gal/hr


:

(units) Gallons in part (a) and gallons/hr in 1: units


part (c), or equivalent.
BC{4 1999
4. The function has derivatives of all orders for all real numbers . Assume (2) = ;3, (2) = 5,
f x f f
0

(2) = 3, and (2) = ;8.


f
00
f
000

(a) Write the third{degree Taylor polynomial for about = 2 and use it to approximate (1 5). f x f :

(b) The fourth derivative of satis es the inequality j (4) ( )j  3 for all in the closed interval [1 5 2].
f f x x : ;

Use the Lagrange error bound on the approximation to (1 5) found in part (a) to explain why f :

(1 5) 6= ;5.
f :

(c) Write the fourth{degree Taylor polynomial, ( ), for ( ) = ( 2 + 2) about = 0. Use to P x g x f x x P

explain why must have a relative minimum at = 0.


g x

( 2)( ) = ;3 + 5( ; 2) + 32 ( ; 2)2 ; 68 ( ; 2)3


8
(a) T3 f; x x x x >
< 3: ( 2)( )
T3 f; x

4 ;1 each error
>
< >

f (1 5) 
: ( 2)(1 5)
T3 f; : : 1: approximation of (1 5) f :

= ;3 + 5(;0 5) + 32 (;0 5)2 ; 43 (;0 5)3


: : :

= ;4 9583 = ;4 958
: :

( 1: value of Lagrange Error Bound


(b) Lagrange Error Bound = 4!3 j1 5 ; 2j4 = 0 0078125 : :
2
1: explanation
f (1 5) ;4 9583 ; 0 0078125 = ;4 966 ;5
: > : : : >

Therefore, (1 5) 6= ;5. f :

8
(c) ( ) = 4 ( 0)( ) >
> 2: ( 0)( )
< ;1 each incorrect, missing,
P x T g; x T4 g; x

= 2 ( 2)( 2 + 2) = ;3 + 5 2 + 23 4
< >
3 or extra term
T f; x x x >
>
: 1: explanation
The coecient of in ( ) is (0). This coecientx P x g
0

is 0, so (0) = 0. g
0

The coecient of 2 in ( ) is 2!(0) . This coecient


00
g
x P x

is 5, so (0) = 10 which is greater than 0.


g
00

Therefore, has a relative minimum at = 0.


g x

Note:
< ;1 max for improper use of +
> ::: or
equality
AB{5 / BC{5 1999
5. The graph of the functionZ x, consisting of three line segments, is
f

given above. Let ( ) = () . g x f t dt 4 (1, 4)


1
(a) Compute (4) and (;2). g g
3

(b) Find the instantaneous rate of change of , with respect to , at g x


2
(2, 1)
= 1.
x
1

(c) Find the absolute minimum value of on the closed interval g –2 –1 O 1 2 3 4


[;2 4]. Justify your answer.
; –1
(4, –1)
(d) The second derivative of is not de ned at = 1 and = 2. g x x –2
How many of these values are {coordinates of points of x

in ection of the graph of ? Justify your answer. g

Z4 (
(a) (4) =
g f t ( ) = 32 + 1 + 21 ; 12 = 52
dt 1: (4)
g
1 2
Z ;2 1: (;2)
g

g (;2) = ( ) = ; 12 (12) = ;6
f t dt
1

(b) 0 (1) = (1) = 4


g f 1: answer

8
(c) g is increasing on [;2 3] and decreasing on [3 4]. ; ; >
< 1: interior analysis
Therefore, has absolute minimum at an 3 1: endpoint analysis
endpoint of [;2 4].
g

;
>
: 1: answer
Since (;2) = ;6 and (4) = 25 ,
g g

the absolute minimum value is ;6.

8
(d) One; = 1 x >
< 1: choice of = 1 only
x

On (;2 1), 00 ( ) = 0 ( ) 0
; g x f x > 3 1: show (1 (1)) is a point of in ection
> ;g

On (1 2), 00 ( ) = 0 ( ) 0
; g x f x <
: 1: show (2 (2)) is not a point of in ection
;g

On (2 4), 00 ( ) = 0 ( ) 0
; g x f x <

Therefore (1 (1)) is a point of in ection and


;g

(2 (2)) is not.
;g
BC{6 1999
6. Let be the function whose graph goes through the point (3 6) and whose derivative is given by
f ;

( ) = 1 +2 .
x
0
e
f x
x

(a) Write an equation of the line tangent to the graph of at = 3 and use it to approximate (3 1). f x f :

(b) Use Euler's method, starting at = 3 with a step size of 0.05, to approximate (3 1). Use to x f : f
00

explain why this approximation is less than (3 1).


Z 31 :
f :

(c) Use f
0
( )
x dx to evaluate (3 1). f :
3

(3) = 1 +9
3 8 1: (3)
(a) f
0
e
= 2 342 or 2 343
: : >< f
0

3 1: equation
>:
; 6 = 1 +9 ( ; 3) e
3
y x 1: approximation of f (3 1)
:

= 6 + 1 +9 ( ; 3) e
3
y x

(3 1)  6 + 1 +9 (0 1) = 6 234 e
3
f : : :

8 1: Euler's method equations or


(b) (3 05)  (3) + (3)(0 05)
f : f f
0
: >>
= 6 + 0 11714 = 6 11714
>> equivalent table
: :
>< 1: Euler approximation to (3 1)
(3 1)  6 11714 + (3 05)(0 05)
f :
4
> (not eligible without rst point)
0
f : : f : :

= 6 11714 + (2 37735)(0 05) = 6 236 >> 1: ( )


>:
00
: : : : f x

1: reason
f
00
( )=
x
x e
2 x
; 2 (1 + ) = ( ; 2) ; 2
x e
x
x e
x

4 x
3 x

For  3, ( ) x
; 2 0 and the graph
f
00
x >
e
x

>
3 x

of is concave upward on (3 3 1). Therefore,


f ; :

the Euler approximation lines at 3 and 3.05 lie


below the graph.

Z 31 1+ 8> Z 3 1 1 +
< 1:
: x

= (3 1) ; (3)
: x

(c) (3 1) ; (3) = e e
f : f dx dx f : f
2 2
2
>: 3
x
3 x

f (3 1) = 6 + 0 2378 = 6 237 or 6 238


: : : :
1: answer
!0Å#ALCULUSÅ!"nÅÅ"#n 
y
,ETÅ2ÅBEÅTHEÅSHADEDÅREGIONÅINÅTHEÅFIRSTÅQUADRANTÅENCLOSEDÅBYÅTHEÅGRAPHSÅOF
Y  E X Å Y    COS X ÅANDÅTHEÅY AXIS ÅASÅSHOWNÅINÅTHEÅFIGUREÅABOVE
 1.5

A &INDÅTHEÅAREAÅOFÅTHEÅREGIONÅ2 1 2
y = e–x
B &INDÅTHEÅVOLUMEÅOFÅTHEÅSOLIDÅGENERATEDÅWHENÅTHEÅREGIONÅ2ÅISÅREVOLVED
0.5 R y = 1 – cos x
ABOUTÅTHEÅX AXIS
C 4HEÅREGIONÅ2ÅISÅTHEÅBASEÅOFÅAÅSOLIDÅ&ORÅTHISÅSOLID ÅEACHÅCROSSÅSECTION x
O 0.5 1 1.5
PERPENDICULARÅTOÅTHEÅX AXISÅISÅAÅSQUAREÅ&INDÅTHEÅVOLUMEÅOFÅTHISÅSOLID

2EGIONÅ2 Å #ORRECTÅLIMITSÅINÅANÅINTEGRALÅINÅA ÅB


E X     COS X ÅATÅXÅÅÅÅ! ORÅC 

! ¦   ÅINTEGRAND
£
!REA  ¨ E X    COS X DX

A Å ¦¤

¦
¦   ÅANSWER
¥
ÅÅORÅ

!  
B 6OLUME Å Q ¨ E X    COS X 
DX £
¦
¦
  ÅINTEGRANDÅANDÅCONSTANT
 ¦
Å ¦¤ ÅÅÅÅÅ  ÅEACHÅERROR
¦
¦
Å Q ÅÅÅORÅ ¦
¦   ÅANSWER
¥

! X   £
¦   ÅINTEGRAND
¦
C 6OLUME  ¨ E    COS X DX ¦
¦
 ¦ ÅÅÅÅÅ  ÅEACHÅERROR
¦
¦
¦ ÅÅÅÅÅÅ.OTEÅÅIFÅNOTÅOFÅTHEÅFORM
Å Å ¦
¤
¦
¦ D
¦
¦ ÅÅÅÅÅÅÅÅÅÅÅÅÅK ¨  F X  GX  DX
¦
¦ C
¦
¦   ÅANSWER
¦
¥

Copyright © 2000 by College Entrance Examination Board and Educational Testing Service. All rights reserved.
AP is a registered trademark of the College Entrance Examination Board.
!0Å#ALCULUSÅ!"nÅÅ"#n 

4WOÅRUNNERS Å!ÅANDÅ" ÅRUNÅONÅAÅSTRAIGHTÅRACETRACKÅFOR 14


13
ÅbÅTÅbÅÅSECONDSÅ4HEÅGRAPHÅABOVE ÅWHICHÅCONSISTSÅOFÅTWOÅLINE 12
11

Velocity of Runner A
(3, 10) (10, 10)

(meters per second)


10
SEGMENTS ÅSHOWSÅTHEÅVELOCITY ÅINÅMETERSÅPERÅSECOND ÅOFÅ2UNNER 9
8
7
!Å4HEÅVELOCITY ÅINÅMETERSÅPERÅSECOND ÅOFÅ2UNNERÅ"ÅISÅGIVENÅBY 6
5

T
4
3
THEÅFUNCTIONÅVÅDEFINEDÅBYÅ VT   2
T  1
0 t
1 2 3 4 5 6 7 8 9 10
A &INDÅTHEÅVELOCITYÅOFÅ2UNNERÅ!ÅANDÅTHEÅVELOCITYÅOFÅ2UNNER Time
(seconds)

"ÅATÅTIMEÅTÅÅÅSECONDSÅ)NDICATEÅUNITSÅOFÅMEASURE
B &INDÅTHEÅACCELERATIONÅOFÅ2UNNERÅ!ÅANDÅTHEÅACCELERATIONÅOFÅ2UNNERÅ"ÅATÅTIMEÅTÅÅÅSECONDS
)NDICATEÅUNITSÅOFÅMEASURE
C &INDÅTHEÅTOTALÅDISTANCEÅRUNÅBYÅ2UNNERÅ!ÅANDÅTHEÅTOTALÅDISTANCEÅRUNÅBYÅ2UNNERÅ"ÅOVERÅTHEÅTIME
INTERVALÅÅbÅTÅbÅÅSECONDSÅ)NDICATEÅUNITSÅOFÅMEASURE

  ¦   ÅVELOCITYÅFORÅ2UNNERÅ!
£
ÅA 2UNNERÅ!ÅVELOCITY Å ¸  Å ¦¤
  ¦   ÅVELOCITYÅFORÅ2UNNERÅ"
¦
¥
ÅÅORÅÅMETERSSEC


2UNNERÅ"Å V  ÅÅÅMETERSSEC



ÅB 2UNNERÅ!ÅACCELERATIONÅÅ ÅÅÅ METERSSEC ¦   ÅACCELERATIONÅFORÅ2UNNERÅ!
£
 Å ¦¤
¦
¦   ÅACCELERATIONÅFORÅ2UNNERÅ"
¥

2UNNERÅ"Å A  V a  
T  T 

Å  ÅMETERSSEC


£
¦   ÅDISTANCEÅFORÅ2UNNERÅ!
¦
¦
 ¦
¦ ÅÅÅÅÅÅ  ÅMETHOD
ÅC 2UNNERÅ!ÅDISTANCE Å    ÅÅÅMETERS ¦
¦
 ¦
¦ ÅÅÅÅÅÅ  ÅANSWER
Å ¦¤
 T ¦
¦   ÅDISTANCEÅFORÅ2UNNERÅ"
2UNNERÅ"ÅDISTANCE Å ¨ DT ÅÅÅMETERS ¦
 T  ¦
¦ ÅÅÅÅÅÅ  ÅINTEGRAL
¦
¦
¦
¦ ÅÅÅÅÅÅ  ÅANSWER
¦
¥

UNITS METERSSECÅINÅPARTÅA Å METERSSEC ÅINÅPARTÅB ÅAND ÅUNITS


METERSÅINÅPARTÅC ÅORÅEQUIVALENT
Copyright © 2000 by College Entrance Examination Board and Educational Testing Service. All rights reserved.
AP is a registered trademark of the College Entrance Examination Board.
!0Å#ALCULUSÅ"#n 

4HEÅ4AYLORÅSERIESÅABOUTÅXÅÅÅFORÅAÅCERTAINÅFUNCTIONÅFÅCONVERGESÅTOÅ F X ÅFORÅALLÅXÅINÅTHEÅINTERVALÅOF
 N N  
CONVERGENCEÅ4HEÅNTHÅDERIVATIVEÅOFÅFÅATÅXÅÅÅISÅGIVENÅBYÅ F N   N ÅANDÅ F   
 N  
A 7RITEÅTHEÅTHIRD DEGREEÅ4AYLORÅPOLYNOMIALÅFORÅFÅABOUTÅXÅÅ
B &INDÅTHEÅRADIUSÅOFÅCONVERGENCEÅOFÅTHEÅ4AYLORÅSERIESÅFORÅFÅABOUTÅXÅÅ
C 3HOWÅTHATÅTHEÅSIXTH DEGREEÅ4AYLORÅPOLYNOMIALÅFORÅFÅABOUTÅXÅÅÅAPPROXIMATESÅ F  ÅWITHÅERRORÅLESS

THANÅ 


    ÅÅ 0 F  X
ÅA F a  Å F aa  Å F aaa 
  
nÅEACHÅERRORÅORÅMISSINGÅTERM
     
0  F  X   X   X    X  
   
ÅÅÅÅÅÅ.OTEÅnÅMAXÅFORÅIMPROPERÅUSE
OFÅEXTRAÅTERMS ÅEQUALITYÅORÅ b

F N   N
B AN   N
N  N  £   ÅGENERALÅTERM
¦
¦
¦
¦
 N X   N 
¦
¦
  ÅSETSÅUPÅRATIOÅTEST
N N  ¦
LIM  LIM
 N 
X  Å ¦
¤   ÅCOMPUTESÅTHEÅLIMIT
N ld  N X   N N ld  N  ¦
¦
¦
¦   ÅAPPLIESÅRATIOÅTESTÅTOÅ
N N  ¦
¦
¦ ÅÅÅÅÅÅGETÅRADIUSÅOFÅCONVERGENCE
X  ¦
¥
 


4HEÅRADIUSÅOFÅCONVERGENCEÅISÅ

C 4HEÅ4AYLORÅSERIESÅABOUTÅXÅÅÅFORÅTHEÅFUNCTIONÅF £ 
¦
¦   ÅERRORÅBOUNDÅÅ
WHENÅEVALUATEDÅATÅXÅÅ ÅISÅANÅALTERNATINGÅSERIES ¦
¦ 
¦
¦   ÅREFERSÅTOÅANÅALTERNATINGÅSERIESÅ
WITHÅABSOLUTEÅVALUEÅOFÅTERMSÅDECREASINGÅTOÅÅ4HE ¦
¤
¦
¦ ÅÅÅÅÅÅANDÅINDICATESÅTHEÅERRORÅBOUNDÅISÅ
ERRORÅINÅAPPROXIMATINGÅ F  ÅWITHÅTHEÅTHÅDEGREE ¦
¦
¦
¦ ÅÅÅÅÅÅFOUNDÅFROMÅTHEÅNEXTÅTERM
4AYLORÅPOLYNOMIALÅATÅXÅÅÅISÅLESSÅTHANÅTHEÅFIRST ¦
¥
OMITTEDÅTERMÅINÅTHEÅSERIES

  
F   0  F   b   
   

Copyright © 2000 by College Entrance Examination Board and Educational Testing Service. All rights reserved.
AP is a registered trademark of the College Entrance Examination Board.
!0Å#ALCULUSÅ"#n 

!ÅMOVINGÅPARTICLEÅHASÅPOSITIONÅ X T YT ÅATÅTIMEÅTÅ4HEÅPOSITIONÅOFÅTHEÅPARTICLEÅATÅTIMEÅTÅÅÅISÅ  ÅAND


 
THEÅVELOCITYÅVECTORÅATÅANYÅTIMEÅTÅÅÅISÅGIVENÅBYÅ     
T T
A &INDÅTHEÅACCELERATIONÅVECTORÅATÅTIMEÅTÅÅ
B &INDÅTHEÅPOSITIONÅOFÅTHEÅPARTICLEÅATÅTIMEÅTÅÅ
C &ORÅWHATÅTIMEÅTÅÅÅDOESÅTHEÅLINEÅTANGENTÅTOÅTHEÅPATHÅOFÅTHEÅPARTICLEÅATÅ X T YT ÅHAVEÅAÅSLOPEÅOFÅ
D 4HEÅPARTICLEÅAPPROACHESÅAÅLINEÅASÅ T l d Å&INDÅTHEÅSLOPEÅOFÅTHISÅLINEÅ3HOWÅTHEÅWORKÅTHATÅLEADSÅTOÅYOUR
CONCLUSION

  £
¦   ÅCOMPONENTSÅOFÅACCELERATION
A ACCELERATIONÅVECTORÅÅ X aaT Y aaT  
  ¦
¦
T T
Å ¦¤ ÅÅÅÅÅÅVECTORÅASÅAÅFUNCTIONÅOFÅT
  ¦
¦
X aa Y aa   ¦   ÅACCELERATIONÅVECTORÅATÅT  
  ¦
¥

  £
¦   ÅANTIDIFFERENTIATION
B X T YT  T #  T  # ¦
¦
T T ¦
Å ¤   ÅUSESÅINITIALÅCONDITIONÅATÅT  
¦
¦
   X  Y   #  # ¦   ÅPOSITIONÅATÅT  
¦
¦
¥
#   #  
.OTEÅMAXÅÅÅ;nn=ÅIFÅNOÅCONSTANTSÅOF
   
X  Y      INTEGRATION
   


DY  £
¦ DY
C ÅÅ  T   ¦  Å  ÅASÅEQUATIONÅINÅT
DX  Å ¦¤ DX
  ¦
¦  ÅSOLUTIONÅFORÅT
T ¦
¥
  
ÅÅÅÅÅÅ      T 
T T 

ÅÅÅÅÅÅT 



DY 
D LIM  LIM T  
T ld DX T ld  £
¦ DY YT
  ¦
¦   ÅCONSIDERSÅLIMITÅOFÅ ÅORÅ
T Å ¤ DX X T
¦
¦
nÅORÅn ¦
¦   ÅANSWER
¥
3INCEÅ X T l d ÅASÅ T l d ÅTHEÅSLOPEÅOFÅTHEÅLINEÅIS .OTEÅÅIFÅNOÅCONSIDERATIONÅOFÅLIMIT

YT T  
LIM  LIM T 
T ld X T T ld 
T
T
Copyright © 2000 by College Entrance Examination Board and Educational Testing Service. All rights reserved.
AP is a registered trademark of the College Entrance Examination Board.
!0Å#ALCULUSÅ!"nÅÅ"#n 

#ONSIDERÅTHEÅCURVEÅGIVENÅBYÅ XY   X Y   
DY X Y  Y 
A 3HOWÅTHATÅ  
DX XY  X 
B &INDÅALLÅPOINTSÅONÅTHEÅCURVEÅWHOSEÅX COORDINATEÅISÅ ÅANDÅWRITEÅANÅEQUATIONÅFORÅTHEÅTANGENTÅLINEÅAT
EACHÅOFÅTHESEÅPOINTS
C &INDÅTHEÅX COORDINATEÅOFÅEACHÅPOINTÅONÅTHEÅCURVEÅWHEREÅTHEÅTANGENTÅLINEÅISÅVERTICAL

DY DY £
¦   IMPLICITÅDIFFERENTIATION
A Y XY  X Y  X   ¦
¦
DX DX  ¤ DY
¦
¦ ÅÅVERIFIESÅEXPRESSIONÅFORÅ
DY ¦
¥ DX
XY  X   X Y  Y 
DX

DY X Y  Y 

DX XY  X 

B 7HENÅXÅÅ Y   Y   £
¦   ÅY   Y  
¦
¦
¦
Y  Y     Å ¤   ÅSOLVESÅFORÅY
¦
¦
Y   Y   ¦
¦   ÅTANGENTÅLINESÅ
¦
¥
YÅÅ ÅYÅÅn .OTEÅÅIFÅNOTÅSOLVINGÅANÅEQUATIONÅOFÅTHE
DY  FORMÅ Y   Y  K
!TÅ   Å  
DX 
4ANGENTÅLINEÅEQUATIONÅISÅYÅÅ

DY    
!TÅ   Å   
DX    
4ANGENTÅLINEÅEQUATIONÅISÅ Y   X  

C 4ANGENTÅLINEÅISÅVERTICALÅWHENÅ XY  X    £
¦ DY
¦
¦   ÅSETSÅDENOMINATORÅOFÅ ÅEQUALÅTOÅ
¦ DX
¦
¦ 
  ¦ 
X Y  X    ÅGIVESÅXÅÅÅORÅ Y  X ¦   ÅSUBSTITUTESÅY   X ÅORÅX  o Y Å
 Å ¦¤
¦
¦ ÅÅÅÅÅÅINTOÅTHEÅEQUATIONÅFORÅTHEÅCURVE
¦
¦
4HEREÅISÅNOÅPOINTÅONÅTHEÅCURVEÅWITH ¦
¦
¦   ÅSOLVESÅFORÅX COORDINATE
X COORDINATEÅ ¦
¦
¥

     
7HENÅ Y  X ÅÅ X  X  
  
 
ÅÅÅÅÅÅÅÅÅÅÅÅÅÅÅÅÅÅÅÅÅ  X  


ÅÅÅÅÅÅÅÅÅÅÅÅÅÅÅÅÅÅÅÅÅÅÅÅ X  

Copyright © 2000 by College Entrance Examination Board and Educational Testing Service. All rights reserved.
AP is a registered trademark of the College Entrance Examination Board.
!0Å#ALCULUSÅ"#n 

DY
#ONSIDERÅTHEÅDIFFERENTIALÅEQUATIONÅGIVENÅBYÅ  X Y    
DX
A /NÅTHEÅAXESÅPROVIDED ÅSKETCHÅAÅSLOPEÅFIELDÅFORÅTHEÅGIVEN y

DIFFERENTIALÅEQUATIONÅATÅTHEÅELEVENÅPOINTSÅINDICATED
1
B 5SEÅTHEÅSLOPEÅFIELDÅFORÅTHEÅGIVENÅDIFFERENTIALÅEQUATIONÅTO
EXPLAINÅWHYÅAÅSOLUTIONÅCOULDÅNOTÅHAVEÅTHEÅGRAPHÅSHOWNÅBELOW –2 –1 O 1 2
x

C &INDÅTHEÅPARTICULARÅSOLUTIONÅ Y  F X ÅTOÅTHEÅGIVENÅDIFFERENTIAL –1

EQUATIONÅWITHÅTHEÅINITIALÅCONDITIONÅ F    
D &INDÅTHEÅRANGEÅOFÅTHEÅSOLUTIONÅFOUNDÅINÅPARTÅC 

A
£
¦ Å  ÅZEROÅSLOPEÅATÅÅPOINTSÅWITH
¦
¦
¦
¦ ÅÅÅÅÅÅY  ÅANDÅX  
¦
¦
Å ¦
¤ Å  ÅNEGATIVEÅSLOPEÅATÅ  ÅANDÅ 
¦
¦
¦
¦ ÅÅÅÅÅÅPOSITIVEÅSLOPEÅATÅ  ÅANDÅ 
¦
¦
¦
¦ ÅÅÅÅÅÅSTEEPERÅSLOPEÅATÅY  ÅTHANÅY  Å
¥

B 4HEÅGRAPHÅDOESÅNOTÅHAVEÅSLOPEÅÅWHEREÅ Y    Å REASON


nÅORÅn
4HEÅSLOPEÅFIELDÅSHOWNÅSUGGESTSÅTHATÅSOLUTIONS
AREÅASYMPTOTICÅTOÅ Y   ÅFROMÅBELOW ÅBUTÅTHE
GRAPHÅDOESÅNOTÅEXHIBITÅTHISÅBEHAVIOR

 £
¦   ÅSEPARATESÅVARIABLES
C DY  X DX ¦
¦
Y    ¦
¦   ÅANTIDERIVATIVES
¦
¦
  ¦
¦   ÅCONSTANTÅOFÅINTEGRATION
  X
Y  
# Å ¦
¦
¤
¦
¦   ÅUSESÅINITIALÅCONDITIONÅF   
¦
¦
  ¦
¦   ÅSOLVESÅFORÅY
 # ÅÅÅ#  ¦
  ¦
¦
¦ ÅÅÅÅÅÅÅIFÅY ÅISÅLINEAR
¥
¦
   .OTE MAXÅÅ;    =ÅIFÅNOÅCONSTANTÅOF
  X  ÅÅÅ Y   
Y   X 
INTEGRATION
.OTE ÅIFÅNOÅSEPARATIONÅOFÅVARIABLES

D ÅRANGEÅISÅ  b Y   Å ANSWER


ÅIFÅnÅNOTÅINÅRANGE
Copyright © 2000 by College Entrance Examination Board and Educational Testing Service. All rights reserved.
AP is a registered trademark of the College Entrance Examination Board.
AP® CALCULUS BC
2001 SCORING GUIDELINES

Question 1
An object moving along a curve in the xy-plane has position  x (t ), y(t )  at time t with
dx dy
 cos  t 3  and  3 sin  t 2 
dt dt
for 0 > t > 3 . At time t = 2, the object is at position (4,5).
(a) Write an equation for the line tangent to the curve at (4,5).
(b) Find the speed of the object at time t = 2.
(c) Find the total distance traveled by the object over the time interval 0 > t > 1 .
(d) Find the position of the object at time t = 3.

dy 3 sin  t 2 
(a) 
dx cos  t 3 
dy 3 sin  22  1 : tangent line
  15.604
dx t 2 cos  2 3

y  5  15.604(x  4)

(b) Speed = cos2 (8)  9 sin2 (4) = 2.275 1 : answer

1 
 2 : distance integral
(c) Distance = ¨0 cos2  t 3   9 sin2  t 2  dt 


 <  1  each integrand error
= 1.458 3: 


 <  1  error in limits



 1 : answer


3 £
¦ 1 : definite integral for x
(d) x (3)  4  ¨2 cos  t 3  dt = 3.953 or 3.954 ¦
¦
¦
¦ 1 : answer for x (3)
3 ¦
¨2 3 sin  t dt = 4.906 4: ¤
2
y(3)  5  ¦ 1 : definite integral for y
¦
¦
¦
¦
¦ 1 : answer for y(3)
¥

Copyright © 2001 by College Entrance Examination Board. All rights reserved.


Advanced Placement Program and AP are registered trademarks of the College Entrance Examination Board.

2
AP® CALCULUS BC
2001 SCORING GUIDELINES

Question 2
The temperature, in degrees Celsius (°C), of the water in a pond is a t W (t )
differentiable function W of time t. The table above shows the water (days) (°C)
0 20
temperature as recorded every 3 days over a 15-day period. 3 31
(a) Use data from the table to find an approximation for W =(12) . Show the 6 28
9 24
computations that lead to your answer. Indicate units of measure. 12 22
(b) Approximate the average temperature, in degrees Celsius, of the water 15 21

over the time interval 0 > t > 15 days by using a trapezoidal


approximation with subintervals of length t  3 days.
(c) A student proposes the function P, given by P (t )  20  10te (t / 3) , as a model for the
temperature of the water in the pond at time t, where t is measured in days and P (t ) is
measured in degrees Celsius. Find P =(12) . Using appropriate units, explain the meaning of
your answer in terms of water temperature.
(d) Use the function P defined in part (c) to find the average value, in degrees Celsius, of P (t )
over the time interval 0 > t > 15 days.

(a) Difference quotient; e.g.


W (15)  W (12) 1 £
¦ 1 : difference quotient
W (12)    °C/day or 2 : ¦¤
15  12 3 ¦
¦ 1 : answer (with units)
¥
W (12)  W (9) 2
W (12)    °C/day or
12  9 3
W (15)  W (9) 1
W (12)    °C/day
15  9 2
3 ¦ 1 : trapezoidal method
£
(b)
2
 20  2(31)  2(28)  2(24)  2(22)  21   376.5 2 : ¦¤
¦
¦ 1 : answer
1 ¥
Average temperature  (376.5)  25.1 °C
15
10 t / 3
(c) P (12)  10e t / 3  te ¦ 1 : P a(12) (with or without units)
£
3 t 12 2 : ¦¤
¦
¦ 1 : interpretation
 30e 4  0.549 °C/day ¥
This means that the temperature is decreasing at the
rate of 0.549 °C/day when t = 12 days.
1 15 £
¦ 1 : integrand
(d)
15 ¨0
 20  10tet / 3 dt  25.757 °C ¦
¦
¦
¦ 1 : limits and
3: ¦
¤
¦
¦ average value constant
¦
¦
¦
¦ 1 : answer
¥
Copyright © 2001 by College Entrance Examination Board. All rights reserved.
Advanced Placement Program and AP are registered trademarks of the College Entrance Examination Board.

3
AP® CALCULUS BC
2001 SCORING GUIDELINES

Question 3
A car is traveling on a straight road with velocity
55 ft/sec at time t = 0. For 0 > t > 18 seconds, the
car’s acceleration a(t ) , in ft/sec2, is the piecewise
linear function defined by the graph above.
(a) Is the velocity of the car increasing at t = 2
seconds? Why or why not?
(b) At what time in the interval 0 > t > 18 , other than t = 0, is the velocity of the car
55 ft/sec? Why?
(c) On the time interval 0 > t > 18 , what is the car’s absolute maximum velocity, in ft/sec,
and at what time does it occur? Justify your answer.
(d) At what times in the interval 0 > t > 18 , if any, is the car’s velocity equal to zero? Justify
your answer.

(a) Since v a(2)  a(2) and a(2)  15  0 , the velocity is 1 : answer and reason
increasing at t = 2.

(b) At time t = 12 because ¦ 1 : t  12


£
2 : ¦¤
12 ¦
¦ 1 : reason
v(12)  v(0)  ¨0 a(t ) dt  0 . ¥

(c) The absolute maximum velocity is 115 ft/sec at 


 1:t 6


t = 6. 
 1 : absolute maximum velocity


The absolute maximum must occur at t = 6 or 
 1 : identifies t  6 and


at an endpoint. 
 t  18 as candidates
4: 

6  or

v(6)  55  ¨0 a(t )dt 

 indicates that v increases,
1 

 55  2(15)  (4)(15)  115  v(0) 
 decreases, then increases
2 

18 
 1 : eliminates t  18
¨6 a(t ) dt  0 so v(18)  v(6) 


(d) The car’s velocity is never equal to 0. The absolute


minimum occurs at t = 16 where ¦ 1 : answer
£
2 : ¦¤
16 ¦
¦ 1 : reason
v(16)  115  ¨6 a(t ) dt  115  105  10  0 . ¥

Copyright © 2001 by College Entrance Examination Board. All rights reserved.


Advanced Placement Program and AP are registered trademarks of the College Entrance Examination Board.

4
AP® CALCULUS BC
2001 SCORING GUIDELINES

Question 4

Let h be a function defined for all x L 0 such that h(4)  3 and the derivative of h is given
x2  2
by h (x )  for all x L 0 .
x
(a) Find all values of x for which the graph of h has a horizontal tangent, and determine
whether h has a local maximum, a local minimum, or neither at each of these values.
Justify your answers.
(b) On what intervals, if any, is the graph of h concave up? Justify your answer.
(c) Write an equation for the line tangent to the graph of h at x = 4.
(d) Does the line tangent to the graph of h at x = 4 lie above or below the graph of h for
x  4 ? Why?

(a) h a(x )  0 at x  o 2  1:x  2






 1 : analysis


4: 
h =(x ) 0 + und 0 + 

2 : conclusions

  1 > not dealing with

 discontinuity at 0
x  2 0 2 


Local minima at x   2 and at x  2

2 £ 1 : h aa(x )
¦
(b) h (x )  1   0 for all x L 0 . Therefore, ¦
¦
x2 ¦
3 : ¤ 1 : h aa(x )  0
the graph of h is concave up for all x L 0 . ¦
¦
¦
¦ 1 : answer
¦
¥

16  2 7
(c) h (4)  
4 2

7
y3 (x  4) 1 : tangent line equation
2

(d) The tangent line is below the graph because 1 : answer with reason
the graph of h is concave up for x  4 .

Copyright © 2001 by College Entrance Examination Board. All rights reserved.


Advanced Placement Program and AP are registered trademarks of the College Entrance Examination Board.

5
AP® CALCULUS BC
2001 SCORING GUIDELINES

Question 5

Let f be the function satisfying f (x )  3x f (x ) , for all real numbers x, with f (1)  4 and
lim f (x )  0 .
x ld
@
(a) Evaluate ¨1 3x f (x ) dx . Show the work that leads to your answer.

(b) Use Euler’s method, starting at x = 1 with a step size of 0.5, to approximate f (2) .
dy
(c) Write an expression for y  f (x ) by solving the differential equation  3xy with the
dx
initial condition f (1)  4 .

@
¦ 1 : use of FTC
£
(a) ¨1 3x f (x ) dx
2 : ¦¤
¦
¦ 1 : answer from limiting process
d b b ¥
= ¨1 f (x ) dx  lim
b ld ¨1 f (x ) dx  lim f (x )
b ld 1
= lim f (b)  f (1)  0  4  4
b ld

(b) f (1.5)  f (1)  f (1)(0.5) £


¦ 1 : Euler's method equations or
¦
¦
= 4  3(1)(4)(0.5)  2 ¦
¦ equivalent table
2 : ¦¤
f (2)  2  f (1.5)(0.5) ¦
¦ 1 : Euler approximation to f (2)
¦
¦
 2  3(1.5)(2)(0.5)  2.5 ¦
¦ (not eligible without first point)
¥

1 £
¦ 1 : separates variables
(c) dy  3x dx ¦
¦
y ¦ 1 : antiderivatives
¦
¦
3 ¦
ln y   x 2  k 5 : ¦¤ 1 : constant of integration
2 ¦
3 2 x 2
¦
¦ 1 : uses initial condition f (1)  4
y  Ce ¦
¦
¦
¦ 1 : solves for y
4  Ce
3 2
; C  4e
3
2 ¦
¥
3 3 2 x 2
Note: max 2/5 [1-1-0-0-0] if no constant
2
y  4e e
of integration

Note: 0/5 if no separation of variables

Copyright © 2001 by College Entrance Examination Board. All rights reserved.


Advanced Placement Program and AP are registered trademarks of the College Entrance Examination Board.

6
AP® CALCULUS BC
2001 SCORING GUIDELINES

Question 6
A function f is defined by
1 2 3 n 1
f (x )   2 x  3 x 2  L  n 1 x n  L
3 3 3 3
for all x in the interval of convergence of the given power series.
(a) Find the interval of convergence for this power series. Show the work that leads to your
answer.
1
f (x ) 
(b) Find lim 3.
x
x l0

(c) Write the first three nonzero terms and the general term for an infinite series that
1
represents ¨0 f (x )dx .
(d) Find the sum of the series determined in part (c).

(n  2)x n 1
3n 2 (n  2) x x £ 1 : sets up ratio test
(a) lim  lim  1 ¦
¦
n  (n  1)x n n  (n  1) 3 3 ¦
¦
¦ 1 : computes limit
3n 1 4: ¦
¤
d
n 1 ¦
¦ 1 : conclusion of ratio test
At x  3 , the series is œ (1)n 3
, which diverges. ¦
¦
¦ 1 : endpoint conclusion
n 0 ¦
¥
d
n 1
At x = 3, the series is œ 3
, which diverges.
n 0
Therefore, the interval of convergence is 3  x  3 .
1
f (x ) 
(b) lim
x l0 x x l 0 32 33
3  lim 2  3 x  4 x 2  L  2
34 9  1 : answer

¨0 f (x )dx  ¨0  3  32 x  33 x 2  L  
1 1 1 2 3 n 1 n £
¦
(c) x  L dx ¦ 1 : antidifferentiation
3n 1 ¦
¦
¦ of series
 13 x  31 x 
x 1 ¦
1 3 1 ¦
= x  L  n 1 x n 1  L
2
 ¦
33 2
3 x 0 3 : ¦¤ 1 : first three terms for
¦
¦
1 1 1 1 ¦ definite integral series
=  2  3  L  n 1  L ¦
¦
3 3 3 3 ¦
¦
¦ 1 : general term
¦
¥
1
(d) The series representing ¨0 f (x )dx is a geometric series.
1
1
3 1
Therefore, ¨0 f (x )dx 
1

2
. 1 : answer
1
3

Copyright © 2001 by College Entrance Examination Board. All rights reserved.


Advanced Placement Program and AP are registered trademarks of the College Entrance Examination Board.

7
AP® CALCULUS BC 2002 SCORING GUIDELINES

Question 1

Let B and C be the functions given by B N   A N and C (N )  ln N .



(a) Find the area of the region enclosed by the graphs of B and C between N  and N  1.

(b) Find the volume of the solid generated when the region enclosed by the graphs of B and C between

N  and N   is revolved about the line O  4.

(c) Let D be the function given by D(N )  B (N )  C(N ). Find the absolute minimum value of DN  on the
1
closed interval > N > 1, and find the absolute maximum value of DN  on the closed interval
2
1
> N > 1. Show the analysis that leads to your answers.
2



¦ 1 : integral
£
(a) Area = ¨ A N  ln N  @N = 1.222 or 1.223 2 ¦¤
¦ 1 : answer
¦
¥

 
 1 : limits and constant
(b) Volume = 3 ¨   4  ln N    4  A N   @N 


 2 : integrand


= 7.5153 or 23.609 
  1  each error


4  Note: 0 / 2 if not of the form



 >

 k ¨  R(x )  r (x ) dx

 =

 1 : answer



 £
¦ 1 : considers D a(N )  0
(c) D N   B N   C N   A N   ¦
¦
N ¦ 1 : identifies critical point
¦
¦
N  0.567143 3 ¤
¦
¦ and endpoints as candidates
¦
¦
Absolute minimum value and absolute ¦
¦ 1 : answers
¦
¥
maximum value occur at the critical point or
at the endpoints. Note: Errors in computation come off
the third point.
D(0.567143)  2.330
D(0.5)  2.3418
D(1)  2.718

The absolute minimum is 2.330.


The absolute maximum is 2.718.

Copyright © 2002 by College Entrance Examination Board. All rights reserved.


Advanced Placement Program and AP are registered trademarks of the College Entrance Examination Board.

2
AP® CALCULUS BC 2002 SCORING GUIDELINES

Question 2

The rate at which people enter an amusement park on a given day is modeled by the function E defined by
15600
E t   .
 t  24t  160 
The rate at which people leave the same amusement park on the same day is modeled by the function L defined by
9890
L t   .
 t  38t  370 
Both -  J  and L  t  are measured in people per hour and time t is measured in hours after midnight. These
functions are valid for 9 > J > 23, the hours during which the park is open. At time J  9, there are no people in
the park.
(a) How many people have entered the park by 5:00 P.M. ( J  % )? Round answer to the nearest whole number.
(b) The price of admission to the park is $15 until 5:00 P.M. ( J  % ). After 5:00 P.M., the price of admission to
the park is $11. How many dollars are collected from admissions to the park on the given day? Round your
answer to the nearest whole number.
J
(c) Let H  t   ¨'  E x   Lx  dx for 9 > J > 23. The value of 0  %  to the nearest whole number is 3725.

Find the value of 0 =  %  and explain the meaning of 0  %  and 0 =  %  in the context of the park.
(d) At what time t, for 9 > J > 23, does the model predict that the number of people in the park is a maximum?

%
£
¦
(a) ¨' - (J ) @J  6004.270 ¦
¦
1 : limits
¦
6004 people entered the park by 5 pm. 3 ¤ 1 : integrand
¦
¦
¦
¦ 1 : answer
¦
¥
% !
(b) 15 ¨ - (J ) @J  11¨ - (J ) @J  104048.165
' % 1 : setup
The amount collected was $104,048.
or
!
¨% - (J ) @J  1271.283
1271 people entered the park between 5 pm and
11 pm, so the amount collected was
$15  (6004)  $11  (1271)  $104, 041.

(c) H (17)  E (17)  L(17)  380.281 


 1 : value of 0 (17)


There were 3725 people in the park at t = 17. 
 2 : meanings


 1 : meaning of 0 (17)
The number of people in the park was decreasing 3 



at the rate of approximately 380 people/hr at 
 1 : meaning of 0 (17)


time t = 17. 
  1  if no reference to J  17


(d) H t   E t   Lt     1 : E (t )  L(t )  0



2 
t = 15.794 or 15.795 
 1 : answer

Copyright © 2002 by College Entrance Examination Board. All rights reserved.
Advanced Placement Program and AP are registered trademarks of the College Entrance Examination Board.

3
AP® CALCULUS BC 2002 SCORING GUIDELINES

Question 3


The figure above shows the path traveled by a roller coaster


car over the time interval  > J > & seconds. The position
of the car at time J seconds can be modeled parametrically
by N  J   10J  4 sin J, O  J    20  J  1  cos J ,
where N and O are measured in meters. The derivatives of these functions are given by
N   J   10  4 cos J, O   J    20  J  sin J  cos J  1.
(a) Find the slope of the path at time J  2. Show the computations that lead to your answer.
(b) Find the acceleration vector of the car at the time when the car’s horizontal position is N = 140.
(c) Find the time J at which the car is at its maximum height, and find the speed, in m/sec, of the car at
this time.
(d) For 0  J  18, there are two times at which the car is at ground level  O  0  . Find these two times
and write an expression that gives the average speed, in m/sec, of the car between these two times. Do
not evaluate the expression.


@O O (2) 18 sin 2  cos 2  1 @O @O @N


(a) Slope =   1 : answer using 
@N J N (2) 10  4 cos 2 @N @J @J
= 1.793 or 1.794

(b) N (J )  10J  4 sin J  140; J  13.647083 £


¦ 1 : identifies acceleration vector
¦
¦
N   J    3.529, O   J    1.225 or 1.226 ¦
¦ as derivative of velocity vector
2 ¦
¤
Acceleration vector is 3.529,1.225  ¦
¦ 1 : computes acceleration vector
¦
or 3.529,1.226  ¦
¦ when N  140
¦
¥

(c) O (J )   20  J  sin J  cos J  1  0 £


¦ 1 : sets O a(J )  0
¦
¦
J = 3.023 or 3.024 at maximum height ¦
3 ¤ 1 : selects first J  0
¦
¦
Speed =  N   J     O   J    N   J  ¦
¦
¦
¥
1 : speed
= 6.027 or 6.028

(d) OJ    when J  23 and J  43 £


¦ 1 : J  23, J  43
¦
¦
 "3 ¦
3¨3
Average speed =  N J     O J   @J 3 ¤ 1 : limits and constant
¦
¦
¦
1 "3 ¦
¦ 1 : integrand
¥
23 ¨ 3
  10  4 cos J    20  J  sin J  cos J  1 @J

Copyright © 2002 by College Entrance Examination Board. All rights reserved.


Advanced Placement Program and AP are registered trademarks of the College Entrance Examination Board.

4
AP® CALCULUS BC 2002 SCORING GUIDELINES

Question 4


The graph of the function B shown above consists of two line segments. Let C be the
N
function given by C (N )  ¨ B  J @J.
(a) Find C  1 , C   1 , and C   1 .
(b) For what values of N in the open interval  2,2  is C increasing? Explain your
reasoning.
(c) For what values of N in the open interval  2,2  is the graph of C concave

down? Explain your reasoning.


(d) On the axes provided, sketch the graph of C on the closed interval  2,2  .
 


  ! 
 1 : C (1)

(a) C   ¨ B J  @J  ¨

B J  @J   

3 
 1 : C (1)
C   B    


 1 : C (1)
C   B   ! 


(b) C is increasing on   N   because ¦ 1 : interval


£
2 ¦¤
C N   B N    on this interval. ¦
¦ 1 : reason
¥

(c) The graph of C is concave down on   N 


because C N   B N    on this interval. ¦ 1 : interval
£
2 ¦¤
or ¦
¦ 1 : reason
¥
because C N   B N  is decreasing on this
interval.

(d)


 1 : C (2)  C (0)  C(2)  0



 1 : appropriate increasing/decreasing
2 

 and concavity behavior



  1  vertical asymptote


Copyright © 2002 by College Entrance Examination Board. All rights reserved.


Advanced Placement Program and AP are registered trademarks of the College Entrance Examination Board.

5
AP® CALCULUS BC 2002 SCORING GUIDELINES

Question 5
@O
Consider the differential equation  2O  4N .
@N
(a) The slope field for the given differential equation is provided. Sketch the solution curve that passes through
the point (0,1) and sketch the solution curve that passes through the point (0, 1) .
(b) Let B be the function that satisfies the given differential equation with the initial condition B  0   1. Use
Euler’s method, starting at N   with a step size of 0.1, to approximate B  0.2 . Show the work that leads
to your answer.
(c) Find the value of > for which O  N  > is a solution to the given differential equation. Justify your
answer.
(d) Let C be the function that satisfies the given differential equation with the initial condition C  0   0. Does
the graph of C have a local extremum at the point (0, 0)? If so, is the point a local maximum or a local
minimum? Justify your answer.


(a)  1 : solution curve through (0,1)



2 

 1 : solution curve through (0, 1)

Curves must go through the indicated
points, follow the given slope lines, and
extend to the boundary of the slope field.

 

(b) B (0.1)  B (0)  B (0)(0.1) £


¦ 1 : Euler’s method equations or
¦
¦
¦
¦ equivalent table applied to (at least)
= 1   2  0  (0.1)  1.2 ¦
¦
2 ¦
¤ two iterations
B (0.2)  B (0.1)  B (0.1)(0.1) ¦
¦
¦
¦ 1 : Euler approximation to B (0.2)
 1.2   2.4  0.4  (0.1)  1.4 ¦
¦
¦
¦ (not eligible without first point)
¥
 
(c) Substitute O  N  > in the DE: 
 @

 1 : uses  2N  >   2 in DE
  N  >   "N  > , so > = 1 2  @N

 1: > 1
OR 

Guess >  1, O  N  
@O
Verify: O  "N  "N    "N   .
@N
 

(d) C has local maximum at (0, 0).  1 : C (0)  0





@O 
C (0)   2(0)  4(0)  0, and 3  1 : shows C (0)  4
@N (0,0) 


 1 : conclusion
@ O @O 

C (N )  2  4, so
@N @N
C (0)  2 C (0)  4  4  0.


Copyright © 2002 by College Entrance Examination Board. All rights reserved.


Advanced Placement Program and AP are registered trademarks of the College Entrance Examination Board.

6
AP® CALCULUS BC 2002 SCORING GUIDELINES

Question 6

The Maclaurin series for the function B is given by


@ $x "
 x n  "x &x !  x n 
f x   œ
n 
n 
 x 
!

"
L
n 
L

on its interval of convergence.


(a) Find the interval of convergence of the Maclaurin series for B. Justify your answer.
(b) Find the first four terms and the general term for the Maclaurin series for B =(N ).

(c) Use the Maclaurin series you found in part (b) to find the value of B    13 .


 2x n  
 1 : sets up ratio


n 2 (n  1)  1 : computes limit of ratio
(a) lim  lim 2x  2x 

n @  2x n  n @ (n  2) 

 1 : identifies interior of interval
n 1 


1 1  of convergence
N   for  x  

2 2  2 : analysis/conclusion at endpoints

@
 5 


At N  , the series is œ which diverges since 

 1 : right endpoint
n  n   


 1 : left endpoint
this is the harmonic series. 
@ 

   1
At N   , the series is œ   n  which 

 1  if endpoints not N  
2
n  
n  
  1  if multiple interval s


converges by the Alternating Series Test.
1 1
Hence, the interval of convergence is   x  .
2 2

(b) B N    "N  &N  $N !  K   N n  K ¦ 1 : first 4 terms


£
2 ¦¤
¦
¦ 1 : general term
¥

(c) The series in (b) is a geometric series.



 1
         1: substitutes N   into infinite
n 
    
B   "  &  K   K  3
! ! ! ! 
 series from (b) or expresses series
2 

 
n
" & $ 
    K  K  from (b) in closed form
! ' % ! 


 1: answer for student's series
$ 

 
 #
!
OR
2 1 1
f (x )  for   x  . Therefore,
1  2x 2 2

 

B  
! 

$
#
!
Copyright © 2002 by College Entrance Examination Board. All rights reserved.
Advanced Placement Program and AP are registered trademarks of the College Entrance Examination Board.

7
AP® CALCULUS BC
2002 SCORING GUIDELINES (Form B)
Question 1

A particle moves in the NO-plane so that its position at any time J, for 3  J  3 , is given by
N (J )  sin(3J ) and OJ   J .
(a) Sketch the path of the particle in the NO-plane provided. Indicate the direction of motion
along the path.
(b) Find the range of N J  and the range of OJ  .
(c) Find the smallest positive value of J for which the N-coordinate of the particle is a local
maximum. What is the speed of the particle at this time?
(d) Is the distance traveled by the particle from J  3 to J  3 greater than #3 ? Justify
your answer.
 

(a)

 1 : graph



 three cycles of sine


2 
 N between  1 and 1



 O between  23 and 23



 1 : direction


(b)   N J    ¦ 1 : closed interval for N (J )


£
2 ¦¤
 3  OJ   3 ¦
¦ 1 : closed interval for O(J )
¥

(c) N a(J )  3 cos 3J  0


3 3 £
¦ 1 : N a(J )  3 cos 3J  0
!J  ; J  ¦
¦
$ ¦
3 ¤ 1 : solves for J
Speed = 9 cos (3J )  4 ¦
¦
¦
¦ 1 : speed at student’s time
3 ¦
¥
At J  ,
$
3
Speed = 9 cos
2  
4 =2

3
¦ 1 : integral for distance
£
(d) Distance = ¨3 9 cos (3J )  4 @J
2 ¦¤
¦
¦ 1 : conclusion with justification
¥
= 17.973 > #3 

Copyright © 2002 by College Entrance Examination Board. All rights reserved.


Advanced Placement Program and AP are registered trademarks of the College Entrance Examination Board.

2
AP® CALCULUS BC
2002 SCORING GUIDELINES (Form B)
Question 2

The number of gallons, 2 J  , of a pollutant in a lake changes at the rate 2 J     !A 0.2 J

gallons per day, where J is measured in days. There are 50 gallons of the pollutant in the lake at
time J = 0. The lake is considered to be safe when it contains 40 gallons or less of pollutant.
(a) Is the amount of pollutant increasing at time J = 9 ? Why or why not?
(b) For what value of J will the number of gallons of pollutant be at its minimum? Justify your
answer.
(c) Is the lake safe when the number of gallons of pollutant is at its minimum? Justify your
answer.
(d) An investigator uses the tangent line approximation to 2 J  at J = 0 as a model for the
amount of pollutant in the lake. At what time J does this model predict that the lake
becomes safe?

1 : answer with reason


(a) 2 (9)  1  3A 0.6  0.646 < 0
so the amount is not increasing at this time.

(b) 2 J     !A 0.2 J


 £
¦ 1 : sets 2 a(J )  0
¦
¦
¦
J =  5 ln 3  = 30.174 3 ¤ 1 : solves for J
¦
¦
2 =J  is negative for 0 < J <  5 ln 3  and positive ¦
¦ 1 : justification
¦
¥
for J >  5 ln 3  . Therefore there is a minimum at
J =  5 ln 3  .

30.174
 1  3A0.2 J @J £ 1 : integrand
(c) 2 (30.174)  50  ¨0 ¦
¦
¦
¦
¦ 1 : limits
= 35.104 < 40, so the lake is safe. 3 ¦¤
¦
¦ 1 : conclusion with reason
¦
¦
¦
¦ based on integral of 2 a(J )
¥

(d) 2     !   . The lake will become safe ¦ 1 : slope of tangent line
£
2 ¦¤
when the amount decreases by 10. A linear model ¦
¦ 1 : answer
¥
predicts this will happen when J = 5.

Copyright © 2002 by College Entrance Examination Board. All rights reserved.


Advanced Placement Program and AP are registered trademarks of the College Entrance Examination Board.

3
AP® CALCULUS BC
2002 SCORING GUIDELINES (Form B)
Question 3

Let 4 be the region in the first quadrant bounded by the O-axis and
3
the graphs of O  "N  N !   and O  N.
4
(a) Find the area of 4.
(b) Find the volume of the solid generated when 4 is revolved
about the N-axis.
(c) Write an expression involving one or more integrals that gives
the perimeter of 4. Do not evaluate.

Region 4
!
"N  N !    N when N  1.94045 = )
"

£
¦ 1 : limits
 !
 ¦
)
(a) Area = ¨ "N  N !    N @N ¦
" 3 ¦¤ 1 : integrand
¦
¦
= 4.514 or 4.515 ¦
¦ 1 : answer
¥

(b) Volume £
¦ 1 : limits and constant
¦
¦
¦
  @N
) ! 3 ¤ 1 : integrand
= 3 ¨   "N  N !     N ¦
 
 " ¦
¦
¦
¦ 1 : answer
¥
= 18.2913 or 57.463

(c) Perimeter = 1  (1.940)  (1.455) 


 1 : uses O   4  3N in integrand



) 3  1 : arc length integral
¨   "  !N  @N 

 
 1 : answer



Copyright © 2002 by College Entrance Examination Board. All rights reserved.


Advanced Placement Program and AP are registered trademarks of the College Entrance Examination Board.

4
AP® CALCULUS BC
2002 SCORING GUIDELINES (Form B)
Question 4

The graph of a differentiable function B on the closed


interval < 3,15 > is shown in the figure above. The graph of
B has a horizontal tangent line at N = 6. Let
N
CN   #  ¨$ B J  @J for !  N  # .
(a) Find C$ , C =$ , and C aa$ .
(b) On what intervals is C decreasing? Justify your answer.
(c) On what intervals is the graph of C concave down? Justify your answer.
#
(d) Find a trapezoidal approximation of ¨! B J @J using six subintervals of length J  ! .

$ £
¦ 1 : C (6)
(a) C $  #  ¨$ B J  @J  # ¦
¦
¦
3 ¦
¤ 1 : C a(6)
C ¢$ = B $ = ! ¦
¦
¦
¦ 1 : C aa(6)
C ¢¢$ = B ¢$ =  ¦
¥

(b) C is decreasing on < 3, 0 > and < 12,15 > since 
 1 : < 3, 0 >


 C aN   B N    for N   and N  12. 3  1 : < 12,15 >



 1 : justification


(c) The graph of C is concave down on (6,15) since ¦ 1 : interval


£
2 ¦¤
 C ¢ = B is decreasing on this interval. ¦
¦ 1 : justification
¥

! 1 : trapezoidal method
(d)        !       
= 12

Copyright © 2002 by College Entrance Examination Board. All rights reserved.


Advanced Placement Program and AP are registered trademarks of the College Entrance Examination Board.

5
AP® CALCULUS BC
2002 SCORING GUIDELINES (Form B)
Question 5

@O !N
Consider the differential equation  .
@N O
(a) Let O  B N  be the particular solution to the given differential equation for   N  #
such that the line O   is tangent to the graph of B. Find the N-coordinate of the point of
tangency, and determine whether B has a local maximum, local minimum, or neither at this
point. Justify your answer.
(b) Let O  CN  be the particular solution to the given differential equation for   N  & ,
with the initial condition C $  " . Find O  CN  .

@O
(a)   when N = 3
@N £
¦ 1:N 3
¦
¦
@ 2O O  O !  N   ¦
2   , 3 ¤ 1 : local minimum
@N (3,2) O2 (3,2) ¦
¦
¦
¦ 1 : justification
so B has a local minimum at this point. ¦
¥

or
Because B is continuous for   N  # , there
is an interval containing N = 3 on which
@O
O< 0. On this interval, is negative to
@N
@O
the left of N = 3 and is positive to the
@N
right of N = 3. Therefore B has a local
minimum at N = 3.

(b) O @O  !  N  @N 
 1 : separates variables


  
 1 : antiderivative of @O term
O  !N  N  + 


 1 : antiderivative of @N term
6 

 1 : constant of integration
&  &  &  + ; + = 8 


 1 : uses initial condition C (6)  4



 1 : solves for O
O  $N  N  $ 
Note: max 3/6 [1-1-1-0-0-0] if no constant
O   $N  N  $
of integration
Note: 0/6 if no separation of variables

Copyright © 2002 by College Entrance Examination Board. All rights reserved.


Advanced Placement Program and AP are registered trademarks of the College Entrance Examination Board.

6
AP® CALCULUS BC
2002 SCORING GUIDELINES (Form B)
Question 6
d
The Maclaurin series for ln  1
1N  is
xn
œ with interval of convergence   N   .
n  n

(a) Find the Maclaurin series for ln  1 1 3N  and determine the interval of convergence.
d
n
(b) Find the value of œ .
n  n
d d
n 
(c) Give a value of F such that œ p converges, but œn p diverges. Give reasons why
n  n n 
your value of F is correct.
d d
1 
(d) Give a value of F such that œ n p diverges, but œn p converges. Give reasons why your
n  n 
value of F is correct.

(a) ln  1 1 3N   ln  1  (13N )  ¦ 1 : series


£
2 ¦¤
d
(3x )n
d
3n n ¦
¦ 1 : interval of convergence
¥
= œ n or œ (1)n n
x
n  n 

We must have   !N   , so interval


1 1
of convergence is   x  .
3 3
d
1 : answer
(b)
(1)n  1  1
œ n  ln  1  (1)   ln 2
n 

 

(c) Some F such that   F b because  1 : correct p


d 
 (1)n
n
œ p converges by AST, but the 3  1 : reason why
 œ n p converges
n  n 
 1

d
 



1 : reason why œ n p diverges
F-series œn p diverges for F b  .
n 

 £
¦
(d) Some F such that  F b  because the ¦ 1 : correct p
¦
¦
d ¦
¦ 1
1
F-series œ np diverges for p  1 and the 3 ¤ 1 : reason why
¦ œ nF diverges
n  ¦
¦
¦ 1
d
 ¦
¦
¦
¥
1 : reason why œn F converges
F-series œ p converges for F   .
n  n

Copyright © 2002 by College Entrance Examination Board. All rights reserved.


Advanced Placement Program and AP are registered trademarks of the College Entrance Examination Board.

7
AP® CALCULUS BC
2004 SCORING GUIDELINES

Question 1

Traffic flow is defined as the rate at which cars pass through an intersection, measured in cars per minute.
The traffic flow at a particular intersection is modeled by the function F defined by
t
F t ! 82  4sin for 0 t 30,
2
where F t is measured in cars per minute and t is measured in minutes.
(a) To the nearest whole number, how many cars pass through the intersection over the 30-minute
period?
(b) Is the traffic flow increasing or decreasing at t ! 7 ? Give a reason for your answer.
(c) What is the average value of the traffic flow over the time interval 10 t 15 ? Indicate units of
measure.
(d) What is the average rate of change of the traffic flow over the time interval 10 t 15 ? Indicate
units of measure.

30 1 : limits
(a) F t dt ! 2474 cars
0 3: 1 : integrand
1 : answer

(b) F 7 ! n1.872 or n1.873 1 : answer with reason


Since F 7 0, the traffic flow is decreasing
at t ! 7.

1 15 1 : limits
(c) F t dt ! 81.899 cars min
5 10 3: 1 : integrand
1 : answer

F 15 n F 10 1 : answer
(d) ! 1.517 or 1.518 cars min 2
15 n 10

Units of cars min in (c) and cars min 2 in (d) 1 : units in (c) and (d)

Copyright © 2004 by College Entrance Examination Board. All rights reserved.


Visit apcentral.collegeboard.com (for AP professionals) and www.collegeboard.com/apstudents (for AP students and parents).

2
AP® CALCULUS BC
2004 SCORING GUIDELINES
Question 2

Let f and g be the functions given by f x ! 2 x 1 n x and


g x ! 3 x n1 x for 0 x 1. The graphs of f and g are shown in the
figure above.
(a) Find the area of the shaded region enclosed by the graphs of f and g.
(b) Find the volume of the solid generated when the shaded region enclosed
by the graphs of f and g is revolved about the horizontal line y ! 2.
(c) Let h be the function given by h x ! k x 1 n x for 0 x 1. For each
k " 0, the region (not shown) enclosed by the graphs of h and g is the
base of a solid with square cross sections perpendicular to the x-axis.
There is a value of k for which the volume of this solid is equal to 15.
Write, but do not solve, an equation involving an integral expression that
could be used to find the value of k.

1 1 : integral
(a) Area ! f x ng x dx 2:
0 1 : answer
1
! 2x 1 n x n 3 x n 1 x dx ! 1.133
0

1 2 2 1 : limits and constant


(b) Volume ! 2ng x n 2n f x dx
0 2 : integrand
1 2
! 2 n 3 x n1 x n 2 n 2x 1 n x 2
dx n1 each error
0 4: Note: 0 2 if integral not of form
! 16.179 b
c R2 x n r 2 x dx
a
1 : answer

1 2 2 : integrand
(c) Volume ! h x ng x dx 3:
0 1 : answer
1 2
kx 1n x n 3 x n1 x dx ! 15
0

Copyright © 2004 by College Entrance Examination Board. All rights reserved.


Visit apcentral.collegeboard.com (for AP professionals) and www.collegeboard.com/apstudents (for AP students and parents).

3
AP® CALCULUS BC
2004 SCORING GUIDELINES
Question 3

An object moving along a curve in the xy-plane has position x t , y t at time t 0 with
dx dy
! 3  cos t 2 . The derivative is not explicitly given. At time t ! 2, the object is at position
dt dt
1, 8 .
(a) Find the x-coordinate of the position of the object at time t ! 4.
dy
(b) At time t ! 2, the value of is n7. Write an equation for the line tangent to the curve at the point
dt
x 2 ,y 2 .
(c) Find the speed of the object at time t ! 2.
(d) For t 3, the line tangent to the curve at x t , y t has a slope of 2t  1. Find the acceleration
vector of the object at time t ! 4.

4 4
(a) x 4 ! x 2  3  cos t 2 dt 1: 3  cos t 2 dt
2 2
3:
4 1 : handles initial condition
!1 3  cos t 2 dt ! 7.132 or 7.133
2 1 : answer

(b) dy dy
dy n7 1 : finds
! dt ! ! n2.983 2: dx t !2
dx t !2 dx 3  cos 4
1 : equation
dt t !2
y n 8 ! n2.983 x n 1

(c) The speed of the object at time t ! 2 is 1 : answer


2 2
x 2  y 2 ! 7.382 or 7.383.

(d) x 4 ! 2.303 1:x 4


dy dy dx dy
y t 2t 1 3 cos t 2 3: 1:
dt dx dt dt
y 4 ! 24.813 or 24.814 1 : answer
The acceleration vector at t ! 4 is
2.303, 24.813 or 2.303, 24.814 .

Copyright © 2004 by College Entrance Examination Board. All rights reserved.


Visit apcentral.collegeboard.com (for AP professionals) and www.collegeboard.com/apstudents (for AP students and parents).

4
AP® CALCULUS BC
2004 SCORING GUIDELINES
Question 4

Consider the curve given by x 2  4 y 2 ! 7  3 x y.

dy 3 y n 2 x
(a) Show that ! .
dx 8 y n 3 x
(b) Show that there is a point P with x-coordinate 3 at which the line tangent to the curve at P is
horizontal. Find the y-coordinate of P.
d2y
(c) Find the value of at the point P found in part (b). Does the curve have a local maximum, a
dx 2
local minimum, or neither at the point P ? Justify your answer.

(a) 2 x  8 y y ! 3 y  3x y 1 : implicit differentiation


2:
8 y n 3x y ! 3 y n 2 x 1 : solves for y
3y n 2x
y !
8 y n 3x

3 y n 2x dy
(b) ! 0; 3 y n 2 x ! 0 1: !0
8 y n 3x dx
3:
1 : shows slope is 0 at 3, 2
When x ! 3, 3 y ! 6 1 : shows 3, 2 lies on curve
y!2

32 4 22 25 and 7 332 25

Therefore, P ! 3, 2 is on the curve and the slope


is 0 at this point.

(c) d2y 8 y n 3x 3 y n 2 n 3 y n 2 x 8 y n 3 d2y


! 2:
dx 2 8 y n 3x 2
dx 2
4: d2y
d2y 16 n 9 n2 2 1 : value of at 3, 2
At P ! 3, 2 , ! !n .
dx 2
16 n 9 2 7 dx 2
1 : conclusion with justification
Since y ! 0 and y 0 at P, the curve has a local
maximum at P.

Copyright © 2004 by College Entrance Examination Board. All rights reserved.


Visit apcentral.collegeboard.com (for AP professionals) and www.collegeboard.com/apstudents (for AP students and parents).

5
AP® CALCULUS BC
2004 SCORING GUIDELINES
Question 5

A population is modeled by a function P that satisfies the logistic differential equation


dP P P
! 1n .
dt 5 12
(a) If P 0 ! 3, what is lim P t ?
t
If P 0 ! 20, what is lim P t ?
t
(b) If P 0 ! 3, for what value of P is the population growing the fastest?
(c) A different population is modeled by a function Y that satisfies the separable differential equation
dY Y t
! 1n .
dt 5 12
Find Y t if Y 0 ! 3.
(d) For the function Y found in part (c), what is lim Y t ?
t

(a) For this logistic differential equation, the carrying 1 : answer


capacity is 12. 2:
1 : answer
If P 0 ! 3, lim P t ! 12.
t
If P 0 ! 20, lim P t ! 12.
t

(b) The population is growing the fastest when P is half 1 : answer


the carrying capacity. Therefore, P is growing the
fastest when P ! 6.

1 1 t 1 t 1 : separates variables
(c) dY ! 1 n dt ! n dt
Y 5 12 5 60 1 : antiderivatives
t t2 1 : constant of integration
ln Y ! n C 5:
5 120 1 : uses initial condition
t t2 1 : solves for Y
n
Y t ! K e5 120
0 1 if Y is not exponential
K !3
t t2
n Note: max 2 5 [1-1-0-0-0] if no
Y t ! 3e 120
5
constant of integration
Note: 0 5 if no separation of variables

(d) lim Y t ! 0 1 : answer


t
0 1 if Y is not exponential

Copyright © 2004 by College Entrance Examination Board. All rights reserved.


Visit apcentral.collegeboard.com (for AP professionals) and www.collegeboard.com/apstudents (for AP students and parents).

6
AP® CALCULUS BC
2004 SCORING GUIDELINES
Question 6

Let f be the function given by f x ! sin 5 x  , and let P x be the third-degree Taylor polynomial
4
for f about x ! 0.
(a) Find P x .

(b) Find the coefficient of x 22 in the Taylor series for f about x ! 0.


1 1 1
(c) Use the Lagrange error bound to show that f nP .
10 10 100
x
(d) Let G be the function given by G x ! f t dt. Write the third-degree Taylor polynomial
0
for G about x ! 0.

2 4: P x
(a) f 0 ! sin !
2 4
n1 each error or missing term
5 2
f 0 ! 5cos !
4 2 deduct only once for sin 4
25 2 evaluation error
f 0 ! n25sin !n
4 2
125 2 deduct only once for cos 4
f 0 ! n125cos !n
4 2 evaluation error
2 5 2 25 2 2 125 2 3 n1 max for all extra terms,  ,
P x !  xn x n x
2 2 2 2! 2 3!
misuse of equality

n522 2 1 : magnitude
(b) 2:
2 22! 1 : sign

4
1 1 4 1 1
(c) f P max f c 1 : error bound in an appropriate
10 10 0 c 1 4! 10 inequality
10
4
625 1 1 1
4! 10 384 100

(d) The third-degree Taylor polynomial for G about 2 : third-degree Taylor polynomial for G
x
2 5 2 25 2 2 about x ! 0
x ! 0 is  tn t dt
0 2 2 4
n1 each incorrect or missing term
2 5 2 2 25 2 3
! x x n x n1 max for all extra terms,  ,
2 4 12
misuse of equality

Copyright © 2004 by College Entrance Examination Board. All rights reserved.


Visit apcentral.collegeboard.com (for AP professionals) and www.collegeboard.com/apstudents (for AP students and parents).

7
AP® CALCULUS BC
2004 SCORING GUIDELINES (Form B)
Question 1

A particle moving along a curve in the plane has position x t , y t at time t, where
dx dy
! t 4  9 and ! 2et  5en t
dt dt
for all real values of t. At time t ! 0, the particle is at the point (4, 1).
(a) Find the speed of the particle and its acceleration vector at time t ! 0.
(b) Find an equation of the line tangent to the path of the particle at time t ! 0.
(c) Find the total distance traveled by the particle over the time interval 0 t 3.
(d) Find the x-coordinate of the position of the particle at time t ! 3.

(a) At time t ! 0: 1 : speed


2:
1 : acceleration vector
Speed ! x (0) 2  y (0) 2 ! 32  7 2 ! 58

Acceleration vector ! x 0 , y 0 ! 0, n3

dy y 0 7 1 : slope
(b) ! ! 2:
dx x 0 3 1 : tangent line
7
Tangent line is y ! x n 4 1
3

3 2 2 2 : distance integral
(c) Distance ! t4  9  2et  5e nt dt
0 n1 each integrand error
3:
! 45.226 or 45.227 n1 error in limits
1 : answer

3 1 : integral
(d) x 3 ! 4 t 4  9 dt 2:
0 1 : answer
! 17.930 or 17.931

Copyright © 2004 by College Entrance Examination Board. All rights reserved.


Visit apcentral.collegeboard.com (for AP professionals) and www.collegeboard.com/apstudents (for AP students and parents).

2
AP® CALCULUS BC
2004 SCORING GUIDELINES (Form B)
Question 2

Let f be a function having derivatives of all orders for all real numbers. The third-degree Taylor
polynomial for f about x ! 2 is given by T x ! 7 n 9 x n 2 2 n 3 x n 2 3 .
(a) Find f 2 and f 2 .
(b) Is there enough information given to determine whether f has a critical point at x ! 2 ?
If not, explain why not. If so, determine whether f 2 is a relative maximum, a relative minimum,
or neither, and justify your answer.
(c) Use T x to find an approximation for f 0 . Is there enough information given to determine
whether f has a critical point at x ! 0 ? If not, explain why not. If so, determine whether f 0 is a
relative maximum, a relative minimum, or neither, and justify your answer.
4
(d) The fourth derivative of f satisfies the inequality f x 6 for all x in the closed interval
0, 2 . Use the Lagrange error bound on the approximation to f 0 found in part (c) to explain why
f 0 is negative.

(a) f 2 !T 2 !7 1: f 2 ! 7
2:
f 2 1 : f 2 ! n18
! n9 so f 2 ! n18
2!

(b) Yes, since f 2 ! T 2 ! 0, f does have a critical 1 : states f 2 ! 0


point at x ! 2. 2: 1 : declares f 2 as a relative
Since f 2 ! n18 0, f 2 is a relative maximum maximum because f 2 0
value.

(c) f 0 T 0 5 1: f 0 T 0 5
It is not possible to determine if f has a critical point 1 : declares that it is not
3:
at x ! 0 because T x gives exact information only possible to determine
at x ! 2. 1 : reason

6 1 : value of Lagrange error


(d) Lagrange error bound ! 0n24 !4
4! 2: bound
f 0 T 0 4 1 1 : explanation
Therefore, f 0 is negative.

Copyright © 2004 by College Entrance Examination Board. All rights reserved.


Visit apcentral.collegeboard.com (for AP professionals) and www.collegeboard.com/apstudents (for AP students and parents).

3
AP® CALCULUS BC
2004 SCORING GUIDELINES (Form B)
Question 3

A test plane flies in a straight line with t (min) 0 5 10 15 20 25 30 35 40


positive velocity v t , in miles per
v t (mpm) 7.0 9.2 9.5 7.0 4.5 2.4 2.4 4.3 7.3
minute at time t minutes, where v is a
differentiable function of t. Selected
values of v t for 0 t 40 are shown in the table above.
(a) Use a midpoint Riemann sum with four subintervals of equal length and values from the table to
40
approximate v t dt. Show the computations that lead to your answer. Using correct units,
0
40
explain the meaning of v t dt in terms of the plane’s flight.
0
(b) Based on the values in the table, what is the smallest number of instances at which the acceleration
of the plane could equal zero on the open interval 0 t 40? Justify your answer.
t 7t
(c) The function f, defined by f t ! 6  cos  3sin , is used to model the velocity of the
10 40
plane, in miles per minute, for 0 t 40. According to this model, what is the acceleration of the
plane at t ! 23 ? Indicates units of measure.
(d) According to the model f, given in part (c), what is the average velocity of the plane, in miles per
minute, over the time interval 0 t 40?

(a) Midpoint Riemann sum is 1 : v 5  v 15  v 25  v 35


10 v 5 v 15 v 25 v 35 3: 1 : answer
10 9.2 7.0 2.4 4.3 229 1 : meaning with units
The integral gives the total distance in miles that the
plane flies during the 40 minutes.

(b) By the Mean Value Theorem, v t ! 0 somewhere in 1 : two instances


2:
the interval 0, 15 and somewhere in the interval 1 : justification
25, 30 . Therefore the acceleration will equal 0 for at
least two values of t.

(c) f 23 ! n0.407 or n0.408 miles per minute2 1 : answer with units

1 40 1 : limits
(d) Average velocity ! f t dt
40 0 3: 1 : integrand
! 5.916 miles per minute 1 : answer

Copyright © 2004 by College Entrance Examination Board. All rights reserved.


Visit apcentral.collegeboard.com (for AP professionals) and www.collegeboard.com/apstudents (for AP students and parents).

4
AP® CALCULUS BC
2004 SCORING GUIDELINES (Form B)
Question 4

The figure above shows the graph of f , the derivative of the function f,
on the closed interval 1 x 5. The graph of f has horizontal tangent
lines at x ! 1 and x ! 3. The function f is twice differentiable with
f 2 ! 6.
(a) Find the x-coordinate of each of the points of inflection of the graph
of f. Give a reason for your answer.
(b) At what value of x does f attain its absolute minimum value on the
closed interval 1 x 5 ? At what value of x does f attain its
absolute maximum value on the closed interval 1 x 5 ? Show
the analysis that leads to your answers.
(c) Let g be the function defined by g x ! x f x . Find an equation for the line tangent to the graph
of g at x ! 2.

(a) x ! 1 and x ! 3 because the graph of f changes from 1 : x ! 1, x ! 3


2:
increasing to decreasing at x ! 1, and changes from 1 : reason
decreasing to increasing at x ! 3.

(b) The function f decreases from x ! n1 to x ! 4, then 1 : indicates f decreases then increases
increases from x ! 4 to x ! 5. 1 : eliminates x ! 5 for maximum
Therefore, the absolute minimum value for f is at x ! 4. 4:
1 : absolute minimum at x ! 4
The absolute maximum value must occur at x ! n1 or
1 : absolute maximum at x ! n1
at x ! 5.
5
f 5 n f n1 ! f t dt 0
n1
Since f 5 f n1 , the absolute maximum value occurs
at x ! n1.

(c) g x ! f x  xf x 2:g x
3:
g 2 ! f 2  2 f 2 ! 6  2 n1 ! 4 1 : tangent line
g 2 ! 2 f 2 ! 12

Tangent line is y ! 4 x n 2  12

Copyright © 2004 by College Entrance Examination Board. All rights reserved.


Visit apcentral.collegeboard.com (for AP professionals) and www.collegeboard.com/apstudents (for AP students and parents).

5
AP® CALCULUS BC
2004 SCORING GUIDELINES (Form B)
Question 5

1
Let g be the function given by g x ! .
x
(a) Find the average value of g on the closed interval 1, 4 .

(b) Let S be the solid generated when the region bounded by the graph of y ! g x , the vertical lines
x ! 1 and x ! 4, and the x-axis is revolved about the x-axis. Find the volume of S.

(c) For the solid S, given in part (b), find the average value of the areas of the cross sections
perpendicular to the x-axis.
(d) The average value of a function f on the unbounded interval [ a, ) is defined to be
b
f x dx
lim a . Show that the improper integral g x dx is divergent, but the average value
b 4
bna
of g on the interval [4, ) is finite.

1 4
1 1 4
4 2 2 1 : integral
(a) dx 2 x
3 1 x 3 1 3 3 3 2: 1 : antidifferentiation
and evaluation

4
1 4 1 : integral
(b) Volume ! dx ! ln x ! ln 4
1 x 1 2: 1 : antidifferentiation
and evaluation

2
1 1 : answer
(c) The cross section at x has area !
x x
4
1 1
Average value ! dx ! ln 4
3 1 x 3

b b
1 1: g x dx ! 2 b n 4
(d) g x dx ! lim dx ! lim 2 b n 4 ! 4
4 b 4 x b
1 : indicates integral diverges
This limit is not finite, so the integral is divergent. 4:
1 b 2 b n4
1: g x dx !
b bn4 4 bn4
g x dx b 1 : finite limit as b
4 1 1 2 b n4
! dx !
bn4 bn4 4 x bn4

2 b n4
lim !0
b bn4

Copyright © 2004 by College Entrance Examination Board. All rights reserved.


Visit apcentral.collegeboard.com (for AP professionals) and www.collegeboard.com/apstudents (for AP students and parents).

6
AP® CALCULUS BC
2004 SCORING GUIDELINES (Form B)
Question 6

Let be the line tangent to the graph of y ! x n at the point (1, 1),
where n " 1, as shown above.
1
(a) Find x n dx in terms of n.
0
(b) Let T be the triangular region bounded by , the x-axis, and the
1
line x ! 1. Show that the area of T is .
2n
(c) Let S be the region bounded by the graph of y ! xn , the line ,
and the x-axis. Express the area of S in terms of n and determine
the value of n that maximizes the area of S.

1
1 n x n 1 1 1 : antiderivative of x n
(a) x dx ! ! 2:
0 n 1 0
n 1 1 : answer

(b) Let b be the length of the base of triangle T. 1 : slope of line is n


1
1 3: 1 : base of T is
is the slope of line , which is n n
b 1
1 : shows area is
2n
1 1
Area T ! b1 !
2 2n

1 n 1 : area of S in terms of n
(c) Area S ! x dx n Area T
0 1 : derivative
1 1 4:
! n 1 : sets derivative equal to 0
n  1 2n
1 : solves for n
d 1 1
Area S ! n  2 !0
dn ( n  1) 2
2n

2
2 n2 ! n  1

2n ! n 1

1
n! ! 1 2
2 n1

Copyright © 2004 by College Entrance Examination Board. All rights reserved.


Visit apcentral.collegeboard.com (for AP professionals) and www.collegeboard.com/apstudents (for AP students and parents).

7
AP® CALCULUS BC
2005 SCORING GUIDELINES

Question 1

1
Let f and g be the functions given by f x !  sin x and g x ! 4n x. Let
4
R be the shaded region in the first quadrant enclosed by the y-axis and the graphs
of f and g, and let S be the shaded region in the first quadrant enclosed by the
graphs of f and g, as shown in the figure above.
(a) Find the area of R.
(b) Find the area of S.
(c) Find the volume of the solid generated when S is revolved about the horizontal
line y ! n1.

1
f x ! g x when  sin x ! 4n x .
4
f and g intersect when x ! 0.178218 and when x ! 1.
Let a ! 0.178218.

a
(a) g x n f x dx ! 0.064 or 0.065 1 : limits
0
3: 1 : integrand
1 : answer

1
(b) f x ng x dx ! 0.410 1 : limits
a
3: 1 : integrand
1 : answer

1 2 2
(c) f x 1 n g x 1 dx ! 4.558 or 4.559 2 : integrand
a 3:
1 : limits, constant, and answer

Copyright © 2005 by College Board. All rights reserved.


Visit apcentral.collegeboard.com (for AP professionals) and www.collegeboard.com/apstudents (for AP students and parents).

2
AP® CALCULUS BC
2005 SCORING GUIDELINES

Question 2

The curve above is drawn in the xy-plane and is described by the equation in
polar coordinates r !  sin 2 for 0 , where r is measured in
meters and is measured in radians. The derivative of r with respect to is
dr
given by ! 1  2 cos 2 .
d
(a) Find the area bounded by the curve and the x-axis.
(b) Find the angle that corresponds to the point on the curve with
x-coordinate n2.
2 dr
(c) For , is negative. What does this fact say about r ? What does this fact say about the curve?
3 3 d

(d) Find the value of in the interval 0


that corresponds to the point on the curve in the first quadrant
2
with greatest distance from the origin. Justify your answer.

1
(a) Area ! r2 d 1 : limits and constant
2 0
3: 1 : integrand
1 2
!  sin 2 d ! 4.382 1 : answer
2 0

(b) n2 ! r cos !  sin 2 cos 1 : equation


2:
! 2.786 1 : answer

dr 2 1 : information about r
(c) Since 0 for , r is decreasing on this
d 3 3 2:
interval. This means the curve is getting closer to the origin. 1 : information about the curve

dr
(d) The only value in 0, where ! 0 is ! . 1: ! or 1.047
2 d 3 2: 3
1 : answer with justification
r
0 0

1.913
3

1.571
2

The greatest distance occurs when ! .


3

Copyright © 2005 by College Board. All rights reserved.


Visit apcentral.collegeboard.com (for AP professionals) and www.collegeboard.com/apstudents (for AP students and parents).

3
AP® CALCULUS BC
2005 SCORING GUIDELINES

Question 3

Distance
0 1 5 6 8
x (cm)
Temperature
T x ”C 100 93 70 62 55

A metal wire of length 8 centimeters (cm) is heated at one end. The table above gives selected values of the temperature
T x , in degrees Celsius ”C , of the wire x cm from the heated end. The function T is decreasing and twice
differentiable.
(a) Estimate T 7 . Show the work that leads to your answer. Indicate units of measure.
(b) Write an integral expression in terms of T x for the average temperature of the wire. Estimate the average temperature
of the wire using a trapezoidal sum with the four subintervals indicated by the data in the table. Indicate units of
measure.
8 8
(c) Find T x dx, and indicate units of measure. Explain the meaning of T x dx in terms of the temperature of the
0 0
wire.
(d) Are the data in the table consistent with the assertion that T x " 0 for every x in the interval 0 x 8 ? Explain
your answer.

T 8 nT 6 55 n 62 7
(a) ! ! n ”C cm 1 : answer
8n6 2 2

1 8 1 8
(b) T x dx 1: T x dx
8 0 8 0
3:
8 1 : trapezoidal sum
Trapezoidal approximation for T x dx :
0 1 : answer
100  93
93  70 70  62 62  55
A 1 4 1 2
2 2 2 2
1
Average temperature A 75.6875 C
8
8
(c) T x dx ! T 8 n T 0 ! 55 n 100 ! n 45”C 1 : value
0 2:
The temperature drops 45”C from the heated end of the wire to the 1 : meaning
other end of the wire.

70 n 93
(d) Average rate of change of temperature on 1, 5 is ! n5.75. 1 : two slopes of secant lines
5 n1 2:
62 n 70 1 : answer with explanation
Average rate of change of temperature on 5, 6 is ! n8.
6n5
No. By the MVT, T c1 ! n5.75 for some c1 in the interval 1, 5
and T c2 ! n8 for some c2 in the interval 5, 6 . It follows that
T must decrease somewhere in the interval c1 , c2 . Therefore T
is not positive for every x in 0, 8 .

Units of ”C cm in (a), and ”C in (b) and (c) 1 : units in (a), (b), and (c)

Copyright © 2005 by College Board. All rights reserved.


Visit apcentral.collegeboard.com (for AP professionals) and www.collegeboard.com/apstudents (for AP students and parents).

4
AP® CALCULUS BC
2005 SCORING GUIDELINES

Question 4
dy
Consider the differential equation ! 2 x n y.
dx
(a) On the axes provided, sketch a slope field for the given differential equation at the
twelve points indicated, and sketch the solution curve that passes through the point
0, 1 . (Note: Use the axes provided in the pink test booklet.)
3
(b) The solution curve that passes through the point 0, 1 has a local minimum at x ! ln . What is the
2
y-coordinate of this local minimum?
(c) Let y ! f x be the particular solution to the given differential equation with the initial condition
f 0 ! 1. Use Euler’s method, starting at x ! 0 with two steps of equal size, to approximate f n 0.4 .
Show the work that leads to your answer.
d2y
(d) Find in terms of x and y. Determine whether the approximation found in part (c) is less than or
dx 2
greater than f n 0.4 . Explain your reasoning.

(a) 1 : zero slopes


3: 1 : nonzero slopes
1 : curve through 0, 1

dy dy
(b) ! 0 when 2x ! y 1 : sets !0
dx 2: dx
3 1 : answer
The y-coordinate is 2ln .
2

(c) f 0.2 f 0 f 0 0.2 1 : Euler's method with two steps


2:
! 1  n1 n 0.2 ! 1.2 1 : Euler approximation to f n 0.4
f 0.4 f 0.2 f 0.2 0.2
1.2 1.6 0.2 1.52

d2y dy d2y
(d) ! 2n ! 2 n 2x  y 1:
dx 2 dx 2: dx 2
d2y 1 : answer with reason
is positive in quadrant II because x 0 and y " 0.
dx 2
1.52 f n0.4 since all solution curves in quadrant II are
concave up.
Copyright © 2005 by College Board. All rights reserved.
Visit apcentral.collegeboard.com (for AP professionals) and www.collegeboard.com/apstudents (for AP students and parents).

5
AP® CALCULUS BC
2005 SCORING GUIDELINES

Question 5

A car is traveling on a straight road. For 0 t 24 seconds, the car’s


velocity v t , in meters per second, is modeled by the piecewise-linear
function defined by the graph above.
24 24
(a) Find v t dt. Using correct units, explain the meaning of v t dt.
0 0

(b) For each of v 4 and v 20 , find the value or explain why it does not
exist. Indicate units of measure.

(c) Let a t be the car’s acceleration at time t, in meters per second per second. For 0 t 24, write a
piecewise-defined function for a t .
(d) Find the average rate of change of v over the interval 8 t 20. Does the Mean Value Theorem guarantee
a value of c, for 8 c 20, such that v c is equal to this average rate of change? Why or why not?

24 1 1
(a) v t dt ! 4 20  12 20  8 20 ! 360 1 : value
0 2 2 2:
The car travels 360 meters in these 24 seconds. 1 : meaning with units

(b) v 4 does not exist because 1 : v 4 does not exist, with explanation
v t nv 4 v t nv 4 3: 1 : v 20
lim 5 0 lim .
t 4 n tn4 t 4 tn4 1 : units
20 n 0 5
v 20 ! ! n m sec 2
16 n 24 2

(c) 5 if 0 t 4 5
1 : finds the values 5, 0, n
0 if 4 t 16 2: 2
a t !
5 1 : identifies constants with correct intervals
n if 16 t 24
2
a t does not exist at t ! 4 and t ! 16.

(d) The average rate of change of v on 8, 20 is 1 : average rate of change of v on 8, 20


2:
v 20 n v 8 5 1 : answer with explanation
! n m sec 2 .
20 n 8 6
No, the Mean Value Theorem does not apply to v on
8, 20 because v is not differentiable at t ! 16.

Copyright © 2005 by College Board. All rights reserved.


Visit apcentral.collegeboard.com (for AP professionals) and www.collegeboard.com/apstudents (for AP students and parents).

6
AP® CALCULUS BC
2005 SCORING GUIDELINES

Question 6

Let f be a function with derivatives of all orders and for which f 2 ! 7. When n is odd, the nth derivative
n n1 !
of f at x ! 2 is 0. When n is even and n 2, the nth derivative of f at x ! 2 is given by f n 2 ! .
3n
(a) Write the sixth-degree Taylor polynomial for f about x ! 2.
2n
(b) In the Taylor series for f about x ! 2, what is the coefficient of x n 2 for n 1?
(c) Find the interval of convergence of the Taylor series for f about x ! 2. Show the work that leads to your
answer.

1! 1 2 3! 1 4 5! 1 6
(a) P6 x 7 x 2 x 2 x 2 1 : polynomial about x ! 2
32 2! 34 4! 36 6!
2 : P6 x
3: n1 each incorrect term
n1 max for all extra terms,
 , misuse of equality

2n n 1 ! 1 1
(b) 1 : coefficient
32n 2n ! 2n
3 2n

(c) The Taylor series for f about x ! 2 is 1 : sets up ratio


1 1: computes limit of ratio
f x !7 x n 2 2n .
n !1 2n 3
2n 1: identifies interior of
1 1 2 n 1 5: interval of convergence
x 2
2 n  1 3 n 1
2 1 : considers both endpoints
L ! lim
n 1 1 1 : analysis/conclusion for
x 2 2n
2n 32n both endpoints
2
2n 32n xn2
lim x 2 2
n 2 n 1
2 2n
33 9
L 1 when x n 2 3.
Thus, the series converges when n1 x 5.
32n 1 1
When x ! 5, the series is 7  !7 ,
n !1 2 n 3
2n 2 n !1
n

1
which diverges, because , the harmonic series, diverges.
n !1
n

( n3) 2n 1 1
When x ! n1, the series is 7  !7 ,
n !1 2 n 3 2n 2 n !1 n

1
which diverges, because , the harmonic series, diverges.
n !1
n
The interval of convergence is n1, 5 .

Copyright © 2005 by College Board. All rights reserved.


Visit apcentral.collegeboard.com (for AP professionals) and www.collegeboard.com/apstudents (for AP students and parents).

7
AP® CALCULUS BC
2005 SCORING GUIDELINES (Form B)

Question 1

An object moving along a curve in the xy-plane has position x t , y t at time t 0 with

dx dy 4
! 12t n 3t 2 and ! ln 1  t n 4 .
dt dt
At time t ! 0, the object is at position n13, 5 . At time t ! 2, the object is at point P with
x-coordinate 3.
(a) Find the acceleration vector at time t ! 2 and the speed at time t ! 2.
(b) Find the y-coordinate of P.
(c) Write an equation for the line tangent to the curve at P.
(d) For what value of t, if any, is the object at rest? Explain your reasoning.

32
(a) x 2 ! 0, y 2 ! n ! n1.882 1 : acceleration vector
17 2:
a 2 ! 0, n1.882 1 : speed
2
Speed ! 122  ln 17 ! 12.329 or 12.330

t 4 2 4
(b) y t ! y 0  ln 1  u n 4 du 1: ln 1  u n 4 du
0 0
3:
2 4 1 : handles initial condition
y 2 ! 5 ln 1  u n 4 du ! 13.671
0 1 : answer

dy
ln 17
(c) At t ! 2, slope ! dt ! ! 0.236 1 : slope
dx 12 2:
dt 1 : equation
y n 13.671 ! 0.236 x n 3

(d) x t ! 0 if t ! 0, 4 1 : reason
2:
y t ! 0 if t ! 4 1 : answer
t !4

Copyright © 2005 by College Board. All rights reserved.


Visit apcentral.collegeboard.com (for AP professionals) and www.collegeboard.com/apstudents (for AP students and parents).

2
AP® CALCULUS BC
2005 SCORING GUIDELINES (Form B)

Question 2

A water tank at Camp Newton holds 1200 gallons of water at time t ! 0. During the time interval
0 t 18 hours, water is pumped into the tank at the rate
t
W t ! 95 t sin 2 gallons per hour.
6
During the same time interval, water is removed from the tank at the rate
t
R t ! 275sin 2 gallons per hour.
3
(a) Is the amount of water in the tank increasing at time t ! 15 ? Why or why not?
(b) To the nearest whole number, how many gallons of water are in the tank at time t ! 18 ?
(c) At what time t, for 0 t 18, is the amount of water in the tank at an absolute minimum? Show the
work that leads to your conclusion.
(d) For t " 18, no water is pumped into the tank, but water continues to be removed at the rate R t
until the tank becomes empty. Let k be the time at which the tank becomes empty. Write, but do not
solve, an equation involving an integral expression that can be used to find the value of k.

(a) No; the amount of water is not increasing at t ! 15 1 : answer with reason
since W 15 n R 15 ! n121.09 0.

18
(b) 1200  W t nR t dt ! 1309.788 1 : limits
0
3: 1 : integrand
1310 gallons
1 : answer

(c) W t nR t !0 1 : interior critical points


t ! 0, 6.4948, 12.9748 1 : amount of water is least at
3:
t (hours) gallons of water t ! 6.494 or 6.495
0 1200 1 : analysis for absolute minimum
6.495 525
12.975 1697
18 1310

The values at the endpoints and the critical points


show that the absolute minimum occurs when
t ! 6.494 or 6.495.

k
(d) R t dt ! 1310 1 : limits
18 2:
1 : equation

Copyright © 2005 by College Board. All rights reserved.


Visit apcentral.collegeboard.com (for AP professionals) and www.collegeboard.com/apstudents (for AP students and parents).

3
AP® CALCULUS BC
2005 SCORING GUIDELINES (Form B)

Question 3

The Taylor series about x ! 0 for a certain function f converges to f x for all x in the interval of
convergence. The nth derivative of f at x ! 0 is given by
n 1
n n1 n 1 
f 0 ! n 2
for n 2.
5 n n1

The graph of f has a horizontal tangent line at x ! 0, and f 0 ! 6.


(a) Determine whether f has a relative maximum, a relative minimum, or neither at x ! 0. Justify your
answer.
(b) Write the third-degree Taylor polynomial for f about x ! 0.
(c) Find the radius of convergence of the Taylor series for f about x ! 0. Show the work that leads to
your answer.

(a) f has a relative maximum at x ! 0 because 1 : answer


f 0 ! 0 and f 0 0. 2:
1 : reason

(b) f 0 ! 6, f 0 ! 0 3: P x
3! 6 4! n1 each incorrect term
f 0 !n 2 2 !n , f 0 !
5 1 25 3 2
5 2 Note: n1 max for use of extra terms
3! x 2 4! x3 3 2 1 3
P x !6n  !6n x  x
5 2!2 3 2
5 2 3! 25 125

n n 1
f 0 n1 n 1
(c) un ! xn ! xn 1 : general term
n! n
5 n n1 2
1 : sets up ratio
n1 n2
n2 4: 1 : computes limit
x n 1 1 : applies ratio test to get
un 1 5n 1 n 2
!
un n1 n 1 n  1 radius of convergence
n 2
xn
5 n n1
n  2 n n1 2 1
! x
n 1 n 5
un 1 1
lim u ! x 1 if x 5.
n n 5
The radius of convergence is 5.

Copyright © 2005 by College Board. All rights reserved.


Visit apcentral.collegeboard.com (for AP professionals) and www.collegeboard.com/apstudents (for AP students and parents).

4
AP® CALCULUS BC
2005 SCORING GUIDELINES (Form B)

Question 4

The graph of the function f above consists of three line


segments.
x
(a) Let g be the function given by g x ! f t dt .
n4
For each of g n1 , g n1 , and g n1 , find the
value or state that it does not exist.
(b) For the function g defined in part (a), find the
x-coordinate of each point of inflection of the graph
of g on the open interval n 4 x 3. Explain
your reasoning.

3
(c) Let h be the function given by h x ! f t dt. Find all values of x in the closed interval
x
4 x 3 for which h x ! 0.
(d) For the function h defined in part (c), find all intervals on which h is decreasing. Explain your
reasoning.

n1 1 15
(a) g n1 ! f t dt ! n 3 5 !n 1 : g n1
n4 2 2
3: 1 : g n1
g n1 ! f n1 ! n2
1 : g n1
g n1 does not exist because f is not differentiable
at x ! n1.

(b) x !1 1 : x ! 1 (only)
2:
g ! f changes from increasing to decreasing 1 : reason
at x ! 1.

(c) x ! n1, 1, 3 2 : correct values


n1 each missing or extra value

(d) h is decreasing on 0, 2 1 : interval


2:
h ! n f 0 when f " 0 1 : reason

Copyright © 2005 by College Board. All rights reserved.


Visit apcentral.collegeboard.com (for AP professionals) and www.collegeboard.com/apstudents (for AP students and parents).

5
AP® CALCULUS BC
2005 SCORING GUIDELINES (Form B)

Question 5

Consider the curve given by y 2 ! 2  xy.

dy y
(a) Show that ! .
dx 2 y n x
1
(b) Find all points x, y on the curve where the line tangent to the curve has slope .
2

(c) Show that there are no points x, y on the curve where the line tangent to the curve is horizontal.

(d) Let x and y be functions of time t that are related by the equation y 2 ! 2  xy. At time t ! 5, the
dy dx
value of y is 3 and ! 6. Find the value of at time t ! 5.
dt dt

(a) 2y y ! y  x y 1 : implicit differentiation


2:
2y n x y ! y 1 : solves for y
y
y !
2y n x

y 1 y 1
(b) ! 1: !
2y n x 2 2: 2y n x 2
2y ! 2y n x 1 : answer
x!0
y!• 2
0, 2 , 0, n 2

y
(c) !0 1: y ! 0
2y n x 2:
1 : explanation
y!0
The curve has no horizontal tangent since
02 2 x 0 for any x.

7
(d) When y ! 3, 32 ! 2  3x so x ! . 1 : solves for x
3
3: 1 : chain rule
dy dy dx y dx
dt dx dt 2 y n x dt 1 : answer
3 dx 9 dx
At t ! 5, 6
7 dt 11 dt
6n
3
dx 22
!
dt t !5 3

Copyright © 2005 by College Board. All rights reserved.


Visit apcentral.collegeboard.com (for AP professionals) and www.collegeboard.com/apstudents (for AP students and parents).

6
AP® CALCULUS BC
2005 SCORING GUIDELINES (Form B)

Question 6

Consider the graph of the function f given by


1
f x ! for x 0, as shown in the figure
x2
above. Let R be the region bounded by the
graph of f, the x- and y-axes, and the vertical
line x ! k , where k 0.

(a) Find the area of R in terms of k.


(b) Find the volume of the solid generated when
R is revolved about the x-axis in terms of k.
(c) Let S be the unbounded region in the first quadrant to the right of the vertical line x ! k and below
the graph of f, as shown in the figure above. Find all values of k such that the volume of the solid
generated when S is revolved about the x-axis is equal to the volume of the solid found in part (b).

k
1
(a) Area of R ! dx ! ln k  2 n ln 2 1 : integral
0 x2
2: 1 : antidifferentiation and
evaluation

k
1 1 : limits
(b) VR ! 2
dx
0 x2 1 : integrand
k 3:
!n ! n 1 : antidifferentiation and
x2 0 2 k2 evaluation

1 1 : improper integral
(c) VS ! 2
dx
k x2 1 : antidifferentiation and
n
! lim n ! 4: evaluation
n x2 k k2 1 : equation
VS ! VR 1 : answer
! n
k2 2 k2
2 1
!
k2 2

k !2

Copyright © 2005 by College Board. All rights reserved.


Visit apcentral.collegeboard.com (for AP professionals) and www.collegeboard.com/apstudents (for AP students and parents).

7
AP® CALCULUS BC
2006 SCORING GUIDELINES

Question 1

Let R be the shaded region bounded by the graph of y ! ln x and the line
y ! x n 2, as shown above.
(a) Find the area of R.
(b) Find the volume of the solid generated when R is rotated about the horizontal
line y ! n3.
(c) Write, but do not evaluate, an integral expression that can be used to find the
volume of the solid generated when R is rotated about the y-axis.

ln x ! x n 2 when x ! 0.15859 and 3.14619.


Let S ! 0.15859 and T ! 3.14619

T
(a) Area of R ! ln x n x n 2 dx ! 1.949 1 : integrand
S
3: 1 : limits
1 : answer

T 2 2
(b) Volume ! ln x  3 n xn23 dx 2 : integrand
S 3:
1 : limits, constant, and answer
! 34.198 or 34.199

T n2 2
(c) Volume ! ( y  2) 2 n e y dy 2 : integrand
S n2 3:
1 : limits and constant

8LI'SPPIKI&SEVH%PPVMKLXWVIWIVZIH
:MWMXETGIRXVEPGSPPIKIFSEVHGSQ JSV%4TVSJIWWMSREPW ERH[[[GSPPIKIFSEVHGSQETWXYHIRXW JSV%4WXYHIRXWERHTEVIRXW 
2
AP® CALCULUS BC
2006 SCORING GUIDELINES

Question 2

At an intersection in Thomasville, Oregon, cars turn


t
left at the rate L t ! 60 t sin 2 cars per hour
3
over the time interval 0 t 18 hours. The graph of
y ! L t is shown above.
(a) To the nearest whole number, find the total
number of cars turning left at the intersection
over the time interval 0 t 18 hours.
(b) Traffic engineers will consider turn restrictions
when L t 150 cars per hour. Find all values
of t for which L t 150 and compute the
average value of L over this time interval.
Indicate units of measure.
(c) Traffic engineers will install a signal if there is any two-hour time interval during which the product of the
total number of cars turning left and the total number of oncoming cars traveling straight through the
intersection is greater than 200,000. In every two-hour time interval, 500 oncoming cars travel straight
through the intersection. Does this intersection require a traffic signal? Explain the reasoning that leads to
your conclusion.

18
(a) L t dt 1658 cars 1 : setup
0 2:
1 : answer

(b) L t ! 150 when t ! 12.42831, 16.12166 1 : t -interval when L t 150


Let R ! 12.42831 and S ! 16.12166 3: 1 : average value integral
L t 150 for t in the interval R, S 1 : answer with units
1 S
L t dt ! 199.426 cars per hour
SnR R

(c) For the product to exceed 200,000, the number of cars 1 : considers 400 cars
turning left in a two-hour interval must be greater than 400. 1 : valid interval h, h  2
4: h 2
15 1 : value of L t dt
L t dt ! 431.931 " 400 h
13
1 : answer and explanation

OR OR

The number of cars turning left will be greater than 400 1 : considers 200 cars per hour
on a two-hour interval if L t 200 on that interval. 1 : solves L t 200
4:
L t 200 on any two-hour subinterval of 1 : discusses 2 hour interval
13.25304, 15.32386 . 1 : answer and explanation

Yes, a traffic signal is required.

8LI'SPPIKI&SEVH%PPVMKLXWVIWIVZIH
:MWMXETGIRXVEPGSPPIKIFSEVHGSQ JSV%4TVSJIWWMSREPW ERH[[[GSPPIKIFSEVHGSQETWXYHIRXW JSV%4WXYHIRXWERHTEVIRXW 
3
AP® CALCULUS BC
2006 SCORING GUIDELINES

Question 3

An object moving along a curve in the xy-plane is at position x t , y t at time t, where

dx dy 4t
! sin n1 1 n 2e n t and !
dt dt 1  t 3

for t 0. At time t ! 2, the object is at the point 6, n3 . (Note: sin n1 x ! arcsin x )


(a) Find the acceleration vector and the speed of the object at time t ! 2.
(b) The curve has a vertical tangent line at one point. At what time t is the object at this point?
(c) Let m t denote the slope of the line tangent to the curve at the point x t , y t . Write an expression for
m t in terms of t and use it to evaluate lim m t .
t

(d) The graph of the curve has a horizontal asymptote y ! c. Write, but do not evaluate, an expression involving
an improper integral that represents this value c.

(a) a 2 ! 0.395 or 0.396, n0.741 or n0.740 1 : acceleration


2:
Speed ! x 2
2
 y 2
2
! 1.207 or 1.208 1 : speed

(b) sin n1 1 n 2ent ! 0 1:x t ! 0


2:
1 n 2en t ! 0 1 : answer
dy
t ! ln 2 ! 0.693 and 0 when t ! ln 2
dt

4t 1 1:m t
(c) m t
1  t sin 1 n 2e nt
3 n1 2:
1 : limit value
4t 1
lim m t lim
t t 1  t sin 1 n 2e nt
3 n1

1
!0 n1
!0
sin 1

(d) Since lim x t ! , 1: integrand


t
1: limits
4t 3:
c ! lim y t ! n3  dt 1: initial value consistent
t 2 1  t3
with lower limit

8LI'SPPIKI&SEVH%PPVMKLXWVIWIVZIH
:MWMXETGIRXVEPGSPPIKIFSEVHGSQ JSV%4TVSJIWWMSREPW ERH[[[GSPPIKIFSEVHGSQETWXYHIRXW JSV%4WXYHIRXWERHTEVIRXW 
4
AP® CALCULUS BC
2006 SCORING GUIDELINES

Question 4

t
0 10 20 30 40 50 60 70 80
(seconds)
vt
5 14 22 29 35 40 44 47 49
(feet per second)

Rocket A has positive velocity v t after being launched upward from an initial height of 0 feet at time t ! 0
seconds. The velocity of the rocket is recorded for selected values of t over the interval 0 t 80 seconds, as
shown in the table above.
(a) Find the average acceleration of rocket A over the time interval 0 t 80 seconds. Indicate units of
measure.
70
(b) Using correct units, explain the meaning of v t dt in terms of the rocket’s flight. Use a midpoint
10
70
Riemann sum with 3 subintervals of equal length to approximate v t dt.
10

3
(c) Rocket B is launched upward with an acceleration of a t ! feet per second per second. At time
t 1
t ! 0 seconds, the initial height of the rocket is 0 feet, and the initial velocity is 2 feet per second. Which of
the two rockets is traveling faster at time t ! 80 seconds? Explain your answer.

(a) Average acceleration of rocket A is 1 : answer

v 80 n v 0 49 n 5 11
! ! ft sec2
80 n 0 80 20
70
(b) Since the velocity is positive, v t dt represents the 1 : explanation
10
3: 1 : uses v 20 , v 40 , v 60
distance, in feet, traveled by rocket A from t ! 10 seconds
to t ! 70 seconds. 1 : value

A midpoint Riemann sum is


20 v 20  v 40  v 60
! 20 22  35  44 ! 2020 ft

(c) Let vB t be the velocity of rocket B at time t. 1: 6 t 1


3 1 : constant of integration
vB t ! dt ! 6 t  1  C
t 1 4: 1 : uses initial condition
2 ! vB 0 ! 6  C 1 : finds vB 80 , compares to v 80 ,
vB t ! 6 t  1 n 4 and draws a conclusion
vB 80 ! 50 " 49 ! v 80

Rocket B is traveling faster at time t ! 80 seconds.

Units of ft sec2 in (a) and ft in (b) 1 : units in (a) and (b)

8LI'SPPIKI&SEVH%PPVMKLXWVIWIVZIH
:MWMXETGIRXVEPGSPPIKIFSEVHGSQ JSV%4TVSJIWWMSREPW ERH[[[GSPPIKIFSEVHGSQETWXYHIRXW JSV%4WXYHIRXWERHTEVIRXW 
5
AP® CALCULUS BC
2006 SCORING GUIDELINES

Question 5

dy 6
Consider the differential equation ! 5 x2 n for y 2. Let y ! f x be the particular solution to this
dx yn2
differential equation with the initial condition f n1 ! n 4.

dy d2y
(a) Evaluate and at n1, n 4 .
dx dx 2
(b) Is it possible for the x-axis to be tangent to the graph of f at some point? Explain why or why not.
(c) Find the second-degree Taylor polynomial for f about x ! n1.
(d) Use Euler’s method, starting at x ! n1 with two steps of equal size, to approximate f 0 . Show the work
that leads to your answer.

dy dy
(a) !6 1:
dx n1, n 4 dx n1, n 4
2
d2y n2 dy d y
! 10 x  6 y n 2 3: 1:
dx 2 dx dx 2
d2y
1:
d2y 1 dx 2
! n10  6 6 ! n9 n1, n 4
dx 2 n1, n 4 n6 2

dy dy
(b) The x-axis will be tangent to the graph of f if ! 0. 1: ! 0 and y ! 0
dx k, 0
2: dx
1 : answer and explanation
The x-axis will never be tangent to the graph of f because
dy
! 5k 2  3 " 0 for all k.
dx k , 0

9 2
(c) P x ! n4  6 x  1 n x 1 1 : quadratic and centered at x ! n1
2 2:
1 : coefficients

(d) f n1 ! n 4 1 : Euler's method with 2 steps


2:
1 1 1 : Euler's approximation to f 0
f 4 6 1
2 2
1 5 5
f 0 1 2
2 4 8

8LI'SPPIKI&SEVH%PPVMKLXWVIWIVZIH
:MWMXETGIRXVEPGSPPIKIFSEVHGSQ JSV%4TVSJIWWMSREPW ERH[[[GSPPIKIFSEVHGSQETWXYHIRXW JSV%4WXYHIRXWERHTEVIRXW 
6
AP® CALCULUS BC
2006 SCORING GUIDELINES

Question 6

The function f is defined by the power series


x 2 x 2 3x3 n1 n nx n
f x !n  n   
2 3 4 n 1
for all real numbers x for which the series converges. The function g is defined by the power series
x x2 x3 n1 n x n
g x !1n  n   
2! 4! 6! 2n !
for all real numbers x for which the series converges.
(a) Find the interval of convergence of the power series for f. Justify your answer.
(b) The graph of y ! f x n g x passes through the point 0, n1 . Find y 0 and y 0 . Determine whether y
has a relative minimum, a relative maximum, or neither at x ! 0. Give a reason for your answer.

n 1
n1 n  1 x n 1 n 1 n 1 2
(a) x 1 : sets up ratio
n2 n1 n nx n n2 n
1 : computes limit of ratio
n 1 2 1 : identifies radius of convergence
lim x x 5:
n n2 n 1 : considers both endpoints
The series converges when n1 x 1. 1 : analysis/conclusion for
1 2 3 both endpoints
When x ! 1, the series is n  n 
2 3 4
This series does not converge, because the limit of the
individual terms is not zero.

1 2 3
When x ! n1, the series is   
2 3 4
This series does not converge, because the limit of the
individual terms is not zero.

Thus, the interval of convergence is n1 x 1.

1 4 9 1
(b) f x !n  x n x2  and f 0 ! n . 1: y 0
2 3 4 2
1: y 0
1 2 3 2 1 4:
g x !n  xn x  and g 0 ! n . 1 : conclusion
2! 4! 6! 2
1 : reasoning
y 0 ! f 0 ng 0 !0
4 2 1
f 0 ! and g 0 ! ! .
3 4! 12
4 1
Thus, y 0 ! n " 0.
3 12
Since y 0 ! 0 and y 0 " 0, y has a relative minimum
at x ! 0.

8LI'SPPIKI&SEVH%PPVMKLXWVIWIVZIH
:MWMXETGIRXVEPGSPPIKIFSEVHGSQ JSV%4TVSJIWWMSREPW ERH[[[GSPPIKIFSEVHGSQETWXYHIRXW JSV%4WXYHIRXWERHTEVIRXW 
7
AP® CALCULUS BC
2006 SCORING GUIDELINES (Form B)

Question 1

x3 x 2 x
Let f be the function given by f x ! n n  3cos x. Let R
4 3 2
be the shaded region in the second quadrant bounded by the graph of f,
and let S be the shaded region bounded by the graph of f and line ,
the line tangent to the graph of f at x ! 0, as shown above.
(a) Find the area of R.
(b) Find the volume of the solid generated when R is rotated about the
horizontal line y ! n2.
(c) Write, but do not evaluate, an integral expression that can be used
to find the area of S.

For x 0, f x ! 0 when x ! n1.37312.


Let P ! n1.37312.

0
(a) Area of R ! f x dx ! 2.903 1 : integral
P 2:
1 : answer

0 2
(b) Volume ! f x 2 n 4 dx ! 59.361 1 : limits and constant
P
4: 2 : integrand
1 : answer

1 1 : tangent line
(c) The equation of the tangent line is y ! 3 n x.
2
3: 1 : integrand
The graph of f and line intersect at A ! 3.38987. 1 : limits

A
1
Area of S ! 3n x n f x dx
0 2

8LI'SPPIKI&SEVH%PPVMKLXWVIWIVZIH
:MWMXETGIRXVEPGSPPIKIFSEVHGSQ JSV%4TVSJIWWMSREPW ERH[[[GSPPIKIFSEVHGSQETWXYHIRXW JSV%4WXYHIRXWERHTEVIRXW 
2
AP® CALCULUS BC
2006 SCORING GUIDELINES (Form B)

Question 2

An object moving along a curve in the xy-plane is at position x t , y t at time t, where

dx dy
! tan e nt and ! sec e nt
dt dt
for t 0. At time t ! 1, the object is at position 2, n 3 .
(a) Write an equation for the line tangent to the curve at position 2, n 3 .
(b) Find the acceleration vector and the speed of the object at time t ! 1.
(c) Find the total distance traveled by the object over the time interval 1 t 2.
(d) Is there a time t 0 at which the object is on the y-axis? Explain why or why not.

dy
dy dt sec e nt 1 dy
(a) ! ! ! 1:
dx dx tan e nt
sin e nt 2: dx 2, n3
dt 1 : equation of tangent line
dy 1
! ! 2.780 or 2.781
dx 2, n3 sin e n1
1
y3! xn2
sin en1

(b) x 1 ! n0.42253, y 1 ! n0.15196 1 : acceleration vector


2:
1 : speed
a 1 ! n0.423, n0.152 or n0.422, n0.151 .

2 2
speed ! sec en1  tan en1 ! 1.138 or 1.139

2 2 2
(c) x t  y t dt ! 1.059 1 : integral
1 2:
1 : answer

1
(d) x 0 ! x1 n x t dt ! 2 n 0.775553 " 0 1 : x 0 expression
0
3: 1: x t " 0
The particle starts to the right of the y-axis. 1 : conclusion and reason
Since x t " 0 for all t 0, the object is always moving
to the right and thus is never on the y-axis.

8LI'SPPIKI&SEVH%PPVMKLXWVIWIVZIH
:MWMXETGIRXVEPGSPPIKIFSEVHGSQ JSV%4TVSJIWWMSREPW ERH[[[GSPPIKIFSEVHGSQETWXYHIRXW JSV%4WXYHIRXWERHTEVIRXW 
3
AP® CALCULUS BC
2006 SCORING GUIDELINES (Form B)

Question 3

The figure above is the graph of a function of x, which models the height of a
skateboard ramp. The function meets the following requirements.
(i) At x ! 0, the value of the function is 0, and the slope of the graph of
the function is 0.
(ii) At x ! 4, the value of the function is 1, and the slope of the graph of
the function is 1.
(iii) Between x ! 0 and x ! 4, the function is increasing.

(a) Let f x ! ax 2 , where a is a nonzero constant. Show that it is not possible to find a value for a so that f
meets requirement (ii) above.
x2
(b) Let g x ! cx3 n , where c is a nonzero constant. Find the value of c so that g meets requirement (ii)
16
above. Show the work that leads to your answer.
(c) Using the function g and your value of c from part (b), show that g does not meet requirement (iii) above.
xn
(d) Let h x ! , where k is a nonzero constant and n is a positive integer. Find the values of k and n so that
k
h meets requirement (ii) above. Show that h also meets requirements (i) and (iii) above.

1 1 1
(a) f 4 ! 1 implies that a ! and f 4 ! 2a 4 ! 1 1:a ! or a !
16 2: 16 8
1 1 : shows a does not work
implies that a ! . Thus, f cannot satisfy (ii).
8

1
(b) g 4 ! 64c n 1 ! 1 implies that c ! . 1 : value of c
32
1 2 2 4 1 1
When c ! , g 4 ! 3c 4 n !3 16 n ! 1
32 16 32 2

3 2 x 1
(c) g x ! x n ! x 3x n 4 1:g x
32 8 32 2:
1 : explanation
4
g x 0 for 0 x , so g does not satisfy (iii).
3

4n 4n
(d) h 4 ! ! 1 implies that 4n ! k . 1: !1
k k
n 4n n1 n4n n1 n n 4n -1
h 4 ! ! ! ! 1 gives n ! 4 and k ! 44 ! 256. 4: 1: !1
k 4 n 4 k
1 : values for k and n
x4 1 : verifications
h x ! h 0 ! 0.
256
4 x3
h x ! h 0 ! 0 and h x " 0 for 0 x 4.
256

8LI'SPPIKI&SEVH%PPVMKLXWVIWIVZIH
:MWMXETGIRXVEPGSPPIKIFSEVHGSQ JSV%4TVSJIWWMSREPW ERH[[[GSPPIKIFSEVHGSQETWXYHIRXW JSV%4WXYHIRXWERHTEVIRXW 
4
AP® CALCULUS BC
2006 SCORING GUIDELINES (Form B)

Question 4

The rate, in calories per minute, at which a person using an


exercise machine burns calories is modeled by the function
1 3
f. In the figure above, f t ! n t 3  t 2  1 for
4 2
0 t 4 and f is piecewise linear for 4 t 24.
(a) Find f 22 . Indicate units of measure.
(b) For the time interval 0 t 24, at what time t is f
increasing at its greatest rate? Show the reasoning that
supports your answer.
(c) Find the total number of calories burned over the time
interval 6 t 18 minutes.
(d) The setting on the machine is now changed so that the person burns f t  c calories per minute. For this
setting, find c so that an average of 15 calories per minute is burned during the time interval 6 t 18.

15 n 3
(a) f 22 ! ! n3 calories/min/min 1 : f 22 and units
20 n 24

(b) f is increasing on 0, 4 and on 12, 16 . 1 : f on 0, 4


15 n 9 3 1 : shows f has a max at t ! 2 on 0, 4
On 12, 16 , f t ! ! since f has 4:
16 n 12 2 1 : shows for 12 t 16, f t f 2
constant slope on this interval. 1 : answer
3
On 0, 4 , f t ! n t 2  3t and
4
3
f t ! n t  3 ! 0 when t ! 2. This is where f
2
has a maximum on 0, 4 since f " 0 on 0, 2
and f 0 on 2, 4 .
On 0, 24 , f is increasing at its greatest rate when
3
t ! 2 because f 2 ! 3 " .
2
18 1 1 : method
(c) f t dt ! 6 9  4 9  15  2 15
6 2 2:
1 : answer
! 132 calories

1 18 1 : setup
(d) We want f t  c dt ! 15.
12 6 2:
1 : value of c
This means 132  12c ! 15(12). So, c ! 4.
OR
132
Currently, the average is ! 11 calories/min.
12
Adding c to f t will shift the average by c.
So c ! 4 to get an average of 15 calories/min.

8LI'SPPIKI&SEVH%PPVMKLXWVIWIVZIH
:MWMXETGIRXVEPGSPPIKIFSEVHGSQ JSV%4TVSJIWWMSREPW ERH[[[GSPPIKIFSEVHGSQETWXYHIRXW JSV%4WXYHIRXWERHTEVIRXW 
5
AP® CALCULUS BC
2006 SCORING GUIDELINES (Form B)

Question 5

Let f be a function with f 4 ! 1 such that all points x, y on the graph of f satisfy the differential equation

dy
! 2y 3 n x .
dx
Let g be a function with g 4 ! 1 such that all points x, y on the graph of g satisfy the logistic differential
equation
dy
! 2y 3 n y .
dx
(a) Find y ! f x .
(b) Given that g 4 ! 1, find lim g x and lim g x . (It is not necessary to solve for g x or to show how
x x
you arrived at your answers.)
(c) For what value of y does the graph of g have a point of inflection? Find the slope of the graph of g at the
point of inflection. (It is not necessary to solve for g x . )

dy
(a) ! 2y 3 n x 1 : separates variables
dx
1 : antiderivatives
1
dy ! 2 3 n x dx 5: 1 : constant of integration
y
1 : uses initial condition
ln y ! 6 x n x 2  C
1 : solution
0 ! 24 n 16  C
C ! n8
Note: max 2 5 [1-1-0-0-0] if no
ln y ! 6 x n x 2 n 8
2 n8
constant of integration
y ! e6 x n x for n x Note: 0 5 if no separation of variables

(b) lim g x ! 3 1 : lim g x ! 3


x x
2:
1 : lim g x ! 0
x
lim g x ! 0
x

d2y dy 3
(c) ! (6 n 4 y ) 1: y !
dx 2
dx 2 2: dy
dy 1:
Because 0 at any point on the graph of g, the dx y !3 2
dx
3
concavity only changes sign at y ! , half the carrying
2
capacity.
dy 3 3 9
!2 3n !
dx y ! 3 2 2 2 2

8LI'SPPIKI&SEVH%PPVMKLXWVIWIVZIH
:MWMXETGIRXVEPGSPPIKIFSEVHGSQ JSV%4TVSJIWWMSREPW ERH[[[GSPPIKIFSEVHGSQETWXYHIRXW JSV%4WXYHIRXWERHTEVIRXW 
6
AP® CALCULUS BC
2006 SCORING GUIDELINES (Form B)

Question 6
1
The function f is defined by f x ! . The Maclaurin series for f is given by
1  x3

1 n x3  x 6 n x 9   n1 n x3n  ,
which converges to f x for n1 x 1.

(a) Find the first three nonzero terms and the general term for the Maclaurin series for f x .
3 6 9 3n
(b) Use your results from part (a) to find the sum of the infinite series n 2
 5 n 8   n1 n 3 n n1
 .
2 2 2 2
x
(c) Find the first four nonzero terms and the general term for the Maclaurin series representing f t dt .
0
1 2
(d) Use the first three nonzero terms of the infinite series found in part (c) to approximate f t dt. What are
0
1 2
the properties of the terms of the series representing f t dt that guarantee that this approximation is
0
1
within of the exact value of the integral?
10,000

(a) f x ! n3 x 2  6 x 5 n 9 x 8   3n n1 n x3n n1  1 : first three terms


2:
1 : general term

1
(b) The given series is the Maclaurin series for f x with x ! . 1: f x
2
n2
2: 1
f x ! n 1  x3 3x 2 1: f
2
1
3
1 4 16
Thus, the sum of the series is f !n !n .
2 1 2 27
1
8
x
1 x4 x7 x10 n1 n x3n 1 1 : first four terms
(c) dt ! x n  n    2:
0 1  t3 4 7 10 3n  1 1 : general term

1 4 1 7
1 2
1 1 2 2
(d) dt . 1 : approximation
0 1  t3 2 4 7
1 : properties of terms
1 3:
The series in part (c) with x ! has terms that alternate, decrease in 1 : absolute value of
2
absolute value, and have limit 0. Hence the error is bounded by the fourth term 0.0001
absolute value of the next term.
1 4 1 7 1 10
1 2
1 1 2 2 2 1
dt n n  ! 0.0001
0 1 t 3 2 4 7 10 10240

8LI'SPPIKI&SEVH%PPVMKLXWVIWIVZIH
:MWMXETGIRXVEPGSPPIKIFSEVHGSQ JSV%4TVSJIWWMSREPW ERH[[[GSPPIKIFSEVHGSQETWXYHIRXW JSV%4WXYHIRXWERHTEVIRXW 
7
%4’'%0'9097&'
7'36-2++9-()0-2)7
5YIWXMSR

20
Let R be the region in the first and second quadrants bounded above by the graph of y ! and
1  x2
below by the horizontal line y ! 2.
(a) Find the area of R.
(b) Find the volume of the solid generated when R is rotated about the x-axis.
(c) The region R is the base of a solid. For this solid, the cross sections perpendicular to the
x-axis are semicircles. Find the volume of this solid.

20
! 2 when x ! •3 1 : correct limits in an integral in
1  x2 (a), (b), or (c)

3
20 1 : integrand
(a) Area ! n 2 dx ! 37.961 or 37.962 2:
n3 1  x2 1 : answer

3 2
20 2 : integrand
(b) Volume ! n 22 dx ! 1871.190 3:
n3
1  x2 1 : answer

3 2
1 20 2 : integrand
(c) Volume ! n2 dx 3:
2 n3
2 1  x2 1 : answer
3 2
20
! n 2 dx ! 174.268
8 n3 1  x2

8LI'SPPIKI&SEVH%PPVMKLXWVIWIVZIH
:MWMXETGIRXVEPGSPPIKIFSEVHGSQ JSV%4TVSJIWWMSREPW ERH[[[GSPPIKIFSEVHGSQETWXYHIRXW JSVWXYHIRXWERHTEVIRXW 
%4’'%0'9097&'
7'36-2++9-()0-2)7
5YIWXMSR

The amount of water in a storage tank, in gallons, is modeled


by a continuous function on the time interval 0 t 7,
where t is measured in hours. In this model, rates are given
as follows:
(i) The rate at which water enters the tank is
f t ! 100t 2 sin t gallons per hour for 0 t 7.
(ii) The rate at which water leaves the tank is
250 for 0 t 3
g t ! gallons per hour.
2000 for 3 t 7
The graphs of f and g, which intersect at t ! 1.617 and t ! 5.076, are shown in the figure above. At
time t ! 0, the amount of water in the tank is 5000 gallons.
(a) How many gallons of water enter the tank during the time interval 0 t 7 ? Round your answer to
the nearest gallon.
(b) For 0 t 7, find the time intervals during which the amount of water in the tank is decreasing.
Give a reason for each answer.
(c) For 0 t 7, at what time t is the amount of water in the tank greatest? To the nearest gallon,
compute the amount of water at this time. Justify your answer.

7 1 : integral
(a) f t dt 8264 gallons 2:
0 1 : answer

(b) The amount of water in the tank is decreasing on the 1 : intervals


intervals 0 t 1.617 and 3 t 5.076 because 2:
1 : reason
f t g t for 0 t 1.617 and 3 t 5.076.

(c) Since f t n g t changes sign from positive to negative 1 : identifies t ! 3 as a candidate


only at t ! 3, the candidates for the absolute maximum are 1 : integrand
at t ! 0, 3, and 7. 5: 1 : amount of water at t ! 3
1 : amount of water at t ! 7
t (hours) gallons of water 1 : conclusion
0 5000
3
3 5000  f t dt n 250 3 ! 5126.591
0

7
7 5126.591  f t dt n 2000 4 ! 4513.807
3

The amount of water in the tank is greatest at 3 hours. At


that time, the amount of water in the tank, rounded to the
nearest gallon, is 5127 gallons.

8LI'SPPIKI&SEVH%PPVMKLXWVIWIVZIH
:MWMXETGIRXVEPGSPPIKIFSEVHGSQ JSV%4TVSJIWWMSREPW ERH[[[GSPPIKIFSEVHGSQETWXYHIRXW JSVWXYHIRXWERHTEVIRXW 
%4’'%0'9097&'
7'36-2++9-()0-2)7
5YIWXMSR

The graphs of the polar curves r ! 2 and r ! 3  2cos are shown in


2 4
the figure above. The curves intersect when ! and ! .
3 3
(a) Let R be the region that is inside the graph of r ! 2 and also inside
the graph of r ! 3  2 cos , as shaded in the figure above. Find the
area of R.
(b) A particle moving with nonzero velocity along the polar curve given
by r ! 3  2cos has position x t , y t at time t, with ! 0
dr dr
when t ! 0. This particle moves along the curve so that ! .
dt d
dr
Find the value of at ! and interpret your answer in terms of the motion of the particle.
dt 3
dy dy dy
(c) For the particle described in part (b), ! . Find the value of at ! and interpret your
dt d dt 3
answer in terms of the motion of the particle.

2 1 4 3
(a) Area ! 2 2 3  2cos 2
d 1 : area of circular sector
3 2 2 3
2 : integral for section of limaçon
! 10.370
4: 1 : integrand
1 : limits and constant
1 : answer

dr dr dr
(b) ! ! n1.732 1:
dt ! 3 d ! 3 2: dt ! 3
1 : interpretation
dr
The particle is moving closer to the origin, since 0
dt
and r " 0 when ! .
3

(c) y ! r sin ! 3  2cos sin 1 : expression for y in terms of


dy dy dy
! ! 0.5 3: 1:
dt ! 3 d ! 3
dt ! 3
1: interpretation
The particle is moving away from the x-axis, since
dy
" 0 and y " 0 when ! .
dt 3

8LI'SPPIKI&SEVH%PPVMKLXWVIWIVZIH
:MWMXETGIRXVEPGSPPIKIFSEVHGSQ JSV%4TVSJIWWMSREPW ERH[[[GSPPIKIFSEVHGSQETWXYHIRXW JSVWXYHIRXWERHTEVIRXW 
%4’'%0'9097&'
7'36-2++9-()0-2)7
5YIWXMSR

Let f be the function defined for x " 0, with f e ! 2 and f , the first derivative of f, given by f x ! x 2 ln x.
(a) Write an equation for the line tangent to the graph of f at the point e, 2 .
(b) Is the graph of f concave up or concave down on the interval 1 x 3 ? Give a reason for your answer.
(c) Use antidifferentiation to find f x .

(a) f e ! e2 1: f e
2:
1 : equation of tangent line
An equation for the line tangent to the graph of f at the
point e, 2 is y n 2 ! e2 x n e .

(b) f x ! x  2 x ln x. 2: f x
3:
1 : answer with reason
For 1 x 3, x " 0 and ln x " 0, so f x " 0. Thus,
the graph of f is concave up on 1, 3 .

(c) Since f x ! x 2 ln x dx, we consider integration by 2 : antiderivative


parts. 4: 1 : uses f e ! 2
u ! ln x dv ! x 2 dx 1 : answer
1 1 3
du ! dx v! x 2 dx ! x
x 3

Therefore,
f x ! x 2 ln x dx
1 3 1 3 1
x ln x x dx
3 3 x
1 1
! x3 ln x n x 3  C .
3 9

e3 e3 2
Since f e ! 2, 2 ! n  C and C ! 2 n e3 .
3 9 9
x3 1 2
Thus, f x ! ln x n x3  2 n e3 .
3 9 9

8LI'SPPIKI&SEVH%PPVMKLXWVIWIVZIH
:MWMXETGIRXVEPGSPPIKIFSEVHGSQ JSV%4TVSJIWWMSREPW ERH[[[GSPPIKIFSEVHGSQETWXYHIRXW JSVWXYHIRXWERHTEVIRXW 
%4’'%0'9097&'
7'36-2++9-()0-2)7
5YIWXMSR

t
0 2 5 7 11 12
(minutes)
r t
5.7 4.0 2.0 1.2 0.6 0.5
(feet per minute)

The volume of a spherical hot air balloon expands as the air inside the balloon is heated. The radius of the
balloon, in feet, is modeled by a twice-differentiable function r of time t, where t is measured in minutes.
For 0 t 12, the graph of r is concave down. The table above gives selected values of the rate of change,
r t , of the radius of the balloon over the time interval 0 t 12. The radius of the balloon is 30 feet when
4 3
t ! 5. (Note: The volume of a sphere of radius r is given by V ! r .)
3
(a) Estimate the radius of the balloon when t ! 5.4 using the tangent line approximation at t ! 5. Is your
estimate greater than or less than the true value? Give a reason for your answer.
(b) Find the rate of change of the volume of the balloon with respect to time when t ! 5. Indicate units of
measure.
(c) Use a right Riemann sum with the five subintervals indicated by the data in the table to approximate
12 12
r t dt. Using correct units, explain the meaning of r t dt in terms of the radius of the
0 0
balloon.
12
(d) Is your approximation in part (c) greater than or less than r t dt ? Give a reason for your answer.
0

(a) r 5.4 r 5 r 5 t 30 2 0.4 30.8 ft 1 : estimate


2:
Since the graph of r is concave down on the interval 1 : conclusion with reason
5 t 5.4, this estimate is greater than r 5.4 .

dV 4 dr dV
(b) !3 r2 2:
dt 3 dt 3: dt
dV 1 : answer
! 4 30 2 2 ! 7200 ft 3 min
dt t ! 5

12
(c) r t dt 2 4.0 3 2.0 2 1.2 4 0.6 1 0.5 1 : approximation
0 2:
1 : explanation
! 19.3 ft
12
r t dt is the change in the radius, in feet, from
0
t ! 0 to t ! 12 minutes.

(d) Since r is concave down, r is decreasing on 0 t 12. 1 : conclusion with reason


Therefore, this approximation, 19.3 ft, is less than
12
r t dt.
0

Units of ft 3 min in part (b) and ft in part (c) 1 : units in (b) and (c)

8LI'SPPIKI&SEVH%PPVMKLXWVIWIVZIH
:MWMXETGIRXVEPGSPPIKIFSEVHGSQ JSV%4TVSJIWWMSREPW ERH[[[GSPPIKIFSEVHGSQETWXYHIRXW JSVWXYHIRXWERHTEVIRXW 
%4’'%0'9097&'
7'36-2++9-()0-2)7
5YIWXMSR

2
Let f be the function given by f x ! e n x .
(a) Write the first four nonzero terms and the general term of the Taylor series for f about x ! 0.
1 n x2 n f x
(b) Use your answer to part (a) to find lim .
x 0 x4
x nt 2
(c) Write the first four nonzero terms of the Taylor series for e dt about x ! 0. Use the first two
0
1 2 nt 2
terms of your answer to estimate e dt.
0

1 2 nt 2
(d) Explain why the estimate found in part (c) differs from the actual value of e dt by less than
0
1
.
200

2 3 n
n x2
n x2 n x2 n x2 n x2 x4 x6
(a) e !1      1 : two of 1, n x 2 ,,n
1! 2! 3! n! 2 6
3:
x x 4 6 n1 n x 2 n 1 : remaining terms
2
!1n x  n    1 : general term
2 6 n!

n 1 2 n n 4
1 n x2 n f x 1 x2 n1 x
(b) !n   1 : answer
x 4 2 6 n!4
n!
1 n x2 n f x 1
Thus, lim !n .
x 0 x 4 2

x
x nt 2 t4 t6 n1 n t 2 n
(c) e dt ! 1 n t2  n    dt 1 : two terms
0 2 6 n!
0 3: 1 : remaining terms
x 3 x5 x 7 1 : estimate
! xn
 n 
3 10 42
Using the first two terms of this series, we estimate that
1 2 nt 2 1 1 1 11
e dt .
0 2 3 8 24

5
1 2 nt 2 11 1 1 1 1
(d) e dt , since 1 : uses the third term as
0 24 2 10 320 200
2: the error bound
n 1 2 n 1
1 2 nt 2
n1 1 : explanation
2
e dt ! , which is an alternating
0
n!0
n ! 2n  1
series with individual terms that decrease in absolute value
to 0.

8LI'SPPIKI&SEVH%PPVMKLXWVIWIVZIH
:MWMXETGIRXVEPGSPPIKIFSEVHGSQ JSV%4TVSJIWWMSREPW ERH[[[GSPPIKIFSEVHGSQETWXYHIRXW JSVWXYHIRXWERHTEVIRXW 
%4’'%0'9097&'
7'36-2++9-()0-2)7 *SVQ& 
5YIWXMSR
2
Let R be the region bounded by the graph of y e 2x x and the
horizontal line y 2, and let S be the region bounded by the graph of
2
y e2x x and the horizontal lines y 1 and y 2, as shown above.
(a) Find the area of R.
(b) Find the area of S.
(c) Write, but do not evaluate, an integral expression that gives the
volume of the solid generated when R is rotated about the
horizontal line y 1.

2
e2 x x 2 when x 0.446057, 1.553943
Let P 0.446057 and Q 1.553943

Q
1 : integrand
x2
(a) Area of R e2 x 2 dx 0.514 3: 1 : limits
P
1 : answer

x2
(b) e2 x 1 when x 0, 2 1 : integrand
3: 1 : limits
2
Area of S e2 x x 2
1 dx Area of R 1 : answer
0
2.06016 Area of R 1.546
OR
P 2
x2 x2
e2 x 1 dx Q P 1 e2 x 1 dx
0 Q
0.219064 1.107886 0.219064 1.546

Q 2
x2 2 2 : integrand
(c) Volume e2 x 1 2 1 dx 3:
P 1 : constant and limits

8LI'SPPIKI&SEVH%PPVMKLXWVIWIVZIH
:MWMXETGIRXVEPGSPPIKIFSEVHGSQ JSV%4TVSJIWWMSREPW ERH[[[GSPPIKIFSEVHGSQETWXYHIRXW JSVWXYHIRXWERHTEVIRXW 
%4’'%0'9097&'
7'36-2++9-()0-2)7 *SVQ& 
5YIWXMSR

An object moving along a curve in the xy-plane is at position x t , y t at time t with

dx t dy
arctan and ln t 2 1
dt 1 t dt

for t 0. At time t 0, the object is at position 3, 4 . (Note: tan 1 x arctan x )


(a) Find the speed of the object at time t 4.
(b) Find the total distance traveled by the object over the time interval 0 t 4.
(c) Find x 4 .
(d) For t 0, there is a point on the curve where the line tangent to the curve has slope 2. At what time t is the
object at this point? Find the acceleration vector at this point.

2 2
(a) Speed x 4 y 4 2.912 1 : speed at t 4

4 2 2
dx dy 1 : integral
(b) Distance dt 6.423 2:
dt dt 1 : answer
0

4
(c) x 4 x 0 x t dt 1 : integrand
0 2:
3: 1 : uses x 0 3
3 2.10794 0.892
1 : answer

dy dy
dt t
(d) The slope is 2, so 2, or ln t 2
1 2 arctan . 1 : dt 2
dx 1 t dx
dt 3: dt
1 : t -value
Since t 0, t 1.35766. At this time, the acceleration is 1 : values for x and y
x t , y t t 1.35766 0.135, 0.955 .

8LI'SPPIKI&SEVH%PPVMKLXWVIWIVZIH
:MWMXETGIRXVEPGSPPIKIFSEVHGSQ JSV%4TVSJIWWMSREPW ERH[[[GSPPIKIFSEVHGSQETWXYHIRXW JSVWXYHIRXWERHTEVIRXW 
%4’'%0'9097&'
7'36-2++9-()0-2)7 *SVQ& 
5YIWXMSR

The wind chill is the temperature, in degrees Fahrenheit F , a human feels based on the air temperature, in
degrees Fahrenheit, and the wind velocity v, in miles per hour mph . If the air temperature is 32 F, then the
wind chill is given by W v 55.6 22.1v 0.16 and is valid for 5 v 60.
(a) Find W 20 . Using correct units, explain the meaning of W 20 in terms of the wind chill.
(b) Find the average rate of change of W over the interval 5 v 60. Find the value of v at which the
instantaneous rate of change of W is equal to the average rate of change of W over the interval 5 v 60.
(c) Over the time interval 0 t 4 hours, the air temperature is a constant 32 F. At time t 0, the wind
velocity is v 20 mph. If the wind velocity increases at a constant rate of 5 mph per hour, what is the rate of
change of the wind chill with respect to time at t 3 hours? Indicate units of measure.

0.84
(a) W 20 22.1 0.16 20 0.285 or 0.286 1 : value
2:
1 : explanation
When v 20 mph, the wind chill is decreasing at
0.286 F mph.

(b) The average rate of change of W over the interval 1 : average rate of change
W 60 W 5 3: 1:W v average rate of change
5 v 60 is 0.253 or 0.254.
60 5 1 : value of v
W 60 W 5
W v when v 23.011.
60 5

dW dW dv dv
(c) W 35 5 0.892 F hr 1: 5
dt t 3 dv dt t 3 dt
1 : uses v 3 35,
3:
OR or
uses v t 20 5t
0.16
W 55.6 22.1 20 5t 1 : answer
dW
0.892 F hr
dt t 3

Units of F mph in (a) and F hr in (c) 1 : units in (a) and (c)

8LI'SPPIKI&SEVH%PPVMKLXWVIWIVZIH
:MWMXETGIRXVEPGSPPIKIFSEVHGSQ JSV%4TVSJIWWMSREPW ERH[[[GSPPIKIFSEVHGSQETWXYHIRXW JSVWXYHIRXWERHTEVIRXW 
%4’'%0'9097&'
7'36-2++9-()0-2)7 *SVQ& 
5YIWXMSR

Let f be a function defined on the closed interval 5 x 5


with f 1 3. The graph of f , the derivative of f, consists
of two semicircles and two line segments, as shown above.
(a) For 5 x 5, find all values x at which f has a
relative maximum. Justify your answer.
(b) For 5 x 5, find all values x at which the graph of f
has a point of inflection. Justify your answer.
(c) Find all intervals on which the graph of f is concave up
and also has positive slope. Explain your reasoning.
(d) Find the absolute minimum value of f x over the closed interval 5 x 5. Explain your reasoning.

(a) f x 0 at x 3, 1, 4 1 : x-values
2:
f changes from positive to negative at 3 and 4. 1 : justification
Thus, f has a relative maximum at x 3 and at x 4.

(b) f changes from increasing to decreasing, or vice versa, at 1 : x-values


2:
x 4, 1, and 2. Thus, the graph of f has points of 1 : justification
inflection when x 4, 1, and 2.

(c) The graph of f is concave up with positive slope where f 1 : intervals


2:
is increasing and positive: 5 x 4 and 1 x 2. 1 : explanation

(d) Candidates for the absolute minimum are where f 1 : identifies x 1 as a candidate
changes from negative to positive (at x 1 ) and at the 3: 1 : considers endpoints
endpoints ( x 5, 5 ). 1 : value and explanation
5
f 5 3 f x dx 3 2 3
1 2
f 1 3
5 3 2 1
f 5 3 f x dx 33
1 2 2
The absolute minimum value of f on 5, 5 is f 1 3.

8LI'SPPIKI&SEVH%PPVMKLXWVIWIVZIH
:MWMXETGIRXVEPGSPPIKIFSEVHGSQ JSV%4TVSJIWWMSREPW ERH[[[GSPPIKIFSEVHGSQETWXYHIRXW JSVWXYHIRXWERHTEVIRXW 
%4’'%0'9097&'
7'36-2++9-()0-2)7 *SVQ& 
5YIWXMSR

dy
Consider the differential equation 3x 2y 1.
dx

d2y
(a) Find in terms of x and y.
dx 2
(b) Find the values of the constants m, b, and r for which y mx b e rx is a solution to the differential
equation.
(c) Let y f x be a particular solution to the differential equation with the initial condition f 0 2. Use
1
Euler’s method, starting at x 0 with a step size of , to approximate f 1 . Show the work that leads to
2
your answer.
(d) Let y g x be another solution to the differential equation with the initial condition g 0 k , where k is
a constant. Euler’s method, starting at x 0 with a step size of 1, gives the approximation g 1 0. Find the
value of k.

dy
d2y dy 2:
1:3 2
(a) 3 2 3 2 3x 2y 1 6x 4y 5 dx
dx 2 dx 1 : answer

dy
(b) If y mx b e rx is a solution, then 1: m re rx
dx
m re rx 3x 2 mx b e rx 1. 3:
1 : value for r
1 : values for m and b
If r 0: m 2b 1, r 2, 0 3 2m,
3 5
so m , r 2, and b .
2 4
OR
If r 0: m 2b 3, r 0, 0 3 2m,
3 9
so m , r 0, b .
2 4

1 1 1 7
(c) f f 0 f 0 2 3 1 : Euler's method with 2 steps
2 2 2 2 2:
1 : Euler's approximation for f 1
1 1 7 9
f 3 2 1
2 2 2 2
1 1 1 7 9 1 23
f 1 f f
2 2 2 2 2 2 4

(d) g 0 3 0 2 k 1 2k 1 1: g 0 g 0 1
2:
g1 g 0 g 0 1 k 2k 1 3k 1 0 1 : value of k
1
k
3

8LI'SPPIKI&SEVH%PPVMKLXWVIWIVZIH
:MWMXETGIRXVEPGSPPIKIFSEVHGSQ JSV%4TVSJIWWMSREPW ERH[[[GSPPIKIFSEVHGSQETWXYHIRXW JSVWXYHIRXWERHTEVIRXW 
%4’'%0'9097&'
7'36-2++9-()0-2)7 *SVQ& 
5YIWXMSR
x 3
Let f be the function given by f x 6e for all x.

(a) Find the first four nonzero terms and the general term for the Taylor series for f about x 0.
x
(b) Let g be the function given by g x f t dt. Find the first four nonzero terms and the general term for the Taylor
0
series for g about x 0.
(c) The function h satisfies h x k f ax for all x, where a and k are constants. The Taylor series for h about x 0
is given by
x2 x3 xn
h x 1 x .
2! 3! n!
Find the values of a and k.

x x2 x3 1 n xn x2 x3
(a) f x 6 1 1 : two of 6, 2 x, ,
3 2!32 3!33 n !3n 3 27
3:
n n
1 : remaining terms
2 3 6 1 x
x x 1 : general term
6 2x
3 27 n!3n 1 missing factor of 6

(b) g 0 0 and g x f x , so 1 : two terms


x2 x3 x4 1 xn n 1 3: 1 : remaining terms
g x 6 x 1 : general term
6 3!32 4!33 n 1 !3n
1 missing factor of 6
x3 x4 6 1 n xn 1
6x x2
9 4 27 n 1 !3n

(c) f x 2e x 3
, so h x 2k e ax 3 1 : computes k f ax
x2 x3 xn x 1 : recognizes h x ex ,
h x 1 x e 3:
2! 3! n! or
ax 3 x equates 2 series for h x
2k e e
a 1 : values for a and k
1 and 2k 1
3
1
a 3 and k
2
OR
2
f x 2 x , so
3
2
h x kf ax 2k ak x
3
h x 1 x
2
2k 1 and ak 1
3
1
k and a 3
2

8LI'SPPIKI&SEVH%PPVMKLXWVIWIVZIH
:MWMXETGIRXVEPGSPPIKIFSEVHGSQ JSV%4TVSJIWWMSREPW ERH[[[GSPPIKIFSEVHGSQETWXYHIRXW JSVWXYHIRXWERHTEVIRXW 
%4’'%0'9097&'
7'36-2++9-()0-2)7
5YIWXMSR

Let R be the region bounded by the graphs of y ! sin x and y ! x3 n 4 x, as shown in the figure
above.
(a) Find the area of R.
(b) The horizontal line y ! n2 splits the region R into two parts. Write, but do not evaluate, an integral
expression for the area of the part of R that is below this horizontal line.
(c) The region R is the base of a solid. For this solid, each cross section perpendicular to the x-axis is a
square. Find the volume of this solid.
(d) The region R models the surface of a small pond. At all points in R at a distance x from the y-axis,
the depth of the water is given by h x ! 3 n x. Find the volume of water in the pond.

(a) sin x ! x3 n 4 x at x ! 0 and x ! 2 1 : limits


2 3: 1 : integrand
Area ! sin x n x3 n 4 x dx ! 4
0 1 : answer

(b) x3 n 4 x ! n2 at r ! 0.5391889 and s ! 1.6751309 1 : limits


s 2:
The area of the stated region is n2 n x3 n 4 x dx 1 : integrand
r

2 2 1 : integrand
(c) Volume ! sin x n x3 n 4 x dx ! 9.978 2:
0 1 : answer

2 1 : integrand
(d) Volume ! 3 n x sin x n x3 n 4 x dx ! 8.369 or 8.370 2:
0 1 : answer

8LI'SPPIKI&SEVH%PPVMKLXWVIWIVZIH
:MWMXXLI'SPPIKI&SEVHSRXLI;IF[[[GSPPIKIFSEVHGSQ
%4’'%0'9097&'
7'36-2++9-()0-2)7
5YIWXMSR

t (hours) 0 1 3 4 7 8 9

L t (people) 120 156 176 126 150 80 0

Concert tickets went on sale at noon t ! 0 and were sold out within 9 hours. The number of people waiting in
line to purchase tickets at time t is modeled by a twice-differentiable function L for 0 t 9. Values of L t at
various times t are shown in the table above.
(a) Use the data in the table to estimate the rate at which the number of people waiting in line was changing at
5:30 P.M. t ! 5.5 . Show the computations that lead to your answer. Indicate units of measure.
(b) Use a trapezoidal sum with three subintervals to estimate the average number of people waiting in line during
the first 4 hours that tickets were on sale.
(c) For 0 t 9, what is the fewest number of times at which L t must equal 0 ? Give a reason for your answer.

(d) The rate at which tickets were sold for 0 t 9 is modeled by r t ! 550tent 2 tickets per hour. Based on the
model, how many tickets were sold by 3 P.M. t ! 3 , to the nearest whole number?

L 7 nL 4 150 n 126 1 : estimate


(a) L 5.5 8 people per hour 2:
7n4 3 1 : units
(b) The average number of people waiting in line during the first 4 hours is 1 : trapezoidal sum
approximately 2:
1 : answer
1 L 0 L1 L1 L 3 L3 L 4
1n 0  (3 n 1)  4n3
4 2 2 2
! 155.25 people
(c) L is differentiable on 0, 9 so the Mean Value Theorem implies 1 : considers change in
L t " 0 for some t in 1, 3 and some t in 4, 7 . Similarly, sign of L
3:
L t 0 for some t in 3, 4 and some t in 7, 8 . Then, since L is 1 : analysis
continuous on 0, 9 , the Intermediate Value Theorem implies that 1 : conclusion
L t ! 0 for at least three values of t in 0, 9 .

OR OR
The continuity of L on 1, 4 implies that L attains a maximum value 1 : considers relative extrema
there. Since L 3 " L 1 and L 3 " L 4 , this maximum occurs on of L on 0, 9
3:
1, 4 . Similarly, L attains a minimum on 3, 7 and a maximum on 1 : analysis
4, 8 . L is differentiable, so L t ! 0 at each relative extreme point 1 : conclusion
on 0, 9 . Therefore L t ! 0 for at least three values of t in 0, 9 .

[Note: There is a function L that satisfies the given conditions with


L t ! 0 for exactly three values of t.]
3
(d) r t dt ! 972.784 1 : integrand
0 2:
1 : limits and answer
There were approximately 973 tickets sold by 3 P.M.
8LI'SPPIKI&SEVH%PPVMKLXWVIWIVZIH
:MWMXXLI'SPPIKI&SEVHSRXLI;IF[[[GSPPIKIFSEVHGSQ
%4’'%0'9097&'
7'36-2++9-()0-2)7
5YIWXMSR

h x h x h x h x 4
x h x

1 11 30 42 99 18
488 448 584
2 80 128
3 3 9
753 1383 3483 1125
3 317
2 4 16 16

Let h be a function having derivatives of all orders for x " 0. Selected values of h and its first four
derivatives are indicated in the table above. The function h and these four derivatives are increasing on
the interval 1 x 3.
(a) Write the first-degree Taylor polynomial for h about x ! 2 and use it to approximate h 1.9 . Is this
approximation greater than or less than h 1.9 ? Explain your reasoning.
(b) Write the third-degree Taylor polynomial for h about x ! 2 and use it to approximate h 1.9 .
(c) Use the Lagrange error bound to show that the third-degree Taylor polynomial for h about x ! 2
approximates h 1.9 with error less than 3 » 10n 4.

(a) P1 x ! 80  128 x n 2 , so h 1.9 P1 1.9 67.2 2 : P1 x


4: 1 : P1 1.9
P1 1.9 h 1.9 since h is increasing on the interval 1 : P1 1.9 h 1.9 with reason
1 x 3.

488 2 448 3
(b) P3 x ! 80  128 x n 2  xn2  xn2 2 : P3 x
6 18 3:
1 : P3 1.9
h 1.9 P3 1.9 67.988

(c) The fourth derivative of h is increasing on the interval 1 : form of Lagrange error estimate
2:
584 1 : reasoning
1 x 3, so max h 4 x ! .
1.9 x 2 9
4
584 1.9 n 2
Therefore, h 1.9 P3 1.9
9 4!
! 2.7037 » 10n 4
3 » 10n 4

8LI'SPPIKI&SEVH%PPVMKLXWVIWIVZIH
:MWMXXLI'SPPIKI&SEVHSRXLI;IF[[[GSPPIKIFSEVHGSQ
%4’'%0'9097&'
7'36-2++9-()0-2)7
5YIWXMSR

A particle moves along the x-axis so that its velocity at time t, for 0 t 6, is given by a differentiable
function v whose graph is shown above. The velocity is 0 at t ! 0, t ! 3, and t ! 5, and the graph has
horizontal tangents at t ! 1 and t ! 4. The areas of the regions bounded by the t-axis and the graph of v on
the intervals 0, 3 , 3, 5 , and 5, 6 are 8, 3, and 2, respectively. At time t ! 0, the particle is at x ! n2.
(a) For 0 t 6, find both the time and the position of the particle when the particle is farthest to the left.
Justify your answer.
(b) For how many values of t, where 0 t 6, is the particle at x ! n8 ? Explain your reasoning.
(c) On the interval 2 t 3, is the speed of the particle increasing or decreasing? Give a reason for your
answer.
(d) During what time intervals, if any, is the acceleration of the particle negative? Justify your answer.

(a) Since v t 0 for 0 t 3 and 5 t 6, and v t " 0 1 : identifies t ! 3 as a candidate


for 3 t 5, we consider t ! 3 and t ! 6. 6
3: 1 : considers v t dt
3 0
x 3 ! n2  v t dt ! n2 n 8 ! n10
0 1 : conclusion
6
x 6 ! n2  v t dt ! n2 n 8  3 n 2 ! n9
0
Therefore, the particle is farthest left at time t ! 3 when
its position is x 3 ! n10.
(b) The particle moves continuously and monotonically from 1 : positions at t ! 3, t ! 5,
x 0 ! n2 to x 3 ! n10. Similarly, the particle moves and t ! 6
3:
continuously and monotonically from x 3 ! n10 to 1 : description of motion
x 5 ! n7 and also from x 5 ! n7 to x 6 ! n9. 1 : conclusion

By the Intermediate Value Theorem, there are three values


of t for which the particle is at x t ! n8.
(c) The speed is decreasing on the interval 2 t 3 since on 1 : answer with reason
this interval v 0 and v is increasing.
(d) The acceleration is negative on the intervals 0 t 1 and 1 : answer
2:
4 t 6 since velocity is decreasing on these intervals. 1 : justification

8LI'SPPIKI&SEVH%PPVMKLXWVIWIVZIH
:MWMXXLI'SPPIKI&SEVHSRXLI;IF[[[GSPPIKIFSEVHGSQ
%4’'%0'9097&'
7'36-2++9-()0-2)7
5YIWXMSR

The derivative of a function f is given by f x ! x n 3 e x for x " 0, and f 1 ! 7.


(a) The function f has a critical point at x ! 3. At this point, does f have a relative minimum, a relative
maximum, or neither? Justify your answer.
(b) On what intervals, if any, is the graph of f both decreasing and concave up? Explain your reasoning.
(c) Find the value of f 3 .

(a) f x 0 for 0 x 3 and f x " 0 for x " 3 1: minimum at x ! 3


2:
1: justification
Therefore, f has a relative minimum at x ! 3.

(b) f x ! ex  x n 3 ex ! x n 2 ex 2: f x
3:
f x " 0 for x " 2 1 : answer with reason

f x 0 for 0 x 3

Therefore, the graph of f is both decreasing and concave up on the


interval 2 x 3.

3 3
(c) f 3 ! f 1  f x dx ! 7  x n 3 e x dx 1 : uses initial condition
1 1
4: 2 : integration by parts
u ! x n 3 dv ! e x dx
1 : answer
du ! dx v ! ex
3 3 x
f 3 ! 7  x n 3 ex n e dx
1 1
3
!7 x n 3 ex n ex
1
3
! 7  3e n e

8LI'SPPIKI&SEVH%PPVMKLXWVIWIVZIH
:MWMXXLI'SPPIKI&SEVHSRXLI;IF[[[GSPPIKIFSEVHGSQ
%4’'%0'9097&'
7'36-2++9-()0-2)7
5YIWXMSR
dy y
Consider the logistic differential equation ! 6 n y . Let y ! f t be the particular solution to the
dt 8
differential equation with f 0 ! 8.
(a) A slope field for this differential equation is given below. Sketch possible
solution curves through the points 3, 2 and 0, 8 .
(Note: Use the axes provided in the exam booklet.)

(b) Use Euler’s method, starting at t ! 0 with two steps of equal size, to
approximate f 1 .
(c) Write the second-degree Taylor polynomial for f about t ! 0, and use it
to approximate f 1 .
(d) What is the range of f for t 0?

(a) 1: solution curve through 0,8


2:
1: solution curve through 3, 2

1 1
(b) f 8 2 7 1 : Euler’s method with two steps
2 2 2:
1 : approximation of f 1
7 1 105
f 1 7
8 2 16

d 2 y 1 dy y dy d2y
(c) ! 6n y  n 2:
dt 2 8 dt 8 dt dt 2
4:
dy 8 1 : second-degree Taylor polynomial
f 0 ! 8; f 0 ! ! 6 n 8 ! n2; and
dt t ! 0 8 1 : approximation of f 1
d2y 1 8 5
f 0 ! ! n2 n2  2 !
dt 2 8 8 2
t !0
The second-degree Taylor polynomial for f about
5
t ! 0 is P2 t ! 8 n 2t  t 2 .
4
29
f 1 P2 1
4
(d) The range of f for t 0 is 6 y 8. 1 : answer

8LI'SPPIKI&SEVH%PPVMKLXWVIWIVZIH
:MWMXXLI'SPPIKI&SEVHSRXLI;IF[[[GSPPIKIFSEVHGSQ
%4’'%0'9097&'
7'36-2++9-()0-2)7 *SVQ& 
5YIWXMSR

A particle moving along a curve in the xy-plane has position x t , y t at time t 0 with
2
dx dy t
! 3t and ! 3cos .
dt dt 2
The particle is at position 1, 5 at time t ! 4.
(a) Find the acceleration vector at time t ! 4.
(b) Find the y-coordinate of the position of the particle at time t ! 0.
(c) On the interval 0 t 4, at what time does the speed of the particle first reach 3.5 ?
(d) Find the total distance traveled by the particle over the time interval 0 t 4.

(a) a 4 ! x 4 , y 4 ! 0.433, n11.872 1 : answer

0 1 : integrand
(b) y 0 ! 5  t2
3cos dt ! 1.600 or 1.601 3: 1 : uses y 4 ! 5
4
2
1 : answer

2 2
(c) Speed ! x t  y t 1 : expression for speed
3: 1 : equation
t2 1 : answer
! 3t  9cos 2 ! 3.5
2

The particle first reaches this speed when


t ! 2.225 or 2.226.

4
t2 1 : integral
(d) 3t  9cos 2 dt ! 13.182 2:
2 1 : answer
0

8LI'SPPIKI&SEVH%PPVMKLXWVIWIVZIH
:MWMXXLI'SPPIKI&SEVHSRXLI;IF[[[GSPPIKIFSEVHGSQ
%4’'%0'9097&'
7'36-2++9-()0-2)7 *SVQ& 
5YIWXMSR

2
For time t 0 hours, let r t ! 120 1 n e n10t represent the speed, in kilometers per hour, at which a
car travels along a straight road. The number of liters of gasoline used by the car to travel x kilometers is
modeled by g x ! 0.05 x 1 n en x 2 .

(a) How many kilometers does the car travel during the first 2 hours?
(b) Find the rate of change with respect to time of the number of liters of gasoline used by the car when
t ! 2 hours. Indicate units of measure.
(c) How many liters of gasoline have been used by the car when it reaches a speed of 80 kilometers per
hour?

2 1 : integral
(a) r t dt ! 206.370 kilometers 2:
0 1 : answer

dg dg dx dx
(b) ; !r t 2 : uses chain rule
dt dx dt dt 3:
1 : answer with units
dg dg
r 2
dt t !2
dx x ! 206.370
! 0.050 120 ! 6 liters hour

(c) Let T be the time at which the car’s speed reaches 1 : equation r t ! 80
80 kilometers per hour. 4: 2 : distance integral
1 : answer
Then, r T ! 80 or T ! 0.331453 hours.

At time T, the car has gone


T
xT ! r t dt ! 10.794097 kilometers
0
and has consumed g x T ! 0.537 liters of gasoline.

8LI'SPPIKI&SEVH%PPVMKLXWVIWIVZIH
:MWMXXLI'SPPIKI&SEVHSRXLI;IF[[[GSPPIKIFSEVHGSQ
%4’'%0'9097&'
7'36-2++9-()0-2)7 *SVQ& 

5YIWXMSR

Distance from the


0 8 14 22 24
river’s edge (feet)
Depth of the water (feet) 0 7 8 2 0

A scientist measures the depth of the Doe River at Picnic Point. The river is 24 feet wide at this location.
The measurements are taken in a straight line perpendicular to the edge of the river. The data are shown
in the table above. The velocity of the water at Picnic Point, in feet per minute, is modeled by
v t ! 16  2sin t  10 for 0 t 120 minutes.
(a) Use a trapezoidal sum with the four subintervals indicated by the data in the table to approximate the
area of the cross section of the river at Picnic Point, in square feet. Show the computations that lead
to your answer.
(b) The volumetric flow at a location along the river is the product of the cross-sectional area and the
velocity of the water at that location. Use your approximation from part (a) to estimate the average
value of the volumetric flow at Picnic Point, in cubic feet per minute, from t ! 0 to t ! 120
minutes.
x
(c) The scientist proposes the function f, given by f x ! 8sin , as a model for the depth of the
24
water, in feet, at Picnic Point x feet from the river’s edge. Find the area of the cross section of the
river at Picnic Point based on this model.
(d) Recall that the volumetric flow is the product of the cross-sectional area and the velocity of the water
at a location. To prevent flooding, water must be diverted if the average value of the volumetric flow
at Picnic Point exceeds 2100 cubic feet per minute for a 20-minute period. Using your answer from
part (c), find the average value of the volumetric flow during the time interval 40 t 60 minutes.
Does this value indicate that the water must be diverted?

07 78 82 20


(a) 8 6 8 2 1 : trapezoidal approximation
2 2 2 2
! 115 ft 2

1 120
(b) 115v t dt 1 : limits and average value
120 0
constant
! 1807.169 or 1807.170 ft 3 min 3:
1 : integrand
1 : answer

24 x 1 : integra1
(c) 8sin dx ! 122.230 or 122.231 ft 2 2:
0 24 1 : answer
(d) Let C be the cross-sectional area approximation from 1 : volumetric flow integral
part (c). The average volumetric flow is 3: 1 : average volumetric flow
1 60
C v t dt 2181.912 or 2181.913 ft 3 min . 1 : answer with reason
20 40

Yes, water must be diverted since the average volumetric flow


for this 20-minute period exceeds 2100 ft 3 min.

8LI'SPPIKI&SEVH%PPVMKLXWVIWIVZIH
:MWMXXLI'SPPIKI&SEVHSRXLI;IF[[[GSPPIKIFSEVHGSQ
%4’'%0'9097&'
7'36-2++9-()0-2)7 *SVQ& 
5YIWXMSR

Let f be the function given by f x ! kx 2 n x3 , where k is a positive constant. Let R be the region in
the first quadrant bounded by the graph of f and the x-axis.
(a) Find all values of the constant k for which the area of R equals 2.
(b) For k " 0, write, but do not evaluate, an integral expression in terms of k for the volume of the solid
generated when R is rotated about the x-axis.
(c) For k " 0, write, but do not evaluate, an expression in terms of k, involving one or more integrals,
that gives the perimeter of R.

(a) For x 0, f x x2 k x 0 if 0 x k 1 : integral


k k 3 1 4 x !k
k4 1 : antiderivative
kx 2 n x3 dx ! x n x ! 4:
0 3 4 x !0 12 1 : value of integral
1 : answer
k4
Area ! ! 2; k ! 4 24
12

k 2 1 : integrand
(b) Volume ! kx 2 n x3 dx 2:
0 1 : limits and constant

k 2
1: 1 f x dx
k 0
2 2
(c) Perimeter ! k  1  2kx n 3 x dx 3:
0 1 : uses f x ! 2kx n 3x 2 in integrand
1 : answer

8LI'SPPIKI&SEVH%PPVMKLXWVIWIVZIH
:MWMXXLI'SPPIKI&SEVHSRXLI;IF[[[GSPPIKIFSEVHGSQ
%4’'%0'9097&'
7'36-2++9-()0-2)7 *SVQ& 
5YIWXMSR

Let g be a continuous function with g 2 ! 5. The graph


of the piecewise-linear function g , the derivative of g, is
shown above for 3 x 7.
(a) Find the x-coordinate of all points of inflection of the
graph of y ! g x for n3 x 7. Justify your
answer.
(b) Find the absolute maximum value of g on the
interval 3 x 7. Justify your answer.
(c) Find the average rate of change of g x on the
interval 3 x 7.
(d) Find the average rate of change of g x on the interval 3 x 7. Does the Mean Value Theorem
applied on the interval 3 x 7 guarantee a value of c, for n3 c 7, such that g c is equal
to this average rate of change? Why or why not?

(a) g changes from increasing to decreasing at x ! 1; 1 : x-values


2:
g changes from decreasing to increasing at x ! 4. 1 : justification

Points of inflection for the graph of y ! g x occur at


x ! 1 and x ! 4.

(b) The only sign change of g from positive to negative in 1 : identifies x ! 2 as a candidate
the interval is at x ! 2. 3: 1 : considers endpoints
1 : maximum value and justification
n3 3 15
g n3 ! 5  g x dx ! 5  n 4!
2 2 2
g 2 !5
7 1 3
g 7 !5 g x dx ! 5  n 4  !
2 2 2

15
The maximum value of g for 3 x 7 is .
2

3 15
g 7 n g n3 n 1 : difference quotient
(c) ! 2 2 ! n3 2:
7 n n3 10 5 1 : answer

g 7 n g n3 1 n n4 1
(d) ! ! 1 : average value of g x
7 n n3 10 2 2:
1 : answer “No” with reason
No, the MVT does not guarantee the existence of a
value c with the stated properties because g is not
differentiable for at least one point in n3 x 7.

8LI'SPPIKI&SEVH%PPVMKLXWVIWIVZIH
:MWMXXLI'SPPIKI&SEVHSRXLI;IF[[[GSPPIKIFSEVHGSQ
%4’'%0'9097&'
7'36-2++9-()0-2)7 *SVQ& 
5YIWXMSR

2x
Let f be the function given by f x ! .
1  x2
(a) Write the first four nonzero terms and the general term of the Taylor series for f about x ! 0.
(b) Does the series found in part (a), when evaluated at x ! 1, converge to f 1 ? Explain why or why
not.
2x
(c) The derivative of ln 1  x 2 is . Write the first four nonzero terms of the Taylor series for
1  x2
ln 1  x 2 about x ! 0.

5 1
(d) Use the series found in part (c) to find a rational number A such that A n ln . Justify
4 100
your answer.

1
(a) ! 1  u  u2   un  1 : two of the first four terms
1n u
3: 1 : remaining terms
1 2 4 6 2 n
! 1 n x  x n x   n x  1 : general term
1  x2
2x
2
! 2 x n 2 x3  2 x5 n 2 x 7   (n1) n 2 x 2 n 1 
1 x

(b) No, the series does not converge when x ! 1 because when 1 : answer with reason
x ! 1, the terms of the series do not converge to 0.

x
2t
(c) ln 1  x 2 ! dt 1 : two of the first four terms
0 1  t2 2:
x
1 : remaining terms
! 2t n 2t 3  2t 5 n 2t 7  dt
0
1 4 1 6 1 8
! x2 n x  x n x 
2 3 4

2 4 6 8
5 1 1 1 1 1 1 1 1 1
(d) ln ! ln 1  ! n  n  1 : uses x !
4 4 2 2 2 3 2 4 2 2
3: 1 : value of A
1 2 1 1 4 7
Let A ! n ! . 1 : justification
2 2 2 32
Since the series is a converging alternating series and the
absolute values of the individual terms decrease to 0,
6
5 1 1 1 1 1
A ln .
4 3 2 3 64 100

8LI'SPPIKI&SEVH%PPVMKLXWVIWIVZIH
:MWMXXLI'SPPIKI&SEVHSRXLI;IF[[[GSPPIKIFSEVHGSQ
%4’'%0'9097&'
7'36-2++9-()0-2)7
5YIWXMSR

Caren rides her bicycle along a straight road from home to school, starting at home at time t ! 0 minutes
and arriving at school at time t ! 12 minutes. During the time interval 0 t 12 minutes, her velocity
v t , in miles per minute, is modeled by the piecewise-linear function whose graph is shown above.
(a) Find the acceleration of Caren’s bicycle at time t ! 7.5 minutes. Indicate units of measure.
12
(b) Using correct units, explain the meaning of v t dt in terms of Caren’s trip. Find the value
0
12
of v t dt.
0

(c) Shortly after leaving home, Caren realizes she left her calculus homework at home, and she returns to
get it. At what time does she turn around to go back home? Give a reason for your answer.
(d) Larry also rides his bicycle along a straight road from home to school in 12 minutes. His velocity is
modeled by the function w given by w t ! sin t , where w t is in miles per minute for
15 12
0 t 12 minutes. Who lives closer to school: Caren or Larry? Show the work that leads to your
answer.

v 8 nv 7 1 : answer
(a) a 7.5 ! v 7.5 ! ! n 0.1 miles minute2 2:
8n7 1 : units
12
(b) v t dt is the total distance, in miles, that Caren rode 1 : meaning of integral
0 2:
1 : value of integral
during the 12 minutes from t ! 0 to t ! 12.
12 2 4 12
v t dt ! v t dt n v t dt  v t dt
0 0 2 4
! 0.2  0.2  1.4 ! 1.8 miles
(c) Caren turns around to go back home at time t ! 2 minutes. 1 : answer
2:
This is the time at which her velocity changes from positive 1 : reason
to negative.
12
(d) w t dt ! 1.6; Larry lives 1.6 miles from school. 2 : Larry’s distance from school
0
12
1 : integral
v t dt ! 1.4; Caren lives 1.4 miles from school. 3: 1 : value
0
Therefore, Caren lives closer to school. 1 : Caren’s distance from school
and conclusion

8LI'SPPIKI&SEVH%PPVMKLXWVIWIVZIH
:MWMXXLI'SPPIKI&SEVHSRXLI;IF[[[GSPPIKIFSEVHGSQ
%4’'%0'9097&'
7'36-2++9-()0-2)7
5YIWXMSR

The rate at which people enter an auditorium for a rock concert is modeled by the function R given by
R t ! 1380t 2 n 675t 3 for 0 t 2 hours; R t is measured in people per hour. No one is in the
auditorium at time t ! 0, when the doors open. The doors close and the concert begins at time t ! 2.
(a) How many people are in the auditorium when the concert begins?
(b) Find the time when the rate at which people enter the auditorium is a maximum. Justify your answer.
(c) The total wait time for all the people in the auditorium is found by adding the time each person waits,
starting at the time the person enters the auditorium and ending when the concert begins. The function
w models the total wait time for all the people who enter the auditorium before time t. The derivative
of w is given by w t ! 2 n t R t . Find w 2 n w 1 , the total wait time for those who enter the
auditorium after time t ! 1.
(d) On average, how long does a person wait in the auditorium for the concert to begin? Consider all people
who enter the auditorium after the doors open, and use the model for total wait time from part (c).

2 1 : integral
(a) R t dt ! 980 people 2:
0 1 : answer

(b) R t ! 0 when t ! 0 and t ! 1.36296 1 : considers R t ! 0


The maximum rate may occur at 0, a ! 1.36296, or 2. 3: 1 : interior critical point
1 : answer and justification
R 0 !0
R a ! 854.527
R 2 ! 120

The maximum rate occurs when t ! 1.362 or 1.363.

2 2
(c) w 2 n w 1 ! w t dt ! 2 n t R t dt ! 387.5 1 : integral
1 1 2:
1 : answer
The total wait time for those who enter the auditorium after
time t ! 1 is 387.5 hours.

1 1 2
(d) w2 ! 2 n t R t dt ! 0.77551 1 : integral
980 980 0 2:
1 : answer
On average, a person waits 0.775 or 0.776 hour.

8LI'SPPIKI&SEVH%PPVMKLXWVIWIVZIH
:MWMXXLI'SPPIKI&SEVHSRXLI;IF[[[GSPPIKIFSEVHGSQ
%4’'%0'9097&'
7'36-2++9-()0-2)7
5YIWXMSR

A diver leaps from the edge of a diving platform into a pool below. The
figure above shows the initial position of the diver and her position at a later
time. At time t seconds after she leaps, the horizontal distance from the front
edge of the platform to the diver’s shoulders is given by x t , and the
vertical distance from the water surface to her shoulders is given by y t ,
where x t and y t are measured in meters. Suppose that the diver’s
shoulders are 11.4 meters above the water when she makes her leap and that
dx dy
! 0.8 and ! 3.6 n 9.8t ,
dt dt
for 0 t A, where A is the time that the diver’s shoulders enter the water.

(a) Find the maximum vertical distance from the water surface to the diver’s
shoulders.
(b) Find A, the time that the diver’s shoulders enter the water.
(c) Find the total distance traveled by the diver’s shoulders from the time
she leaps from the platform until the time her shoulders enter the water.

(d) Find the angle , 0 , between the path of the diver and the
2
water at the instant the diver’s shoulders enter the water.

dy dy
(a) ! 0 only when t ! 0.36735. Let b ! 0.36735. 1 : considers !0
dt dt
3: 1 : integral or y t
The maximum vertical distance from the water surface to the diver’s
shoulders is 1 : answer
b
dy
y b ! 11.4  dt ! 12.061 meters.
0 dt

Alternatively, y t ! 11.4  3.6t n 4.9t 2 , so y b ! 12.061 meters.

A
dy
(b) y A ! 11.4  dt ! 11.4  3.6 A n 4.9 A2 ! 0 when 1 : equation
0 dt 2:
1 : answer
A ! 1.936 seconds.

A 2 2
dx dy 1 : integral
(c)  dt ! 12.946 meters 2:
0
dt dt 1 : answer

dy dy dt dy
(d) At time A, ! ! n19.21913. 1: at time A
dx dx dt t!A
2: dx
1 : answer
The angle between the path of the diver and the water is
tan n1 19.21913 ! 1.518 or 1.519.

8LI'SPPIKI&SEVH%PPVMKLXWVIWIVZIH
:MWMXXLI'SPPIKI&SEVHSRXLI;IF[[[GSPPIKIFSEVHGSQ
%4’'%0'9097&'
7'36-2++9-()0-2)7
5YIWXMSR

dy
Consider the differential equation ! 6 x 2 n x 2 y. Let y ! f x be a particular solution to this
dx
differential equation with the initial condition f n1 ! 2.
(a) Use Euler’s method with two steps of equal size, starting at x ! n1, to approximate f 0 . Show the
work that leads to your answer.
d2y
(b) At the point n1, 2 , the value of is n12. Find the second-degree Taylor polynomial for
dx 2
f about x ! n1.
(c) Find the particular solution y ! f x to the given differential equation with the initial condition
f n1 ! 2.

1 dy
(a) f f 1 x 1 : Euler’s method with two steps
2 dx n1, 2 2:
1 : answer
1
2 4 4
2

1 dy
f 0 f x
2 dx 1
n ,4
2
1 1 17
4
2 2 4

2
(b) P2 x ! 2  4 x  1 n 6 x  1 1 : answer

dy
(c) ! x2 6 n y 1 : separation of variables
dx
2 : antiderivatives
1
dy ! x 2 dx 6: 1 : constant of integration
6n y
1 1 : uses initial condition
n ln 6 n y ! x3  C 1 : solves for y
3
1
n ln 4 ! n  C
3 Note: max 3 6 [1-2-0-0-0] if no constant of
1 integration
C ! n ln 4
3 Note: 0 6 if no separation of variables
1 1
ln 6 n y ! n x3 n n ln 4
3 3
1 3
n x 1
6 n y ! 4e 3

1 3
n x 1
y ! 6 n 4e 3

8LI'SPPIKI&SEVH%PPVMKLXWVIWIVZIH
:MWMXXLI'SPPIKI&SEVHSRXLI;IF[[[GSPPIKIFSEVHGSQ
%4’'%0'9097&'
7'36-2++9-()0-2)7
5YIWXMSR

x 2 3 5 8 13
f x 1 4 –2 3 6

Let f be a function that is twice differentiable for all real numbers. The table above gives values of f for
selected points in the closed interval 2 x 13.
(a) Estimate f 4 . Show the work that leads to your answer.
13
(b) Evaluate 3 n 5f x dx. Show the work that leads to your answer.
2

13
(c) Use a left Riemann sum with subintervals indicated by the data in the table to approximate f x dx.
2
Show the work that leads to your answer.
(d) Suppose f 5 ! 3 and f x 0 for all x in the closed interval 5 x 8. Use the line tangent to
the graph of f at x ! 5 to show that f 7 4. Use the secant line for the graph of f on 5 x 8 to
4
show that f 7 .
3

f 5 n f 3
(a) f 4 n3 1 : answer
5n3
13 13 13
(b) 3 n 5f x dx ! 3 dx n 5 f x dx 1 : uses Fundamental Theorem
2 2 2
2: of Calculus
! 3 13 n 2 n 5 f (13) n f (2) ! 8
1 : answer
13
(c) f x dx f 2 3 2 f 3 5 3 1 : left Riemann sum
2 2:
1 : answer
 f 5 8 n 5  f 8 13 n 8 ! 18

(d) An equation for the tangent line is y ! n2  3 x n 5 . 1 : tangent line


Since f x 0 for all x in the interval 5 x 8, the 1 : shows f 7 4
line tangent to the graph of y ! f x at x ! 5 lies above 4: 1 : secant line
the graph for all x in the interval 5 x 8. 4
1 : shows f 7
3
Therefore, f 7 2 3 2 4.
5
An equation for the secant line is y ! n2  xn5 .
3
Since f x 0 for all x in the interval 5 x 8, the
secant line connecting 5, f 5 and 8, f 8 lies below
the graph of y ! f x for all x in the interval 5 x 8.
5 4
Therefore, f 7 2 2 .
3 3

8LI'SPPIKI&SEVH%PPVMKLXWVIWIVZIH
:MWMXXLI'SPPIKI&SEVHSRXLI;IF[[[GSPPIKIFSEVHGSQ
%4’'%0'9097&'
7'36-2++9-()0-2)7
5YIWXMSR

x 2 x3 xn
The Maclaurin series for e x is e x ! 1  x      . The continuous function f is defined
2 6 n!
x n1 2
e n1
by f x ! 2
for x 1 and f 1 ! 1. The function f has derivatives of all orders at x ! 1.
x n1
x n1 2
(a) Write the first four nonzero terms and the general term of the Taylor series for e about x ! 1.
(b) Use the Taylor series found in part (a) to write the first four nonzero terms and the general term of the
Taylor series for f about x ! 1.
(c) Use the ratio test to find the interval of convergence for the Taylor series found in part (b).
(d) Use the Taylor series for f about x ! 1 to determine whether the graph of f has any points of
inflection.

4 6 2n
2 x n1 x n1 x n1 1 : first four terms
(a) 1  x n 1      2:
2 6 n! 1 : general term

2 4 6
x n1 x n1 x n1 x n 1 2n 1 : first four terms
(b) 1       2:
2 6 24 n 1 ! 1 : general term

x n 1 2n2
n2 ! n 1 ! 2 x n1 2
(c) lim ! lim x n1 ! lim !0 1 : sets up ratio
n x n 1 2n n n2 ! n n2
3: 1 : computes limit of ratio
n 1 !
1 : answer
Therefore, the interval of convergence is n , .

4 3 6 5
(d) f x !1 x n1 2  x n1 4  1: f x
6 24 2:
2n 2 n n 1 1 : answer
 x n 1 2n n 2 
(n  1)!

Since every term of this series is nonnegative, f x 0 for all x.


Therefore, the graph of f has no points of inflection.

8LI'SPPIKI&SEVH%PPVMKLXWVIWIVZIH
:MWMXXLI'SPPIKI&SEVHSRXLI;IF[[[GSPPIKIFSEVHGSQ
%4’'%0'9097&'
7'36-2++9-()0-2)7 *SVQ& 
5YIWXMSR

A baker is creating a birthday cake. The base of the cake is the


region R in the first quadrant under the graph of y ! f x for
x
0 x 30, where f x ! 20sin . Both x and y are
30
measured in centimeters. The region R is shown in the figure
2 x
above. The derivative of f is f x ! cos .
3 30

(a) The region R is cut out of a 30-centimeter-by-20-


centimeter rectangular sheet of cardboard, and the
remaining cardboard is discarded. Find the area of the
discarded cardboard.
(b) The cake is a solid with base R. Cross sections of the cake perpendicular to the x-axis are
semicircles. If the baker uses 0.05 gram of unsweetened chocolate for each cubic centimeter of cake,
how many grams of unsweetened chocolate will be in the cake?
(c) Find the perimeter of the base of the cake.

30 2 : integral
(a) Area 30 20 f x dx 218.028 cm 2 3:
0 1 : answer

30 2
f x
(b) Volume ! dx ! 2356.194 cm3 2 : integral
0
2 2 3:
1 : answer
Therefore, the baker needs 2356.194 » 0.05 ! 117.809 or 117.810 grams
of chocolate.

30 2 2 : integral
(c) Perimeter ! 30  1 f x dx ! 81.803 or 81.804 cm 3:
0 1 : answer

8LI'SPPIKI&SEVH%PPVMKLXWVIWIVZIH
:MWMXXLI'SPPIKI&SEVHSRXLI;IF[[[GSPPIKIFSEVHGSQ
%4’'%0'9097&'
7'36-2++9-()0-2)7 *SVQ& 
5YIWXMSR

A storm washed away sand from a beach, causing the edge of the water to get closer to a nearby road. The
rate at which the distance between the road and the edge of the water was changing during the storm is
modeled by f t ! t  cos t n 3 meters per hour, t hours after the storm began. The edge of the water
was 35 meters from the road when the storm began, and the storm lasted 5 hours. The derivative of f t
1
is f t ! n sin t.
2 t
(a) What was the distance between the road and the edge of the water at the end of the storm?
(b) Using correct units, interpret the value f 4 ! 1.007 in terms of the distance between the road and
the edge of the water.
(c) At what time during the 5 hours of the storm was the distance between the road and the edge of the
water decreasing most rapidly? Justify your answer.
(d) After the storm, a machine pumped sand back onto the beach so that the distance between the road
and the edge of the water was growing at a rate of g p meters per day, where p is the number of
days since pumping began. Write an equation involving an integral expression whose solution would
give the number of days that sand must be pumped to restore the original distance between the road
and the edge of the water.

5 1 : integral
(a) 35  f t dt ! 26.494 or 26.495 meters 2:
0 1 : answer

(b) Four hours after the storm began, the rate of change of 1 : interpretation of f 4
the distance between the road and the edge of the water 2:
1 : units
is increasing at a rate of 1.007 meters hours 2 .

(c) f t ! 0 when t ! 0.66187 and t ! 2.84038 1 : considers f t ! 0


The minimum of f for 0 t 5 may occur at 0, 3: 1 : answer
0.66187, 2.84038, or 5. 1 : justification

f 0 ! n2
f 0.66187 ! n1.39760
f 2.84038 ! n2.26963
f 5 ! n0.48027

The distance between the road and the edge of the


water was decreasing most rapidly at time t ! 2.840
hours after the storm began.

5 x 1 : integral of g
(d) n f t dt ! g p dp 2:
0 0 1 : answer

8LI'SPPIKI&SEVH%PPVMKLXWVIWIVZIH
:MWMXXLI'SPPIKI&SEVHSRXLI;IF[[[GSPPIKIFSEVHGSQ
%4’'%0'9097&'
7'36-2++9-()0-2)7 *SVQ& 
5YIWXMSR

A continuous function f is defined on the closed interval 4 x 6. The graph of f consists of a line
segment and a curve that is tangent to the x-axis at x ! 3, as shown in the figure above. On the interval
0 x 6, the function f is twice differentiable, with f x " 0.
(a) Is f differentiable at x ! 0 ? Use the definition of the derivative with one-sided limits to justify your
answer.
(b) For how many values of a, 4 a 6, is the average rate of change of f on the interval a , 6
equal to 0 ? Give a reason for your answer.
(c) Is there a value of a, 4 a 6, for which the Mean Value Theorem, applied to the interval a, 6 ,
1
guarantees a value c, a c 6, at which f c ! ? Justify your answer.
3
x
(d) The function g is defined by g x ! f t dt for 4 x 6. On what intervals contained in
0
n 4, 6 is the graph of g concave up? Explain your reasoning.

f h n f 0 2
(a) lim ! 1 : sets up difference quotient at x ! 0
h 0n h 3 2:
1 : answer with justification
f h n f 0
lim 0
h 0  h
Since the one-sided limits do not agree, f is not
differentiable at x ! 0.
f 6 n f a
(b) ! 0 when f a ! f 6 . There are 1 : expression for average rate of change
6na 2:
two values of a for which this is true. 1 : answer with reason
(c) Yes, a ! 3. The function f is differentiable on the 1 : answers “yes” and identifies a ! 3
2:
interval 3 x 6 and continuous on 3 x 6. 1 : justification
f 6 n f 3 1n 0 1
Also, ! ! .
6n3 6n3 3
By the Mean Value Theorem, there is a value c,
1
3 c 6, such that f c ! .
3
(d) g x ! f x , g x ! f x 1: g x ! f x
g x " 0 when f x " 0 3: 1 : considers g x " 0
This is true for n 4 x 0 and 3 x 6. 1 : answer

8LI'SPPIKI&SEVH%PPVMKLXWVIWIVZIH
:MWMXXLI'SPPIKI&SEVHSRXLI;IF[[[GSPPIKIFSEVHGSQ
%4’'%0'9097&'
7'36-2++9-()0-2)7 *SVQ& 
5YIWXMSR

The graph of the polar curve r ! 1 n 2cos for 0 is


shown above. Let S be the shaded region in the third quadrant
bounded by the curve and the x-axis.

(a) Write an integral expression for the area of S.


dx dy
(b) Write expressions for and in terms of .
d d
(c) Write an equation in terms of x and y for the line tangent to
the graph of the polar curve at the point where ! .
2
Show the computations that lead to your answer.

(a) r 0 ! n1; r ! 0 when ! . 1 : limits and constant


3 2:
3 1 : integrand
1 2
Area of S ! 1 n 2cos d
2 0

(b) x ! r cos and y ! r sin 1 : uses x ! r cos and y ! r sin


4: dr
dr 1:
! 2sin d
d 2 : answer
dx dr
! cos n r sin ! 4sin cos n sin
d d
dy dr
! sin  r cos ! 2sin 2  1 n 2 cos cos
d d

(c) When ! , we have x ! 0, y ! 1. 1 : values for x and y


2
dy
dy dy d 3: 1 : expression for
! ! n2 dx
dx dx d
!
2 !
2 1 : tangent line equation
The tangent line is given by y ! 1 n 2 x.

8LI'SPPIKI&SEVH%PPVMKLXWVIWIVZIH
:MWMXXLI'SPPIKI&SEVHSRXLI;IF[[[GSPPIKIFSEVHGSQ
%4’'%0'9097&'
7'36-2++9-()0-2)7 *SVQ& 
5YIWXMSR

Let f be a twice-differentiable function defined on the interval


n1.2 x 3.2 with f 1 ! 2. The graph of f , the derivative
of f, is shown above. The graph of f crosses the x-axis at
x ! n1 and x ! 3 and has a horizontal tangent at x ! 2. Let g
be the function given by g x ! e f x
.

(a) Write an equation for the line tangent to the graph of g


at x ! 1.
(b) For n1.2 x 3.2, find all values of x at which g has a
local maximum. Justify your answer.
2
(c) The second derivative of g is g x ! e f x
f x  f x . Is g n1 positive, negative, or
zero? Justify your answer.
(d) Find the average rate of change of g , the derivative of g, over the interval 1, 3 .

(a) g 1 ! e f 1
! e2 1: g x
g x !e f x
f x , g 1 !e f 1
f 1 ! n 4e 2 3: 1 : g 1 and g 1
1 : tangent line equation
The tangent line is given by y ! e 2 n 4e 2 x n 1 .

(b) g x ! e f x
f x 1 : answer
2:
ef x
" 0 for all x 1 : justification
So, g changes from positive to negative only when f
changes from positive to negative. This occurs at x ! n 1
only. Thus, g has a local maximum at x ! n1.

2
(c) g n1 ! e f n1
f n1  f n1 1 : answer
2:
1 : justification
e f n1 " 0 and f n1 ! 0
Since f is decreasing on a neighborhood of n1,
f n1 0. Therefore, g n1 0.

g 3 ng 1 ef 3
f 3 n ef 1
f 1 1 : difference quotient
(d) ! ! 2e 2 2:
3 n1 2 1 : answer

8LI'SPPIKI&SEVH%PPVMKLXWVIWIVZIH
:MWMXXLI'SPPIKI&SEVHSRXLI;IF[[[GSPPIKIFSEVHGSQ
%4’'%0'9097&'
7'36-2++9-()0-2)7 *SVQ& 

5YIWXMSR
The function f is defined by the power series
2 n n
f x ! 1 x 1  x 1   x 1  ! x 1
n!0
for all real numbers x for which the series converges.
(a) Find the interval of convergence of the power series for f. Justify your answer.
(b) The power series above is the Taylor series for f about x ! n1. Find the sum of the series for f.
x 1
(c) Let g be the function defined by g x ! f t dt. Find the value of g n , if it exists, or explain
n1 2
1
why g n cannot be determined.
2

(d) Let h be the function defined by h x ! f x 2 n 1 . Find the first three nonzero terms and the general
1
term of the Taylor series for h about x ! 0, and find the value of h .
2

(a) The power series is geometric with ratio x  1 . 1 : identifies as geometric


The series converges if and only if x  1 1. 3: 1: x 1 1
Therefore, the interval of convergence is n2 x 0. 1 : interval of convergence

OR OR

n 1
x 1
lim n
! x 1 1 when n2 x 0 1 : sets up limit of ratio
n x 1
3: 1 : radius of convergence
At x ! n2, the series is n1 n , which diverges since the 1 : interval of convergence
n! 0

terms do not converge to 0. At x ! 0, the series is 1,


n! 0
which similarly diverges. Therefore, the interval of
convergence is n2 x 0.

(b) Since the series is geometric, 1 : answer


n 1 1
f x ! x 1 ! ! n for n2 x 0.
n !0
1n x 1 x
1 1
1 n
2 1 x !n
2 1 : antiderivative
(c) g n ! n dx ! n ln x ! ln 2 2:
2 n1 x x !n1 1 : value

(d) h x ! f x 2 n 1 ! 1  x 2  x 4   x2n  1 : first three terms


1 3 4 1 : general term
h ! f n ! 3:
2 4 3 1
1 : value of h
2

8LI'SPPIKI&SEVH%PPVMKLXWVIWIVZIH
:MWMXXLI'SPPIKI&SEVHSRXLI;IF[[[GSPPIKIFSEVHGSQ
%4’'%0'9097&'
7'36-2++9-()0-2)7

5YIWXMSR

There is no snow on Janet’s driveway when snow begins to fall at midnight. From midnight to 9 A.M., snow
accumulates on the driveway at a rate modeled by f t ! 7tecos t cubic feet per hour, where t is measured in
hours since midnight. Janet starts removing snow at 6 A.M. t ! 6 . The rate g t , in cubic feet per hour, at
which Janet removes snow from the driveway at time t hours after midnight is modeled by
0 for 0 t 6
g t 125 for 6 t 7
108 for 7 t 9 .
(a) How many cubic feet of snow have accumulated on the driveway by 6 A.M.?
(b) Find the rate of change of the volume of snow on the driveway at 8 A.M.
(c) Let h t represent the total amount of snow, in cubic feet, that Janet has removed from the driveway at time
t hours after midnight. Express h as a piecewise-defined function with domain 0 t 9.
(d) How many cubic feet of snow are on the driveway at 9 A.M.?

6 1 : integral
(a) f t dt ! 142.274 or 142.275 cubic feet 2:
0 1 : answer

(b) Rate of change is f 8 n g 8 ! n59.582 or n59.583 cubic feet per hour. 1 : answer

(c) h 0 ! 0 1 : h t for 0 t 6
t t 3: 1 : h t for 6 t 7
For 0 t 6, h t ! h 0  g s ds ! 0  0 ds ! 0.
0 0 1 : h t for 7 t 9
t t
For 6 t 7, h t ! h 6  g s ds ! 0  125 ds ! 125 t n 6 .
6 6
t t
For 7 t 9, h t ! h 7  g s ds ! 125  108 ds ! 125  108 t n 7 .
7 7

0 for 0 t 6
Thus, h t 125 t 6 for 6 t 7
125 108 t 7 for 7 t 9

1 : integral
9
(d) Amount of snow is f t dt n h 9 ! 26.334 or 26.335 cubic feet. 3: 1:h 9
0
1 : answer

8LI'SPPIKI&SEVH
:MWMXXLI'SPPIKI&SEVHSRXLI;IF[[[GSPPIKIFSEVHGSQ
%4’'%0'9097&'
7'36-2++9-()0-2)7

5YIWXMSR

t
0 2 5 7 8
(hours)
E t
(hundreds of 0 4 13 21 23
entries)
A zoo sponsored a one-day contest to name a new baby elephant. Zoo visitors deposited entries in a special box
between noon t ! 0 and 8 P.M. t ! 8 . The number of entries in the box t hours after noon is modeled by a
differentiable function E for 0 t 8. Values of E t , in hundreds of entries, at various times t are shown in
the table above.
(a) Use the data in the table to approximate the rate, in hundreds of entries per hour, at which entries were being
deposited at time t ! 6. Show the computations that lead to your answer.
1 8
(b) Use a trapezoidal sum with the four subintervals given by the table to approximate the value of E t dt.
8 0
1 8
Using correct units, explain the meaning of E t dt in terms of the number of entries.
8 0

(c) At 8 P.M., volunteers began to process the entries. They processed the entries at a rate modeled by the function
P, where P t ! t 3 n 30t 2  298t n 976 hundreds of entries per hour for 8 t 12. According to the model,
how many entries had not yet been processed by midnight t ! 12 ?
(d) According to the model from part (c), at what time were the entries being processed most quickly? Justify
your answer.

E 7 nE 5
(a) E 6 4 hundred entries per hour 1 : answer
7n5
1 8
(b) E t dt 1 : trapezoidal sum
8 0
3: 1 : approximation
1 E 0 E 2 E 2 E 5 E 5 E 7 E 7 E 8
2·  3·  2·  1· 1 : meaning
8 2 2 2 2
! 10.687 or 10.688
1 8
E t dt is the average number of hundreds of entries in the box
8 0
between noon and 8 P.M.
12 1 : integral
(c) 23 n P t dt ! 23 n 16 ! 7 hundred entries 2:
8 1 : answer
(d) P t ! 0 when t ! 9.183503 and t ! 10.816497. 1 : considers P t ! 0
t P (t ) 3: 1 : identifies candidates
8 0 1 : answer with justification
9.183503 5.088662
10.816497 2.911338
12 8
Entries are being processed most quickly at time t ! 12.

8LI'SPPIKI&SEVH
:MWMXXLI'SPPIKI&SEVHSRXLI;IF[[[GSPPIKIFSEVHGSQ
%4’'%0'9097&'
7'36-2++9-()0-2)7

5YIWXMSR

A particle is moving along a curve so that its position at time t is x t , y t , where x t ! t 2 n 4t  8 and
y t is not explicitly given. Both x and y are measured in meters, and t is measured in seconds. It is known
dy
that ! tet n 3 n 1.
dt
(a) Find the speed of the particle at time t ! 3 seconds.
(b) Find the total distance traveled by the particle for 0 t 4 seconds.
(c) Find the time t, 0 t 4, when the line tangent to the path of the particle is horizontal. Is the direction of
motion of the particle toward the left or toward the right at that time? Give a reason for your answer.
(d) There is a point with x-coordinate 5 through which the particle passes twice. Find each of the following.
(i) The two values of t when that occurs
(ii) The slopes of the lines tangent to the particle’s path at that point
1
(iii) The y-coordinate of that point, given y 2 ! 3 
e

2 2
(a) Speed ! x 3  y 3 ! 2.828 meters per second 1 : answer

(b) x t ! 2t n 4 1 : integral
2:
4 2 2 1 : answer
Distance ! 2t n 4  tet n 3 n 1 dt ! 11.587 or 11.588 meters
0

dy dy dt dy
(c) ! ! 0 when tet n 3 n 1 ! 0 and 2t 4 0 1 : considers !0
dx dx dt dx
This occurs at t ! 2.20794. 3: 1 : t ! 2.207 or 2.208
1 : direction of motion with
Since x 2.20794 " 0, the particle is moving toward the right at
reason
time t ! 2.207 or 2.208.

(d) x t ! 5 at t ! 1 and t ! 3 1 : t ! 1 and t ! 3


dy dy dt 3: 1 : slopes
At time t ! 1, the slope is ! ! 0.432.
dx t !1 dx dt t !1 1 : y -coordinate
dy dy dt
At time t ! 3, the slope is ! ! 1.
dx t !3 dx dt t !3
3
1 dy
y 1 ! y 3 !3  dt ! 4
e 2 dt

8LI'SPPIKI&SEVH
:MWMXXLI'SPPIKI&SEVHSRXLI;IF[[[GSPPIKIFSEVHGSQ
%4’'%0'9097&'
7'36-2++9-()0-2)7

5YIWXMSR

Let R be the region in the first quadrant bounded by the graph of y ! 2 x , the horizontal line y ! 6, and the
y-axis, as shown in the figure above.
(a) Find the area of R.
(b) Write, but do not evaluate, an integral expression that gives the volume of the solid generated when R is
rotated about the horizontal line y ! 7.
(c) Region R is the base of a solid. For each y, where 0 y 6, the cross section of the solid taken
perpendicular to the y-axis is a rectangle whose height is 3 times the length of its base in region R. Write,
but do not evaluate, an integral expression that gives the volume of the solid.

x !9 1 : integrand
9 4 3 2
(a) Area ! 6 n 2 x dx ! 6 x n x ! 18 3: 1 : antiderivative
0 3 x !0
1 : answer

9 2 2 2 : integrand
(b) Volume ! 7n2 x n 7n6 dx 3:
0 1 : limits and constant

y2
(c) Solving y ! 2 x for x yields x ! . 2 : integrand
4 3:
1 : answer
y2 y2 3 4
Each rectangular cross section has area 3 ! y .
4 4 16
6
3 4
Volume ! y dy
0 16

8LI'SPPIKI&SEVH
:MWMXXLI'SPPIKI&SEVHSRXLI;IF[[[GSPPIKIFSEVHGSQ
%4’'%0'9097&'
7'36-2++9-()0-2)7

5YIWXMSR

dy
Consider the differential equation ! 1 n y. Let y ! f x be the particular solution to this differential
dx
equation with the initial condition f 1 ! 0. For this particular solution, f x 1 for all values of x.
(a) Use Euler’s method, starting at x ! 1 with two steps of equal size, to approximate f 0 . Show the work
that leads to your answer.
f x
(b) Find lim . Show the work that leads to your answer.
x 1 x3 n1
dy
(c) Find the particular solution y ! f x to the differential equation ! 1 n y with the initial condition
dx
f 1 ! 0.

1 dy
(a) f f 1 x 1 : Euler’s method with two steps
2 dx 1, 0 2:
1 : answer
1 1
0 1
2 2

1 dy
f 0 f x
2 dx 1 1
,n
2 2
1 3 1 5
2 2 2 4

(b) Since f is differentiable at x ! 1, f is continuous at x ! 1. So, 1 : use of L’Hospital’s Rule


2:
3
lim f x ! 0 ! lim x n 1 and we may apply L’Hospital’s 1 : answer
x 1 x 1
Rule.
f x f x lim f x
x 1 1
lim ! lim ! !
x 1 x3 n1 x 1 3x 2
lim 3 x 2 3
x 1

dy
(c) !1n y 1 : separation of variables
dx
1 : antiderivatives
1
dy ! 1 dx 5: 1 : constant of integration
1n y
1 : uses initial condition
n ln 1 n y ! x  C 1 : solves for y
n ln 1 ! 1  C C ! n1
Note: max 2 5 [1-1-0-0-0] if no
ln 1 n y ! 1 n x
constant of integration
1 n y ! e1n x Note: 0 5 if no separation of variables

f x ! 1 n e1n x

8LI'SPPIKI&SEVH
:MWMXXLI'SPPIKI&SEVHSRXLI;IF[[[GSPPIKIFSEVHGSQ
%4’'%0'9097&'
7'36-2++9-()0-2)7

5YIWXMSR

cos x n 1
for x 0
f x ! x2
1
n for x ! 0
2
The function f, defined above, has derivatives of all orders. Let g be the function defined by
x
g x !1 f t dt.
0

(a) Write the first three nonzero terms and the general term of the Taylor series for cos x about x ! 0. Use this
series to write the first three nonzero terms and the general term of the Taylor series for f about x ! 0.
(b) Use the Taylor series for f about x ! 0 found in part (a) to determine whether f has a relative maximum,
relative minimum, or neither at x ! 0. Give a reason for your answer.
(c) Write the fifth-degree Taylor polynomial for g about x ! 0.
(d) The Taylor series for g about x ! 0, evaluated at x ! 1, is an alternating series with individual terms that
decrease in absolute value to 0. Use the third-degree Taylor polynomial for g about x ! 0 to estimate the
1
value of g 1 . Explain why this estimate differs from the actual value of g 1 by less than .
6!

x 2 x4 n x2n
(a) cos x ! 1 n  n  n1  1 : terms for cos x
2 4! 2n !
2 : terms for f
1 x2 x4 n 1 x 2n 3:
f x !n  n   n1  1 : first three terms
2 4! 6! 2n  2 !
1 : general term

(b) f 0 is the coefficient of x in the Taylor series for f about x ! 0, 1 : determines f 0


2:
so f 0 ! 0. 1 : answer with reason

f 0 1
! is the coefficient of x 2 in the Taylor series for f about
2! 4!
1
x ! 0, so f 0 ! .
12
Therefore, by the Second Derivative Test, f has a relative minimum at
x ! 0.

x x3 x5 1 : two correct terms


(c) P5 x ! 1 n  n 2:
2 3·4! 5 ·6! 1 : remaining terms

1 1 37
(d) g 1 1 1 : estimate
2 3·4! 72 2:
1 : explanation
Since the Taylor series for g about x ! 0 evaluated at x ! 1 is
alternating and the terms decrease in absolute value to 0, we know
37 1 1
g1 n .
72 5 ·6! 6!

8LI'SPPIKI&SEVH
:MWMXXLI'SPPIKI&SEVHSRXLI;IF[[[GSPPIKIFSEVHGSQ
%4’'%0'9097&'
7'36-2++9-()0-2)7 *SVQ& 

5YIWXMSR

-RXLIJMKYVIEFSZI R MWXLIWLEHIHVIKMSRMRXLIJMVWXUYEHVERXFSYRHIHF]XLI
KVETLSJ y ! PR  n x  XLILSVM^SRXEPPMRI y !  ERHXLIZIVXMGEPPMRI x ! 
E  *MRHXLIEVIESJ R
F  *MRHXLIZSPYQISJXLIWSPMHKIRIVEXIH[LIR R MWVIZSPZIHEFSYXXLI
LSVM^SRXEPPMRI y ! 
G  8LIVIKMSR R MWXLIFEWISJEWSPMH*SVXLMWWSPMHIEGLGVSWWWIGXMSR
TIVTIRHMGYPEVXSXLIxE\MWMWEWUYEVI*MRHXLIZSPYQISJXLIWSPMH

 'SVVIGXPMQMXWMRERMRXIKVEPMR E  F 
SV G 

 MRXIKVERH
E  n PR  n x dx !  SV 
 ERW[IV

  
F  n PR  n x n n dx MRXIKVERH
 
ERW[IV
!  SV 

  MRXIKVERH
G  n PR  n x dx !  SV 
 ERW[IV

8LI'SPPIKI&SEVH
:MWMXXLI'SPPIKI&SEVHSRXLI;IF[[[GSPPIKIFSEVHGSQ
%4’'%0'9097&'
7'36-2++9-()0-2)7 *SVQ& 

5YIWXMSR

8LIZIPSGMX]ZIGXSVSJETEVXMGPIQSZMRKMRXLITPERILEWGSQTSRIRXWKMZIRF]
dx dy
!  GSW t  WMR et  ERH !   WMR t   JSV  t 
dt dt
%XXMQI t !  XLITSWMXMSRSJXLITEVXMGPIMW n  
E  *SV  t  JMRHEPPZEPYIWSJ t EX[LMGLXLIPMRIXERKIRXXSXLITEXLSJXLITEVXMGPIMWZIVXMGEP
F  ;VMXIERIUYEXMSRJSVXLIPMRIXERKIRXXSXLITEXLSJXLITEVXMGPIEX t ! 
G  *MRHXLIWTIIHSJXLITEVXMGPIEX t ! 
H  *MRHXLIEGGIPIVEXMSRZIGXSVSJXLITEVXMGPIEX t ! 

dx
E  8LIXERKIRXPMRIMWZIVXMGEP[LIR x t !  ERH y t  WIXW !
 dy
 3R  t  XLMWLETTIRWEX t !  ERH t !  ERW[IV
SV

dy y  dy
F ! !  
dx t ! x  dx t !


 x 
x  ! n  x t dt !  y 

IUYEXMSR

y  ! y t dt ! 


 8LIPMRIXERKIRXXSXLITEXLSJXLITEVXMGPIEX t ! LEW
IUYEXMSR y !    x n  

 
G  7TIIH ! x   y  !  ERW[IV

H  %GGIPIVEXMSRZIGXSV x   y  ! n  x 



y 

8LI'SPPIKI&SEVH
:MWMXXLI'SPPIKI&SEVHSRXLI;IF[[[GSPPIKIFSEVHGSQ
%4’'%0'9097&'
7'36-2++9-()0-2)7 *SVQ& 

5YIWXMSR

t       
P t        

8LIJMKYVIEFSZIWLS[WEREFSZIKVSYRHW[MQQMRKTSSPMRXLIWLETISJEG]PMRHIV[MXLEVEHMYWSJJIIXERHE
LIMKLXSJJIIX8LITSSPGSRXEMRWGYFMGJIIXSJ[EXIVEXXMQI t !  (YVMRKXLIXMQIMRXIVZEP  t 
LSYVW[EXIVMWTYQTIHMRXSXLITSSPEXXLIVEXI P t GYFMGJIIXTIVLSYV8LIXEFPIEFSZIKMZIWZEPYIWSJ P t
JSVWIPIGXIHZEPYIWSJ t(YVMRKXLIWEQIXMQIMRXIVZEP[EXIVMWPIEOMRKJVSQXLITSSPEXXLIVEXI R t GYFMGJIIX
TIVLSYV[LIVI R t ! en t   2SXI8LIZSPYQI V SJEG]PMRHIV[MXLVEHMYW r ERHLIMKLX h MWKMZIRF]
V ! r  h
E  9WIEQMHTSMRX6MIQERRWYQ[MXLXLVIIWYFMRXIVZEPWSJIUYEPPIRKXLXSETTVS\MQEXIXLIXSXEPEQSYRXSJ[EXIV
XLEX[EWTYQTIHMRXSXLITSSPHYVMRKXLIXMQIMRXIVZEP  t  LSYVW7LS[XLIGSQTYXEXMSRWXLEXPIEHXS
]SYVERW[IV
F  'EPGYPEXIXLIXSXEPEQSYRXSJ[EXIVXLEXPIEOIHSYXSJXLITSSPHYVMRKXLIXMQIMRXIVZEP  t  LSYVW
G  9WIXLIVIWYPXWJVSQTEVXW E ERH F XSETTVS\MQEXIXLIZSPYQISJ[EXIVMRXLITSSPEXXMQI t !  LSYVW
6SYRH]SYVERW[IVXSXLIRIEVIWXGYFMGJSSX
H  *MRHXLIVEXIEX[LMGLXLIZSPYQISJ[EXIVMRXLITSSPMWMRGVIEWMRKEXXMQI t !  LSYVW,S[JEWXMWXLI[EXIV
PIZIPMRXLITSSPVMWMRKEX t !  LSYVW#-RHMGEXIYRMXWSJQIEWYVIMRFSXLERW[IVW

 QMHTSMRXWYQ
E P t dt        JX  
 ERW[IV

 MRXIKVEP
F R t dt !  JX  
 ERW[IV

 
G   P t dt n R t dt !  ERW[IV
 

%XXMQI t !  LSYVWXLIZSPYQISJ[EXIVMRXLITSSPMW
ETTVS\MQEXIP]  JX  

H V t !P t nRt V 
V  ! P  n R  !  n en ! SV JX  LV dV dh
  IUYEXMSRVIPEXMRK ERH
dt dt
 dh
V !   h  
dt t !
dV dh
! 
dt dt YRMXWSJ JX  LV ERH JX LV
dh  dV
 SV  JX LV
dt t !  dt t !

8LI'SPPIKI&SEVH
:MWMXXLI'SPPIKI&SEVHSRXLI;IF[[[GSPPIKIFSEVHGSQ
%4’'%0'9097&'
7'36-2++9-()0-2)7 *SVQ& 

5YIWXMSR

%WUYMVVIPWXEVXWEXFYMPHMRK A EXXMQI t !  ERHXVEZIPWEPSRKE


WXVEMKLX[MVIGSRRIGXIHXSFYMPHMRK B*SV  t  XLI
WUYMVVIPvWZIPSGMX]MWQSHIPIHF]XLITMIGI[MWIPMRIEVJYRGXMSR
HIJMRIHF]XLIKVETLEFSZI
E  %X[LEXXMQIWMRXLIMRXIVZEP  t  MJER]HSIWXLI
WUYMVVIPGLERKIHMVIGXMSR#+MZIEVIEWSRJSV]SYVERW[IV
F  %X[LEXXMQIMRXLIMRXIVZEP  t  MWXLIWUYMVVIP
JEVXLIWXJVSQFYMPHMRK A#,S[JEVJVSQFYMPHMRK A MWXLI
WUYMVVIPEXXLMWXMQI#
G  *MRHXLIXSXEPHMWXERGIXLIWUYMVVIPXVEZIPWHYVMRKXLIXMQIMRXIVZEP  t 
H  ;VMXII\TVIWWMSRWJSVXLIWUYMVVIPvWEGGIPIVEXMSR a t  ZIPSGMX] v t  ERHHMWXERGI x t JVSQFYMPHMRK A XLEX
EVIZEPMHJSVXLIXMQIMRXIVZEP  t 

E  8LIWUYMVVIPGLERKIWHMVIGXMSR[LIRIZIVMXWZIPSGMX]GLERKIWWMKR  t ZEPYIW



8LMWSGGYVWEX t !  ERH t !  I\TPEREXMSR

F  :IPSGMX]MWEX t !   t !  ERH t !  MHIRXMJMIWGERHMHEXIW



ERW[IVW
t TSWMXMSREXXMQIt
 

  


   

 
   


8LIWUYMVVIPMWJEVXLIWXJVSQFYMPHMRK A EXXMQI t ! 
MXWKVIEXIWXHMWXERGIJVSQXLIFYMPHMRKMW


G  8LIXSXEPHMWXERGIXVEZIPIHMW v t dt !      !  ERW[IV


 n n
H  *SV  t  a t ! ! n  a t
 n 
  v t 
v t !  n  t n  ! nt  
 x t

x   

t
x t ! x   nu   du

u !t
!   nu   u
u !

! nt  t n 

8LI'SPPIKI&SEVH
:MWMXXLI'SPPIKI&SEVHSRXLI;IF[[[GSPPIKIFSEVHGSQ
%4’'%0'9097&'
7'36-2++9-()0-2)7 *SVQ& 

5YIWXMSR

 x
0IX f ERH g FIXLIJYRGXMSRWHIJMRIHF] f x ! ERH g x !  JSVEPP x " 
x    x
E  *MRHXLIEFWSPYXIQE\MQYQZEPYISJ g SRXLISTIRMRXIVZEP  MJXLIQE\MQYQI\MWXW*MRHXLI
EFWSPYXIQMRMQYQZEPYISJ g SRXLISTIRMRXIVZEP  MJXLIQMRMQYQI\MWXW.YWXMJ]]SYVERW[IVW
F  *MRHXLIEVIESJXLIYRFSYRHIHVIKMSRMRXLIJMVWXUYEHVERXXSXLIVMKLXSJXLIZIVXMGEPPMRI x !  FIPS[
XLIKVETLSJ fERHEFSZIXLIKVETLSJ g

    x n  x x   n  x
E g x ! 
! 
g x
   x    x GVMXMGEPTSMRX

 ERW[IVW
*SV x "  g x !  JSV x ! 
 NYWXMJMGEXMSR

g x "  JSV  x


g x  JSV x "


g !


 8LIVIJSVI g LEWEQE\MQYQZEPYISJEX x !  ERH

g LEWRSQMRMQYQZEPYISRXLISTIRMRXIVZEP  

b
F f x ng x dx ! PMQ f x ng x dx MRXIKVEP
 b 
x!b  ERXMHMJJIVIRXMEXMSR

! PMQ PR x n PR    x  ERW[IV
b  x !

 
! PMQ PR b n PR   b   PR 
b  

b 
! PMQ PR
b   b 

b 
! PMQ PR
b   b 

 b 
! PMQ PR
b   b 
 
! PR
 

8LI'SPPIKI&SEVH
:MWMXXLI'SPPIKI&SEVHSRXLI;IF[[[GSPPIKIFSEVHGSQ
%4’'%0'9097&'
7'36-2++9-()0-2)7 *SVQ& 

5YIWXMSR
n n  x n
8LI1EGPEYVMRWIVMIWJSVXLIJYRGXMSR f MWKMZIRF] f x ! SRMXWMRXIVZEPSJGSRZIVKIRGI
n!
n n
E  *MRHXLIMRXIVZEPSJGSRZIVKIRGIJSVXLI1EGPEYVMRWIVMIWSJ f.YWXMJ]]SYVERW[IV
 x
F  7LS[XLEX y ! f x MWEWSPYXMSRXSXLIHMJJIVIRXMEPIUYEXMSR x y n y ! JSV x R [LIVI R MWXLI
  x
VEHMYWSJGSRZIVKIRGIJVSQTEVX E 

 x n 
n  n n n n n
E PMQ PMQ  x PMQ  x x WIXWYTVEXMS
n x
n n n n n
PMQMXIZEPYEXMSR
n n
VEHMYWSJGSRZIVKIRGI
 
x  JSV x GSRWMHIVWFSXLIRHTSMRXW

EREP]WMWERHMRXIVZEPSJ

8LIVIJSVIXLIVEHMYWSJGSRZIVKIRGIMW  GSRZIVKIRGI


 n n n n

 ;LIR x ! n  XLIWIVMIWMW ! 
 n!
n n n!
n n
8LMWMWXLILEVQSRMGWIVMIW[LMGLHMZIVKIW

 n n n n n
 ;LIR x !  XLIWIVMIWMW ! 
 n!
n n n!
n n
8LMWMWXLIEPXIVREXMRKLEVQSRMGWIVMIW[LMGLGSRZIVKIW
 
 8LIMRXIVZEPSJGSRZIVKIRGIJSVXLI1EGPEYVMRWIVMIWSJ f MW n  
 

x 
x 
x 
n n  x n
F y! n  n   WIVMIWJSV y
   n n
WIVMIWJSV xy
  n n  x n 
!  x  n  x  x n   WIVMIWJSV xy n y
 n n
EREP]WMW[MXLKISQIXVMGWIVMIW
   n n  x n n 
y !  x n  x   x n  
 n n
  n n n  x n
xy !  x  n  x  x n  
 n n
xy n y !  x  n  x   x  n  n n  x n 
n nn
!  x  n  x   x n  n x 

n nn n
8LIWIVMIW  n  x   x  n  n x  ! n x MWE
n!
 
KISQIXVMGWIVMIWXLEXGSRZIVKIWXS JSV x  8LIVIJSVI
  x 
 
xy y  x JSV x 
  x 

8LI'SPPIKI&SEVH
:MWMXXLI'SPPIKI&SEVHSRXLI;IF[[[GSPPIKIFSEVHGSQ
%4’'%0'9097&'
7'36-2++9-()0-2)7

5YIWXMSR

At time t, a particle moving in the xy-plane is at position x t , y t , where x t and y t are not explicitly
dx dy
given. For t 0, 4t 1 and sin t 2 . At time t 0, x 0 0 and y 0 4.
dt dt
(a) Find the speed of the particle at time t 3, and find the acceleration vector of the particle at time t 3.
(b) Find the slope of the line tangent to the path of the particle at time t 3.
(c) Find the position of the particle at time t 3.
(d) Find the total distance traveled by the particle over the time interval 0 t 3.

2 2
(a) Speed x 3 y 3 13.006 or 13.007 1 : speed
2:
1 : acceleration
Acceleration x 3 ,y 3
4, 5.466 or 4, 5.467

y 3
(b) Slope 0.031 or 0.032 1 : answer
x 3

3
dx
(c) x 3 0 dt 21 2 : x-coordinate
0 dt
1 : integral
3 1 : answer
dy
y 3 4 dt 3.226 4:
0 dt 2 : y -coordinate
1 : integral
At time t 3, the particle is at position 21, 3.226 .
1 : answer

3 2 2
dx dy 1 : integral
(d) Distance dt 21.091 2:
0
dt dt 1 : answer

8LI'SPPIKI&SEVH
:MWMXXLI'SPPIKI&SEVHSRXLI;IF[[[GSPPIKIFSEVHSVK
%4’'%0'9097&'
7'36-2++9-()0-2)7

5YIWXMSR

t
0 2 5 9 10
(minutes)
H t
66 60 52 44 43
(degrees Celsius)

As a pot of tea cools, the temperature of the tea is modeled by a differentiable function H for 0 t 10, where
time t is measured in minutes and temperature H t is measured in degrees Celsius. Values of H t at selected
values of time t are shown in the table above.
(a) Use the data in the table to approximate the rate at which the temperature of the tea is changing at time
t 3.5. Show the computations that lead to your answer.
1 10
(b) Using correct units, explain the meaning of H t dt in the context of this problem. Use a trapezoidal
10 0
1 10
sum with the four subintervals indicated by the table to estimate H t dt.
10 0
10
(c) Evaluate H t dt . Using correct units, explain the meaning of the expression in the context of this
0
problem.
(d) At time t 0, biscuits with temperature 100 C were removed from an oven. The temperature of the
biscuits at time t is modeled by a differentiable function B for which it is known that
B t 13.84e 0.173t . Using the given models, at time t 10, how much cooler are the biscuits than
the tea?

H 5 H 2
(a) H 3.5 1 : answer
5 2
52 60
2.666 or 2.667 degrees Celsius per minute
3

1 10
(b) H t dt is the average temperature of the tea, in degrees Celsius, 1 : meaning of expression
10 0
over the 10 minutes. 3: 1 : trapezoidal sum
1 : estimate
1 10 1 66 60 60 52 52 44 44 43
H t dt 2 3 4 1
10 0 10 2 2 2 2
52.95

10
(c) H t dt H 10 H 0 43 66 23 1 : value of integral
0 2:
The temperature of the tea drops 23 degrees Celsius from time t 0 to 1 : meaning of expression
time t 10 minutes.

10
(d) B 10 100 B t dt 34.18275; H 10 B 10 8.817 1 : integrand
0
The biscuits are 8.817 degrees Celsius cooler than the tea. 3: 1 : uses B 0 100
1 : answer

8LI'SPPIKI&SEVH
:MWMXXLI'SPPIKI&SEVHSRXLI;IF[[[GSPPIKIFSEVHSVK
%4’'%0'9097&'
7'36-2++9-()0-2)7

5YIWXMSR

Let f x e 2 x . Let R be the region in the first quadrant bounded by the


graph of f, the coordinate axes, and the vertical line x k , where
k 0. The region R is shown in the figure above.
(a) Write, but do not evaluate, an expression involving an integral that
gives the perimeter of R in terms of k.
(b) The region R is rotated about the x-axis to form a solid. Find the
volume, V, of the solid in terms of k.
dk 1
(c) The volume V, found in part (b), changes as k changes. If ,
dt 3
dV 1
determine when k .
dt 2

(a) f x 2e 2 x 1: f x
k 2 3: 1 : integral
Perimeter 1 k e2 k 1 2e 2 x dx
0 1 : answer

1 : integrand
k 2 k 4x x k 1 : limits
(b) Volume e2 x dx e dx e4 x e4k 4:
0 0 4 x 0 4 4 1 : antiderivative
1 : answer

dV dk
(c) e4k 1 : applies chain rule
dt dt 2:
1 : answer
1 dV 1
When k , e2 .
2 dt 3

8LI'SPPIKI&SEVH
:MWMXXLI'SPPIKI&SEVHSRXLI;IF[[[GSPPIKIFSEVHSVK
%4’'%0'9097&'
7'36-2++9-()0-2)7

5YIWXMSR

The continuous function f is defined on the interval 4 x 3.


The graph of f consists of two quarter circles and one line
segment, as shown in the figure above.
x
Let g x 2x f t dt.
0

(a) Find g 3 . Find g x and evaluate g 3.


(b) Determine the x-coordinate of the point at which g has an
absolute maximum on the interval 4 x 3.
Justify your answer.
(c) Find all values of x on the interval 4 x 3 for which
the graph of g has a point of inflection. Give a reason for
your answer.
(d) Find the average rate of change of f on the interval
4 x 3. There is no point c, 4 c 3, for which f c is equal to that average rate of change.
Explain why this statement does not contradict the Mean Value Theorem.

3 9
(a) g 3 2 3 f t dt 6 1: g 3
0 4
g x 2 f x 3: 1: g x
1: g 3
g 3 2 f 3 2

5
(b) g x 0 when f x 2. This occurs at x . 1 : considers g x 0
2
5 5 3: 1 : identifies interior candidate
g x 0 for 4 x and g x 0 for x 3. 1 : answer with justification
2 2
5
Therefore g has an absolute maximum at x .
2

(c) g x f x changes sign only at x 0. Thus the graph 1 : answer with reason
of g has a point of inflection at x 0.

(d) The average rate of change of f on the interval 4 x 3 is 1 : average rate of change
2:
f 3 f ( 4) 2 1 : explanation
.
3 ( 4) 7
To apply the Mean Value Theorem, f must be differentiable
at each point in the interval 4 x 3. However, f is not
differentiable at x 3 and x 0.

8LI'SPPIKI&SEVH
:MWMXXLI'SPPIKI&SEVHSRXLI;IF[[[GSPPIKIFSEVHSVK
%4’'%0'9097&'
7'36-2++9-()0-2)7

5YIWXMSR

At the beginning of 2010, a landfill contained 1400 tons of solid waste. The increasing function W models
the total amount of solid waste stored at the landfill. Planners estimate that W will satisfy the differential
dW 1
equation W 300 for the next 20 years. W is measured in tons, and t is measured in years from
dt 25
the start of 2010.
(a) Use the line tangent to the graph of W at t 0 to approximate the amount of solid waste that the landfill
1
contains at the end of the first 3 months of 2010 (time t ).
4
d 2W d 2W
(b) Find 2
in terms of W. Use to determine whether your answer in part (a) is an underestimate or
dt dt 2
1
an overestimate of the amount of solid waste that the landfill contains at time t .
4
dW 1
(c) Find the particular solution W W t to the differential equation W 300 with initial
dt 25
condition W 0 1400.

dW 1 1 dW
(a) W 0 300 1400 300 44 1: at t 0
dt t 0 25 25 2: dt
The tangent line is y 1400 44t. 1 : answer
1 1
W 1400 44 1411 tons
4 4

d 2W 1 dW 1 d 2W
(b) W 300 and W 1400 1:
dt 2 25 dt 625 2: dt 2
d W 2
1 1 : answer with reason
Therefore 0 on the interval 0 t .
dt 2 4
The answer in part (a) is an underestimate.

dW 1
(c) W 300 1 : separation of variables
dt 25
1 : antiderivatives
1 1
dW dt 5: 1 : constant of integration
W 300 25
1 1 : uses initial condition
ln W 300 t C
25 1 : solves for W
1
ln 1400 300 0 C ln 1100 C
25 Note: max 2 5 [1-1-0-0-0] if no constant of
1
t integration
W 300 1100e 25
Note: 0 5 if no separation of variables
1
t
W t 300 1100e 25 , 0 t 20

8LI'SPPIKI&SEVH
:MWMXXLI'SPPIKI&SEVHSRXLI;IF[[[GSPPIKIFSEVHSVK
%4’'%0'9097&'
7'36-2++9-()0-2)7

5YIWXMSR

5
Let f x sin x 2 cos x. The graph of y f x is
shown above.
(a) Write the first four nonzero terms of the Taylor series for
sin x about x 0, and write the first four nonzero terms
of the Taylor series for sin x 2 about x 0.

(b) Write the first four nonzero terms of the Taylor series for
cos x about x 0. Use this series and the series for
sin x 2 , found in part (a), to write the first four nonzero
terms of the Taylor series for f about x 0.
6
(c) Find the value of f 0 .
(d) Let P4 x be the fourth-degree Taylor polynomial for f about x 0. Using information from the graph of
1 1 1
y f 5 x shown above, show that P4 f .
4 4 3000

x3 x5 x7 1 : series for sin x


(a) sin x x
3! 5! 7! 3:
x6 x10 x14 2 : series for sin x 2
sin x 2 x2
3! 5! 7!

x2 x4 x6
(b) cos x 1 1 : series for cos x
2! 4! 6! 3:
2 : series for f x
x2 x4 121x6
f x 1
2 4! 6!

6
f 0
(c) is the coefficient of x6 in the Taylor series for f about 1 : answer
6!
x 0. Therefore f (6) 0 121.

5
(d) The graph of y f x indicates that max f (5) x 40. 1 : form of the error bound
0 x 1 2:
4 1 : analysis
Therefore
max f (5) x
0 x 1 5
1 1 4 1 40 1 1
P4 f .
4 4 5! 4 120 45 3072 3000

8LI'SPPIKI&SEVH
:MWMXXLI'SPPIKI&SEVHSRXLI;IF[[[GSPPIKIFSEVHSVK
%4’'%0'9097&'
7'36-2++9-()0-2)7 *SVQ& 

5YIWXMSR

A cylindrical can of radius 10 millimeters is used to measure rainfall in Stormville. The can is initially empty,
and rain enters the can during a 60-day period. The height of water in the can is modeled by the function S,
where S t is measured in millimeters and t is measured in days for 0 t 60. The rate at which the height
of the water is rising in the can is given by S t 2sin 0.03t 1.5.
(a) According to the model, what is the height of the water in the can at the end of the 60-day period?
(b) According to the model, what is the average rate of change in the height of water in the can over the
60-day period? Show the computations that lead to your answer. Indicate units of measure.
(c) Assuming no evaporation occurs, at what rate is the volume of water in the can changing at time t 7?
Indicate units of measure.
(d) During the same 60-day period, rain on Monsoon Mountain accumulates in a can identical to the one in
Stormville. The height of the water in the can on Monsoon Mountain is modeled by the function M, where
1
M t 3t 3 30t 2 330t . The height M t is measured in millimeters, and t is measured in days
400
for 0 t 60. Let D t M t S t . Apply the Intermediate Value Theorem to the function D on
the interval 0 t 60 to justify that there exists a time t, 0 t 60, at which the heights of water in the
two cans are changing at the same rate.

1 : limits
60
(a) S 60 S t dt 171.813 mm 3: 1 : integrand
0
1 : answer

S 60 S 0
(b) 2.863 or 2.864 mm day 1 : answer
60

(c) V t 100 S t 1 : relationship between V and S


2:
V 7 100 S 7 602.218 1 : answer

The volume of water in the can is increasing at a rate of


602.218 mm3 day.

(d) D 0 0.675 0 and D 60 69.37730 0 1 : considers D 0 and D 60


2:
1 : justification
Because D is continuous, the Intermediate Value Theorem
implies that there is a time t, 0 t 60, at which D t 0.
At this time, the heights of water in the two cans are changing
at the same rate.

1 : units in (b) or (c)

8LI'SPPIKI&SEVH
:MWMXXLI'SPPIKI&SEVHSRXLI;IF[[[GSPPIKIFSEVHSVK
%4’'%0'9097&'
7'36-2++9-()0-2)7 *SVQ& 

5YIWXMSR

The polar curve r is given by r 3 sin , where 0 2 .


(a) Find the area in the second quadrant enclosed by the coordinate axes and the graph of r.

(b) For , there is one point P on the polar curve r with x-coordinate 3. Find the angle that
2
corresponds to point P. Find the y-coordinate of point P. Show the work that leads to your answers.
(c) A particle is traveling along the polar curve r so that its position at time t is x t , y t and such that
d dy 2
2. Find at the instant that , and interpret the meaning of your answer in the context of
dt dt 3
the problem.

1 : integrand
1 2
(a) Area r d 47.513 3: 1 : limits and constant
2 2
1 : answer

(b) 3 r cos 3 sin cos 1 : equation


2.01692 3: 1 : value of
y r sin 6.272 1 : y -coordinate

(c) y r sin 3 sin sin 1 : uses chain rule


dy dy d 3: 1 : answer
2.819
dt 2 3 d dt 2 3 1 : interpretation

The y-coordinate of the particle is decreasing at a rate of 2.819.

8LI'SPPIKI&SEVH
:MWMXXLI'SPPIKI&SEVHSRXLI;IF[[[GSPPIKIFSEVHSVK
%4’'%0'9097&'
7'36-2++9-()0-2)7 *SVQ& 

5YIWXMSR

The functions f and g are given by f x x and g x 6 x.


Let R be the region bounded by the x-axis and the graphs of f and
g, as shown in the figure above.
(a) Find the area of R.
(b) The region R is the base of a solid. For each y, where
0 y 2, the cross section of the solid taken perpendicular to
the y-axis is a rectangle whose base lies in R and whose height is 2y. Write, but do not evaluate, an integral
expression that gives the volume of the solid.
(c) There is a point P on the graph of f at which the line tangent to the graph of f is perpendicular to the graph
of g. Find the coordinates of point P.

x 4 1 : integral
4 1 2 3 2 22
(a) Area x dx 2 2 x 2 3: 1 : antiderivative
0 2 3 x 0 3
1 : answer

(b) y x x y2 2 : integrand
3:
y 6 x x 6 y 1 : answer

Width 6 y y2

2
Volume 2y 6 y y 2 dy
0

(c) g x 1 1: f x
3: 1 : equation
Thus a line perpendicular to the graph of g has slope 1.
1 : answer
1
f x
2 x
1 1
1 x
2 x 4
1 1
The point P has coordinates , .
4 2

8LI'SPPIKI&SEVH
:MWMXXLI'SPPIKI&SEVHSRXLI;IF[[[GSPPIKIFSEVHSVK
%4’'%0'9097&'
7'36-2++9-()0-2)7 *SVQ& 

5YIWXMSR

The graph of the differentiable function y f x with domain


0 x 10 is shown in the figure above. The area of the region enclosed
between the graph of f and the x-axis for 0 x 5 is 10, and the area of
the region enclosed between the graph of f and the x-axis for 5 x 10
is 27. The arc length for the portion of the graph of f between x 0 and
x 5 is 11, and the arc length for the portion of the graph
of f between x 5 and x 10 is 18. The function f has exactly two
critical points that are located at x 3 and x 8.
(a) Find the average value of f on the interval 0 x 5.
10
(b) Evaluate 3f x 2 dx. Show the computations that lead to
0
your answer.
x
(c) Let g x f t dt. On what intervals, if any, is the graph of g both concave up and decreasing? Explain
5
your reasoning.
x x
(d) The function h is defined by h x 2f . The derivative of h is h x f . Find the arc length of
2 2
the graph of y h x from x 0 to x 20.

1 5 10
(a) Average value f x dx 2 1 : answer
5 0 5

10 5 10
(b) 3f x 2 dx 3 f x dx f x dx 20 2 : answer
0 0 5
3 10 27 20 71

(c) g x f x 1: g x f x
g x 0 on 0 x 5 3: 1 : analysis
g x is increasing on 3 x 8. 1 : answer and reason
The graph of g is concave up and decreasing on 3 x 5.

20 2
20 2 x
(d) Arc length 1 h x dx 1 f dx 1 : integral
0
0
2
3: 1 : substitution
x 1 1 : answer
Let u . Then du dx and
2 2
20 2
x 10 2
1 f dx 2 1 f u du 2 11 18 58
0
2 0

8LI'SPPIKI&SEVH
:MWMXXLI'SPPIKI&SEVHSRXLI;IF[[[GSPPIKIFSEVHSVK
%4’'%0'9097&'
7'36-2++9-()0-2)7 *SVQ& 

5YIWXMSR

t
0 10 40 60
(seconds)
B t
100 136 9 49
(meters)
vt
2.0 2.3 2.5 4.6
(meters per second)

Ben rides a unicycle back and forth along a straight east-west track. The twice-differentiable function B models
Ben’s position on the track, measured in meters from the western end of the track, at time t, measured in seconds
from the start of the ride. The table above gives values for B t and Ben’s velocity, v t , measured in meters
per second, at selected times t.
(a) Use the data in the table to approximate Ben’s acceleration at time t 5 seconds. Indicate units of measure.
60
(b) Using correct units, interpret the meaning of v t dt in the context of this problem. Approximate
0
60
v t dt using a left Riemann sum with the subintervals indicated by the data in the table.
0

(c) For 40 t 60, must there be a time t when Ben’s velocity is 2 meters per second? Justify your answer.
(d) A light is directly above the western end of the track. Ben rides so that at time t, the distance L t between
2 2
Ben and the light satisfies L t 122 B t . At what rate is the distance between Ben and the light
changing at time t 40 ?

v 10 v 0 0.3
(a) a 5 0.03 meters sec2 1 : answer
10 0 10

60
(b) v t dt is the total distance, in meters, Ben rides over the 1 : meaning of integral
0 2:
60-second interval t 0 to t 60. 1 : approximation

60
v t dt 2.0 10 2.3 40 10 2.5 60 40 139 meters
0

B 60 B 40 49 9
(c) Because 2, the Mean Value Theorem 1 : difference quotient
60 40 20 2:
1 : conclusion with justification
implies there is a time t, such that v t 2.

(d) 2 L t L t 2B t B t 1 : derivatives
B 40 v 40 9 2.5 3 3: 1 : uses B t v t
L 40 meters sec
L 40 144 81 2 1 : answer

1 : units in (a) or (b)

8LI'SPPIKI&SEVH
:MWMXXLI'SPPIKI&SEVHSRXLI;IF[[[GSPPIKIFSEVHSVK
%4’'%0'9097&'
7'36-2++9-()0-2)7 *SVQ& 

5YIWXMSR

Let f x ln 1 x3 .

x 2 x3 x 4 xn
(a) The Maclaurin series for ln 1 x is x 1n 1 . Use the series to write
2 3 4 n
the first four nonzero terms and the general term of the Maclaurin series for f.
(b) The radius of convergence of the Maclaurin series for f is 1. Determine the interval of convergence. Show
the work that leads to your answer.
x
(c) Write the first four nonzero terms of the Maclaurin series for f t 2 . If g x f t 2 dt , use the first
0
two nonzero terms of the Maclaurin series for g to approximate g 1 .
(d) The Maclaurin series for g, evaluated at x 1, is a convergent alternating series with individual terms
that decrease in absolute value to 0. Show that your approximation in part (c) must differ from g 1 by
1
less than .
5

x6 x9 x12 n 1 x3 n 1 : first four terms


(a) x3 1 2:
2 3 4 n 1 : general term

(b) The interval of convergence is centered at x 0. 2 : answer with analysis


1 1 1 1
At x 1, the series is 1 , which
2 3 4 n
diverges because the harmonic series diverges.
1 1 1 1
At x 1, the series is 1 1n 1
, the
2 3 4 n
alternating harmonic series, which converges.
Therefore the interval of convergence is 1 x 1.

(c) The Maclaurin series for f x , f t 2 , and g x are 1 : two terms for f t 2

n 1 4: 1 : other terms for f t 2


f x : 1 3 x 3n 1
3x2 3 x5 3 x8 3 x11
n 1 1 : first two terms for g x
n 1 1 : approximation
f t2 : 1 3t 6 n 2
3t 4 3t10 3t16 3t 22
n 1

n 1 3x 6n 1 3x5 3 x11 3x17 3x 23


g x : 1
n 1
6n 1 5 11 17 23
3 3 18
Thus g 1 .
5 11 55

(d) The Maclaurin series for g evaluated at x 1 is alternating, and the 1 : analysis
terms decrease in absolute value to 0.
18 3 117 3 1
Thus g 1 .
55 17 17 5

8LI'SPPIKI&SEVH
:MWMXXLI'SPPIKI&SEVHSRXLI;IF[[[GSPPIKIFSEVHSVK
%4’'%0'9097&'
7'36-2++9-()0-2)7

5YIWXMSR

t (minutes) 0 4 9 15 20
W t (degrees Fahrenheit) 55.0 57.1 61.8 67.9 71.0

The temperature of water in a tub at time t is modeled by a strictly increasing, twice-differentiable function W,
where W t is measured in degrees Fahrenheit and t is measured in minutes. At time t ! 0, the temperature of
the water is 55”F. The water is heated for 30 minutes, beginning at time t ! 0. Values of W t at selected
times t for the first 20 minutes are given in the table above.
(a) Use the data in the table to estimate W 12 . Show the computations that lead to your answer. Using correct
units, interpret the meaning of your answer in the context of this problem.
20 20
(b) Use the data in the table to evaluate W t dt. Using correct units, interpret the meaning of W t dt
0 0
in the context of this problem.
1 20
(c) For 0 t 20, the average temperature of the water in the tub is W t dt . Use a left Riemann sum
20 0
1 20
with the four subintervals indicated by the data in the table to approximate W t dt . Does this
20 0
approximation overestimate or underestimate the average temperature of the water over these 20 minutes?
Explain your reasoning.
(d) For 20 t 25, the function W that models the water temperature has first derivative given by
W t ! 0.4 t cos 0.06t . Based on the model, what is the temperature of the water at time t ! 25 ?

W 15 n W 9 67.9 n 61.8
(a) W 12 1 : estimate
15 n 9 6 2:
! 1.017 (or 1.016) 1 : interpretation with units
The water temperature is increasing at a rate of approximately
1.017 ”F per minute at time t ! 12 minutes.
20
(b) W t dt ! W 20 n W 0 ! 71.0 n 55.0 ! 16 1 : value
0 2:
The water has warmed by 16 ”F over the interval from t ! 0 to 1 : interpretation with units
t ! 20 minutes.
1 20 1
(c) W t dt 4 W 0 5 W 4 6 W 9 5 W 15 1 : left Riemann sum
20 0 20
1 3: 1 : approximation
4 55.0 5 57.1 6 61.8 5 67.9
20 1 : underestimate with reason
1
1215.8 60.79
20
This approximation is an underestimate, because a left Riemann
sum is used and the function W is strictly increasing.
25
(d) W 25 ! 71.0  W t dt 1 : integral
20 2:
! 71.0  2.043155 ! 73.043 1 : answer

8LI'SPPIKI&SEVH
:MWMXXLI'SPPIKI&SEVHSRXLI;IF[[[GSPPIKIFSEVHSVK
%4’'%0'9097&'
7'36-2++9-()0-2)7

5YIWXMSR

For t 0, a particle is moving along a curve so that its position at time t is x t , y t . At time t ! 2, the
dx t2 dy
particle is at position 1, 5 . It is known that ! and ! sin 2 t.
dt e t dt
(a) Is the horizontal movement of the particle to the left or to the right at time t ! 2 ? Explain your answer.
Find the slope of the path of the particle at time t ! 2.
(b) Find the x-coordinate of the particle’s position at time t ! 4.
(c) Find the speed of the particle at time t ! 4. Find the acceleration vector of the particle at time t ! 4.
(d) Find the distance traveled by the particle from time t ! 2 to t ! 4.

(a) dx !
2
1 : moving to the right with reason
dt t !2 e2
dy dt
3: 1 : considers
dx dt
dx
Because " 0, the particle is moving to the right 1 : slope at t ! 2
dt t !2
at time t ! 2.

dy dy dt t !2
! ! 3.055 or 3.054
dx t !2 dx dt t !2

4
t2 1 : integral
(b) x 4 ! 1  dt ! 1.253 or 1.252 2:
2 et 1 : answer

2 2
(c) Speed ! x 4  y 4 ! 0.575 or 0.574 1 : speed
2:
1 : acceleration
Acceleration ! x 4 , y 4
! n 0.041, 0.989

4 2 2 1 : integral
(d) Distance ! x t  y t dt 2:
2 1 : answer
! 0.651 or 0.650

8LI'SPPIKI&SEVH
:MWMXXLI'SPPIKI&SEVHSRXLI;IF[[[GSPPIKIFSEVHSVK
%4’'%0'9097&'
7'36-2++9-()0-2)7

5YIWXMSR

Let f be the continuous function defined on n4, 3


whose graph, consisting of three line segments and a
semicircle centered at the origin, is given above. Let g
x
be the function given by g x ! f t dt.
1

(a) Find the values of g 2 and g n2 .

(b) For each of g n3 and g n3 , find the value or


state that it does not exist.
(c) Find the x-coordinate of each point at which the
graph of g has a horizontal tangent line. For each
of these points, determine whether g has a relative minimum, relative maximum, or neither a minimum nor
a maximum at the point. Justify your answers.
(d) For n 4 x 3, find all values of x for which the graph of g has a point of inflection. Explain your
reasoning.
2 1 1 1
(a) g 2 ! f t dt ! n 1 !n 1:g 2
1 2 2 4 2:
n2 1
1 : g n2
g n2 ! f t dt ! n f t dt
1 n2
3 3
!n n ! n
2 2 2 2

(b) g x ! f x g n3 ! f n 3 ! 2 1 : g n3
2:
g x ! f x g n3 ! f n3 ! 1 1 : g n3

(c) The graph of g has a horizontal tangent line where 1 : considers g x ! 0


g x ! f x ! 0. This occurs at x ! n1 and x ! 1. 3: 1 : x ! n1 and x ! 1
1 : answers with justifications
g x changes sign from positive to negative at x ! n1.
Therefore, g has a relative maximum at x ! n1.

g x does not change sign at x ! 1. Therefore, g has


neither a relative maximum nor a relative minimum at x ! 1.

(d) The graph of g has a point of inflection at each of 1 : answer


2:
x ! n2, x ! 0, and x ! 1 because g x ! f x changes 1 : explanation
sign at each of these values.

8LI'SPPIKI&SEVH
:MWMXXLI'SPPIKI&SEVHSRXLI;IF[[[GSPPIKIFSEVHSVK
%4’'%0'9097&'
7'36-2++9-()0-2)7

5YIWXMSR

x 1 1.1 1.2 1.3 1.4


J \ 8 10 12 13 14.5

The function f is twice differentiable for x " 0 with f 1 ! 15 and f 1 ! 20. Values of f , the derivative of
f, are given for selected values of x in the table above.
(a) Write an equation for the line tangent to the graph of f at x ! 1. Use this line to approximate f 1.4 .
(b) Use a midpoint Riemann sum with two subintervals of equal length and values from the table to
1.4 1.4
approximate f x dx. Use the approximation for f x dx to estimate the value of f 1.4 . Show
1 1
the computations that lead to your answer.
(c) Use Euler’s method, starting at x ! 1 with two steps of equal size, to approximate f 1.4 . Show the
computations that lead to your answer.
(d) Write the second-degree Taylor polynomial for f about x ! 1. Use the Taylor polynomial to approximate
f 1.4 .

(a) f 1 ! 15, f 1 ! 8 1 : tangent line


2:
1 : approximation
An equation for the tangent line is
y ! 15  8 x n 1 .

f 1.4 15 8 1.4 1 18.2

1.4
(b) f x dx 0.2 10 0.2 13 4.6 1 : midpoint Riemann sum
1
3: 1 : Fundamental Theorem of Calculus
1.4 1 : answer
f 1.4 ! f 1  f x dx
1

f 1.4 15 4.6 1 9. 6

(c) f 1.2 f 1 0.2 8 16.6 1 : Euler’s method with two steps


2:
1 : answer
f 1.4 16.6 0.2 12 19.0

(d) T2 x ! 15  8 x n 1  20 x n 1 2 1 : Taylor polynomial


2! 2:
1 : approximation
! 15  8 x n 1  10 x n 1 2

2
f 1.4 15 8 1.4 1 10 1.4 1 19.8

8LI'SPPIKI&SEVH
:MWMXXLI'SPPIKI&SEVHSRXLI;IF[[[GSPPIKIFSEVHSVK
%4’'%0'9097&'
7'36-2++9-()0-2)7

5YIWXMSR

The rate at which a baby bird gains weight is proportional to the difference between its adult weight and its
current weight. At time t ! 0, when the bird is first weighed, its weight is 20 grams. If B t is the weight of the
bird, in grams, at time t days after it is first weighed, then
dB 1
! 100 n B .
dt 5
Let y ! B t be the solution to the differential equation above with initial condition B 0 ! 20.
(a) Is the bird gaining weight faster when it weighs 40 grams or when it
weighs 70 grams? Explain your reasoning.
d 2B d 2B
(b) Find in terms of B. Use to explain why the graph of B
dt 2 dt 2
cannot resemble the following graph.
(c) Use separation of variables to find y ! B t , the particular solution to
the differential equation with initial condition B 0 ! 20.

dB 1 dB
(a) ! 60 ! 12 1 : uses
dt B ! 40 5 2: dt
1 : answer with reason
dB 1
! 30 ! 6
dt B ! 70 5

dB dB
Because " , the bird is gaining
dt B ! 40 dt B ! 70
weight faster when it weighs 40 grams.

d 2B 1 dB 1 1 1 d 2B
(b) 100 B 100 B 1: in terms of B
dt 2 5 dt 5 5 25 2: dt 2
Therefore, the graph of B is concave down for 1 : explanation
20 B 100. A portion of the given graph is
concave up.

dB 1
(c) ! 100 n B 1 : separation of variables
dt 5
1 1 1 : antiderivatives
dB ! dt 5: 1 : constant of integration
100 n B 5
1 1 : uses initial condition
n ln 100 n B ! t  C
5 1 : solves for B
Because 20 B 100, 100 n B ! 100 n B.
1 Note: max 2 5 [1-1-0-0-0] if no constant of
n ln 100 n 20 ! 0 C n ln 80 ! C
5 integration
100 n B ! 80e n t 5
B t 100 80e nt 5
, t 0 Note: 0 5 if no separation of variables

8LI'SPPIKI&SEVH
:MWMXXLI'SPPIKI&SEVHSRXLI;IF[[[GSPPIKIFSEVHSVK
%4’'%0'9097&'
7'36-2++9-()0-2)7

5YIWXMSR

The function g has derivatives of all orders, and the Maclaurin series for g is
n x 2 n 1 x x3 x5
n1 ! n  n .
n !0
2n  3 3 5 7

(a) Using the ratio test, determine the interval of convergence of the Maclaurin series for g.
1
(b) The Maclaurin series for g evaluated at x ! is an alternating series whose terms decrease in absolute
2
1 17
value to 0. The approximation for g using the first two nonzero terms of this series is . Show that
2 120
1 1
this approximation differs from g by less than .
2 200
(c) Write the first three nonzero terms and the general term of the Maclaurin series for g x .

x 2 n  3 2n  3 2n  3
(a) ! x2 1 : sets up ratio
2n  5 x 2 n 1 2n  5
1 : computes limit of ratio
1 : identifies interior of
2n  3 5:
lim x2 ! x2 interval of convergence
n 2n  5
1 : considers both endpoints
x2 1 n1 x 1 1 : analysis and interval of convergence
The series converges when n1 x 1.

1 1 1 1
When x ! n1, the series is n  n  n
3 5 7 9
This series converges by the Alternating Series Test.

1 1 1 1
When x ! 1, the series is n  n 
3 5 7 9
This series converges by the Alternating Series Test.

Therefore, the interval of convergence is 1 x 1.

1 5
1 17 2 1 1 1 : uses the third term as an error bound
(b) g n ! 2:
2 120 7 224 200 1 : error bound

1 3 2 5 4 n 2n  1 2 n 1 : first three terms


(c) g x ! n x  x   n1 x  2:
3 5 7 2n  3 1 : general term

8LI'SPPIKI&SEVH
:MWMXXLI'SPPIKI&SEVHSRXLI;IF[[[GSPPIKIFSEVHSVK
ßÐr ÝßÔÝËÔËÍ ÞÝ
îðïí ÍÝÑÎ×ÒÙ ÙË×ÜÛÔ×ÒÛÍ

Ï«»-¬·±² ï

On a certain workday, the rate, in tons per hour, at which unprocessed gravel arrives at a gravel processing plant
t2
is modeled by G t 90 45cos , where t is measured in hours and 0 t 8. At the beginning of the
18
workday t 0 , the plant has 500 tons of unprocessed gravel. During the hours of operation, 0 t 8, the
plant processes gravel at a constant rate of 100 tons per hour.

(a) Find G 5 . Using correct units, interpret your answer in the context of the problem.
(b) Find the total amount of unprocessed gravel that arrives at the plant during the hours of operation on this
workday.
(c) Is the amount of unprocessed gravel at the plant increasing or decreasing at time t 5 hours? Show the
work that leads to your answer.
(d) What is the maximum amount of unprocessed gravel at the plant during the hours of operation on this
workday? Justify your answer.

(a) G 5 24.588 (or 24.587) 1:G 5


2:
1 : interpretation with units
The rate at which gravel is arriving is decreasing by 24.588
(or 24.587) tons per hour per hour at time t 5 hours.

8 1 : integral
(b) G t dt 825.551 tons 2:
0 1 : answer

(c) G 5 98.140764 100 1 : compares G 5 to 100


2:
1 : conclusion
At time t 5, the rate at which unprocessed gravel is arriving
is less than the rate at which it is being processed.
Therefore, the amount of unprocessed gravel at the plant is
decreasing at time t 5.

(d) The amount of unprocessed gravel at time t is given by 1 : considers A t 0


t
A t 500 G s 100 ds. 3: 1 : answer
0
1 : justification
A t G t 100 0 t 4.923480

t At
0 500
4.92348 635.376123
8 525.551089

The maximum amount of unprocessed gravel at the plant during


this workday is 635.376 tons.

w îðïí ̸» ݱ´´»¹» Þ±¿®¼ò


Ê·-·¬ ¬¸» ݱ´´»¹» Þ±¿®¼ ±² ¬¸» É»¾æ ©©©ò½±´´»¹»¾±¿®¼ò±®¹ò
ßÐr ÝßÔÝËÔËÍ ÞÝ
îðïí ÍÝÑÎ×ÒÙ ÙË×ÜÛÔ×ÒÛÍ

Ï«»-¬·±² î

The graphs of the polar curves r 3 and r 4 2sin are shown in the figure
5
above. The curves intersect when and .
6 6
(a) Let S be the shaded region that is inside the graph of r 3 and also inside the
graph of r 4 2sin . Find the area of S.
(b) A particle moves along the polar curve r 4 2sin so that at time t
2
seconds, t . Find the time t in the interval 1 t 2 for which
the x-coordinate of the particle’s position is 1.
(c) For the particle described in part (b), find the position vector in terms of t. Find the velocity vector at
time t 1.5.

1 : integrand
1 5 6 2
(a) Area 6 4 2sin d 24.709 (or 24.708) 3: 1 : limits and constant
2 6
1 : answer

(b) x r cos x 4 2sin cos 1: x or x t


x t 4 2sin t 2
cos t 2 3: 1: x 1 or x t 1
x t 1 when t 1.428 (or 1.427) 1 : answer

(c) y r sin y 4 2sin sin 2 : position vector


3:
y t 4 2sin t 2
sin t 2 1 : velocity vector

Position vector x t ,y t
4 2 sin t 2 cos t 2 , 4 2 sin t 2 sin t 2

v 1.5 x 1.5 , y 1.5


8.072, 1.673 or 8.072, 1.672

w îðïí ̸» ݱ´´»¹» Þ±¿®¼ò


Ê·-·¬ ¬¸» ݱ´´»¹» Þ±¿®¼ ±² ¬¸» É»¾æ ©©©ò½±´´»¹»¾±¿®¼ò±®¹ò
ßÐr ÝßÔÝËÔËÍ ÞÝ
îðïí ÍÝÑÎ×ÒÙ ÙË×ÜÛÔ×ÒÛÍ

Ï«»-¬·±² í

t
0 1 2 3 4 5 6
(minutes)
C t
0 5.3 8.8 11.2 12.8 13.8 14.5
(ounces)
Hot water is dripping through a coffeemaker, filling a large cup with coffee. The amount of coffee in the cup at
time t, 0 t 6, is given by a differentiable function C, where t is measured in minutes. Selected values of
C t , measured in ounces, are given in the table above.
(a) Use the data in the table to approximate C 3.5 . Show the computations that lead to your answer, and
indicate units of measure.
(b) Is there a time t, 2 t 4, at which C t 2 ? Justify your answer.
(c) Use a midpoint sum with three subintervals of equal length indicated by the data in the table to approximate
1 6 1 6
the value of C t dt. Using correct units, explain the meaning of C t dt in the context of the
6 0 6 0
problem.
0.4 t
(d) The amount of coffee in the cup, in ounces, is modeled by B t 16 16e . Using this model, find the
rate at which the amount of coffee in the cup is changing when t 5.

C 4 C 3 12.8 11.2 1 : approximation


(a) C 3.5 1.6 ounces min 2:
4 3 1 1 : units

C 4 C 2
(b) C is differentiable C is continuous (on the closed interval) 1:
2: 4 2
C 4 C 2 12.8 8.8 1 : conclusion, using MVT
2
4 2 2
Therefore, by the Mean Value Theorem, there is at least
one time t, 2 t 4, for which C t 2.

(c) 1 1
6
C t dt 2 ·C 1 2 ·C 3 2 ·C 5 1 : midpoint sum
6 0 6
1 3: 1 : approximation
2 · 5.3 2 ·11.2 2 ·13.8 1 : interpretation
6
1
60.6 10.1 ounces
6
1 6
C t dt is the average amount of coffee in the cup, in
6 0
ounces, over the time interval 0 t 6 minutes.

0.4t 0.4 t
(d) B t 16 0.4 e 6.4e 1:B t
2:
1:B 5
0.4 5 6.4
B 5 6.4e ounces min
e2

w îðïí ̸» ݱ´´»¹» Þ±¿®¼ò


Ê·-·¬ ¬¸» ݱ´´»¹» Þ±¿®¼ ±² ¬¸» É»¾æ ©©©ò½±´´»¹»¾±¿®¼ò±®¹ò
ßÐr ÝßÔÝËÔËÍ ÞÝ
îðïí ÍÝÑÎ×ÒÙ ÙË×ÜÛÔ×ÒÛÍ

Ï«»-¬·±² ì

The figure above shows the graph of f , the derivative


of a twice-differentiable function f, on the closed interval
0 x 8. The graph of f has horizontal tangent lines
at x 1, x 3, and x 5. The areas of the regions
between the graph of f and the x-axis are labeled in the
figure. The function f is defined for all real numbers and
satisfies f 8 4.

(a) Find all values of x on the open interval 0 x 8


for which the function f has a local minimum.
Justify your answer.
(b) Determine the absolute minimum value of f on the
closed interval 0 x 8. Justify your answer.
(c) On what open intervals contained in 0 x 8 is the graph of f both concave down and increasing?
Explain your reasoning.
5
(d) The function g is defined by g x f x 3 . If f 3 , find the slope of the line tangent to the
2
graph of g at x 3.

(a) x 6 is the only critical point at which f changes sign from 1 : answer with justification
negative to positive. Therefore, f has a local minimum at
x 6.

(b) From part (a), the absolute minimum occurs either at x 6 or 1 : considers x 0 and x 6
at an endpoint. 3: 1 : answer
0
f 0 f 8 f x dx 1 : justification
8
8
f 8 f x dx 4 12 8
0
6
f 6 f 8 f x dx
8
8
f 8 f x dx 4 7 3
6
f 8 4

The absolute minimum value of f on the closed interval 0, 8


is 8.

(c) The graph of f is concave down and increasing on 0 x 1 1 : answer


2:
and 3 x 4, because f is decreasing and positive on these 1 : explanation
intervals.
2
(d) g x 3 f x ·f x 2: g x
3:
2 5 2 1 : answer
g 3 3 f 3 ·f 3 3 ·4 75
2

w îðïí ̸» ݱ´´»¹» Þ±¿®¼ò


Ê·-·¬ ¬¸» ݱ´´»¹» Þ±¿®¼ ±² ¬¸» É»¾æ ©©©ò½±´´»¹»¾±¿®¼ò±®¹ò
ßÐr ÝßÔÝËÔËÍ ÞÝ
îðïí ÍÝÑÎ×ÒÙ ÙË×ÜÛÔ×ÒÛÍ

Ï«»-¬·±² ë

dy
Consider the differential equation y2 2x 2 . Let y f x be the particular solution to the differential
dx
equation with initial condition f 0 1.

f x 1
(a) Find lim . Show the work that leads to your answer.
x 0 sin x

1
(b) Use Euler’s method, starting at x 0 with two steps of equal size, to approximate f .
2

(c) Find y f x , the particular solution to the differential equation with initial condition f 0 1.

(a) lim f x 1 1 1 0 and lim sin x 0 1 : L’Hospital’s Rule


x 0 x 0 2:
1 : answer
Using L’Hospital’s Rule,
f x 1 f x f 0 ( 1) 2 · 2
lim lim 2
x 0 sin x x 0 cos x cos 0 1

1 1
(b) f f 0 f 0 1 : Euler’s method
4 4 2:
1 : answer
1 1
1 2
4 2

1 1 1 1
f f f
2 4 4 4
2
1 1 1 1 11
2· 2
2 2 4 4 32

dy
(c) y2 2x 2 1 : separation of variables
dx
dy 1 : antiderivatives
2x 2 dx 5: 1 : constant of integration
y2
dy 1 : uses initial condition
2x 2 dx 1 : solves for y
y2
1
x2 2x C Note: max 2 5 [1-1-0-0-0] if no constant of
y
1 integration
02 2 ·0 C C 1
1
1 Note: 0 5 if no separation of variables
x2 2x 1
y
1 1
y 2 2
x 2x 1 x 1

w îðïí ̸» ݱ´´»¹» Þ±¿®¼ò


Ê·-·¬ ¬¸» ݱ´´»¹» Þ±¿®¼ ±² ¬¸» É»¾æ ©©©ò½±´´»¹»¾±¿®¼ò±®¹ò
ßÐr ÝßÔÝËÔËÍ ÞÝ
îðïí ÍÝÑÎ×ÒÙ ÙË×ÜÛÔ×ÒÛÍ

Ï«»-¬·±² ë

Note: This solution is valid for x 1.

w îðïí ̸» ݱ´´»¹» Þ±¿®¼ò


Ê·-·¬ ¬¸» ݱ´´»¹» Þ±¿®¼ ±² ¬¸» É»¾æ ©©©ò½±´´»¹»¾±¿®¼ò±®¹ò
ßÐr ÝßÔÝËÔËÍ ÞÝ
îðïí ÍÝÑÎ×ÒÙ ÙË×ÜÛÔ×ÒÛÍ

Ï«»-¬·±² ê

A function f has derivatives of all orders at x 0. Let Pn x denote the nth-degree Taylor polynomial
for f about x 0.

(a) It is known that f 0 4 and that P1 1 3. Show that f 0 2.


2
2 1
(b) It is known that f 0 and f 0 . Find P3 x .
3 3
(c) The function h has first derivative given by h x f 2 x . It is known that h 0 7. Find the
third-degree Taylor polynomial for h about x 0.

(a) P1 x f 0 f 0 x 4 f 0 x 1 : uses P1 x
2:
1 : verifies f 0 2
1 1
P1 4 f 0 · 3
2 2
1
f 0 · 1
2
f 0 2

2 x2 1 x3 1 : first two terms


(b) P3 x 4 2x · ·
3 2! 3 3!
3: 1 : third term
1 2 1 3 1 : fourth term
4 2x x x
3 18

(c) Let Qn x denote the Taylor polynomial of degree n for h about 2 : applies h x f 2x
x 0. 4: 1 : constant term
1 : remaining terms
1 2
h x f 2x Q3 x 4 2 2x 2x
3
x2 4 x3
Q3 x 4x 4· · C; C Q3 0 h 0 7
2 3 3
4 3
Q3 x 7 4x 2 x2 x
9

OR

h x f 2x , h x 2 f 2x , h x 4f 2x
8
h 0 f 0 4, h 0 2f 0 4, h 0 4f 0
3
x2 8 x3 4 3
Q3 x 7 4x 4· · 7 4x 2 x2 x
2! 3 3! 9

w îðïí ̸» ݱ´´»¹» Þ±¿®¼ò


Ê·-·¬ ¬¸» ݱ´´»¹» Þ±¿®¼ ±² ¬¸» É»¾æ ©©©ò½±´´»¹»¾±¿®¼ò±®¹ò
r
ßÐ Ý¿´½«´«- ÞÝ
îðïì ͽ±®·²¹ Ù«·¼»´·²»-

w îðïì ̸» ݱ´´»¹» Þ±¿®¼ò ݱ´´»¹» Þ±¿®¼ô ß¼ª¿²½»¼ д¿½»³»²¬ Ю±¹®¿³ô ßÐô ßÐ Ý»²¬®¿´ô ¿²¼ ¬¸» ¿½±®² ´±¹±
¿®» ®»¹·-¬»®»¼ ¬®¿¼»³¿®µ- ±º ¬¸» ݱ´´»¹» Þ±¿®¼ò
Ê·-·¬ ¬¸» ݱ´´»¹» Þ±¿®¼ ±² ¬¸» É»¾æ ©©©ò½±´´»¹»¾±¿®¼ò±®¹ò
ßÐ Ý»²¬®¿´ ·- ¬¸» ±ºº·½·¿´ ±²´·²» ¸±³» º±® ¬¸» ßРЮ±¹®¿³æ ¿°½»²¬®¿´ò½±´´»¹»¾±¿®¼ò±®¹ò
ßÐr ÝßÔÝËÔËÍ ßÞñÝßÔÝËÔËÍ ÞÝ
îðïì ÍÝÑÎ×ÒÙ ÙË×ÜÛÔ×ÒÛÍ

Ï«»-¬·±² ï

Grass clippings are placed in a bin, where they decompose. For 0 t 30, the amount of grass clippings
t
remaining in the bin is modeled by A t 6.687 0.931 , where A(t ) is measured in pounds and t is measured
in days.
(a) Find the average rate of change of A t over the interval 0 t 30. Indicate units of measure.

(b) Find the value of A 15 . Using correct units, interpret the meaning of the value in the context of the
problem.
(c) Find the time t for which the amount of grass clippings in the bin is equal to the average amount of grass
clippings in the bin over the interval 0 t 30.
(d) For t 30, L t , the linear approximation to A at t 30, is a better model for the amount of grass
clippings remaining in the bin. Use L t to predict the time at which there will be 0.5 pound of grass
clippings remaining in the bin. Show the work that leads to your answer.

A 30 A0
(a) 0.197 (or 0.196) lbs/day 1 : answer with units
30 0

(b) A 15 0.164 (or 0.163) 1 : A 15


2:
1 : interpretation
The amount of grass clippings in the bin is decreasing at a rate
of 0.164 (or 0.163) lbs/day at time t 15 days.

1 30
1 30 1: A t dt
(c) A t A t dt t 12.415 (or 12.414) 2: 30 0
30 0
1 : answer

(d) L t A 30 A 30 t 30 2 : expression for L t


4: 1: L t 0.5
A 30 0.055976 1 : answer
A 30 0.782928

L t 0.5 t 35.054

w îðïì ̸» ݱ´´»¹» Þ±¿®¼ò


Ê·-·¬ ¬¸» ݱ´´»¹» Þ±¿®¼ ±² ¬¸» É»¾æ ©©©ò½±´´»¹»¾±¿®¼ò±®¹ò
ßÐr ÝßÔÝËÔËÍ ÞÝ
îðïì ÍÝÑÎ×ÒÙ ÙË×ÜÛÔ×ÒÛÍ

Ï«»-¬·±² î

The graphs of the polar curves r 3 and r 3 2sin 2 are


shown in the figure above for 0 .
(a) Let R be the shaded region that is inside the graph of r 3
and inside the graph of r 3 2sin 2 . Find the area of R.

dx
(b) For the curve r 3 2sin 2 , find the value of at
d
.
6

(c) The distance between the two curves changes for 0 .


2
Find the rate at which the distance between the two curves is changing with respect to when .
3
d
(d) A particle is moving along the curve r 3 2sin 2 so that 3 for all times t 0. Find the value
dt
dr
of at .
dt 6

9 1 2 2
(a) Area 3 2sin 2 d 1 : integrand
4 2 0
9.708 (or 9.707) 3: 1 : limits
1 : answer

(b) x 3 2 sin 2 cos 1 : expression for x


2:
dx 1 : answer
2.366
d 6

(c) The distance between the two curves is 1 : expression for distance
2:
D 3 3 2 sin 2 2sin 2 . 1 : answer

dD
2
d 3

dr dr d dr
(d) 3 1 : chain rule with respect to t
dt d dt d 2:
dr 1 : answer
2 3 6
dt 6

w îðïì ̸» ݱ´´»¹» Þ±¿®¼ò


Ê·-·¬ ¬¸» ݱ´´»¹» Þ±¿®¼ ±² ¬¸» É»¾æ ©©©ò½±´´»¹»¾±¿®¼ò±®¹ò
ßÐr ÝßÔÝËÔËÍ ßÞñÝßÔÝËÔËÍ ÞÝ
îðïì ÍÝÑÎ×ÒÙ ÙË×ÜÛÔ×ÒÛÍ

Ï«»-¬·±² í

The function f is defined on the closed interval 5, 4 . The graph


of f consists of three line segments and is shown in the figure above.
x
Let g be the function defined by g x f t dt.
3

(a) Find g 3 .
(b) On what open intervals contained in 5 x 4 is the graph
of g both increasing and concave down? Give a reason for your
answer.
g x
(c) The function h is defined by h x . Find h 3 .
5x

(d) The function p is defined by p x f x2 x . Find the slope


of the line tangent to the graph of p at the point where x 1.

3
(a) g 3 f t dt 6 4 1 9 1 : answer
3

(b) g x f x 1 : answer
2:
1 : reason
The graph of g is increasing and concave down on the
intervals 5 x 3 and 0 x 2 because g f is
positive and decreasing on these intervals.

5x g x g x 5 5x g x 5g x
(c) h x 2
2:h x
5x 25 x 2 3:
1 : answer

5 3 g 3 5g 3
h 3 2
25·3
15 2 5 9 75 1
225 225 3

(d) p x f x2 x 2x 1 2: p x
3:
1 : answer
p 1 f 2 3 2 3 6

w îðïì ̸» ݱ´´»¹» Þ±¿®¼ò


Ê·-·¬ ¬¸» ݱ´´»¹» Þ±¿®¼ ±² ¬¸» É»¾æ ©©©ò½±´´»¹»¾±¿®¼ò±®¹ò
ßÐr ÝßÔÝËÔËÍ ßÞñÝßÔÝËÔËÍ ÞÝ
îðïì ÍÝÑÎ×ÒÙ ÙË×ÜÛÔ×ÒÛÍ

Ï«»-¬·±² ì

Train A runs back and forth on an east-west section of


railroad track. Train A’s velocity, measured in meters per t minutes 0 2 5 8 12
minute, is given by a differentiable function v A t , where
vA t meters minute 0 100 40 –120 –150
time t is measured in minutes. Selected values for v A t
are given in the table above.
(a) Find the average acceleration of train A over the interval 2 t 8.
(b) Do the data in the table support the conclusion that train A’s velocity is 100 meters per minute at some time t
with 5 t 8 ? Give a reason for your answer.
(c) At time t 2, train A’s position is 300 meters east of the Origin Station, and the train is moving to the east.
Write an expression involving an integral that gives the position of train A, in meters from the Origin Station, at
time t 12. Use a trapezoidal sum with three subintervals indicated by the table to approximate the position of
the train at time t 12.
(d) A second train, train B, travels north from the Origin Station. At time t the velocity of train B is given by
vB t 5t 2 60t 25, and at time t 2 the train is 400 meters north of the station. Find the rate, in meters
per minute, at which the distance between train A and train B is changing at time t 2.

vA 8 vA 2 120 100 110


(a) average accel m/min 2 1 : average acceleration
8 2 6 3

(b) v A is differentiable v A is continuous 1 : vA 8 100 v A 5


2:
vA 8 120 100 40 v A 5 1 : conclusion, using IVT

Therefore, by the Intermediate Value Theorem, there is a time t,


5 t 8, such that v A t 100.
12 12
(c) s A 12 sA 2 vA t dt 300 v A t dt 1 : position expression
2 2
12 100 40 40 120 120 150 3: 1 : trapezoidal sum
v A t dt 3 3 4 1 : position at time t 12
2 2 2 2
450
s A 12 300 450 150
The position of Train A at time t 12 minutes is approximately 150
meters west of Origin Station.

(d) Let x be train A’s position, y train B’s position, and z the distance 2 : implicit differentiation of
between train A and train B. 3: distance relationship
dz dx dy
z 2 x2 y2 2z 2x 2y 1 : answer
dt dt dt
x 300, y 400 z 500
vB 2 20 120 25 125
dz
500 300 100 400 125
dt
dz 80000
160 meters per minute
dt 500
w îðïì ̸» ݱ´´»¹» Þ±¿®¼ò
Ê·-·¬ ¬¸» ݱ´´»¹» Þ±¿®¼ ±² ¬¸» É»¾æ ©©©ò½±´´»¹»¾±¿®¼ò±®¹ò
ßÐr ÝßÔÝËÔËÍ ÞÝ
îðïì ÍÝÑÎ×ÒÙ ÙË×ÜÛÔ×ÒÛÍ

Ï«»-¬·±² ë

2
Let R be the shaded region bounded by the graph of y xe x , the line y 2 x,
and the vertical line x 1, as shown in the figure above.
(a) Find the area of R.
(b) Write, but do not evaluate, an integral expression that gives the volume of the
solid generated when R is rotated about the horizontal line y 2.
(c) Write, but do not evaluate, an expression involving one or more integrals that
gives the perimeter of R.

1 2
(a) Area xe x 2x dx 1 : integrand
0
x 1 3: 1 : antiderivative
1 x2
e x2 1 : answer
2 x 0
1 1 e 1
e 1
2 2 2

1 2
2 2 2 : integrand
(b) Volume xe x 2 2x 2 dx 3:
0
1 : limits and constant

d 2 2 2 2 2
(c) y xe x ex 2 x2ex ex 1 2 x2 1: y ex 1 2 x2
dx
3: 1 : integral
1
2 2 1 : answer
Perimeter 5 2 e 1 ex 1 2 x2 dx
0

w îðïì ̸» ݱ´´»¹» Þ±¿®¼ò


Ê·-·¬ ¬¸» ݱ´´»¹» Þ±¿®¼ ±² ¬¸» É»¾æ ©©©ò½±´´»¹»¾±¿®¼ò±®¹ò
ßÐr ÝßÔÝËÔËÍ ÞÝ
îðïì ÍÝÑÎ×ÒÙ ÙË×ÜÛÔ×ÒÛÍ

Ï«»-¬·±² ê

n 1 2n n
The Taylor series for a function f about x 1 is given by 1 x 1 and converges to f x for
n 1
n
x 1 R, where R is the radius of convergence of the Taylor series.
(a) Find the value of R.
(b) Find the first three nonzero terms and the general term of the Taylor series for f , the derivative of f, about
x 1.
(c) The Taylor series for f about x 1, found in part (b), is a geometric series. Find the function f to which
the series converges for x 1 R. Use this function to determine f for x 1 R.

(a) Let an be the nth term of the Taylor series. 1 : sets up ratio
3: 1 : computes limit of ratio
n 2 n 1
an 1 1 2n 1 x 1 n 1 : determines radius of convergence
·
an n 1 1 n 1 n
2 x 1 n

2n x 1
n 1

2n x 1
lim 2 x 1
n n 1
1
2 x 1 1 x 1
2

1
The radius of convergence is R .
2

(b) The first three nonzero terms are 2 : first three nonzero terms
2
3:
2 4 x 1 8 x 1 . 1 : general term

n 1 n n 1
The general term is 1 2 x 1 for n 1.

(c) The common ratio is 2 x 1. 1: f x


3: 1 : antiderivative
2 2 1
f x for x 1 1: f x
1 2 x 1 2x 1 2
2
f x dx ln 2 x 1 C
2x 1

f 1 0
ln 1 C 0 C 0
1
f x ln 2 x 1 for x 1
2

w îðïì ̸» ݱ´´»¹» Þ±¿®¼ò


Ê·-·¬ ¬¸» ݱ´´»¹» Þ±¿®¼ ±² ¬¸» É»¾æ ©©©ò½±´´»¹»¾±¿®¼ò±®¹ò
r
ßÐ Ý¿´½«´«- ÞÝ
îðïë ͽ±®·²¹ Ù«·¼»´·²»-

w îðïë ̸» ݱ´´»¹» Þ±¿®¼ò ݱ´´»¹» Þ±¿®¼ô ß¼ª¿²½»¼ д¿½»³»²¬ Ю±¹®¿³ô ßÐô ßÐ Ý»²¬®¿´ô ¿²¼ ¬¸» ¿½±®² ´±¹±
¿®» ®»¹·-¬»®»¼ ¬®¿¼»³¿®µ- ±º ¬¸» ݱ´´»¹» Þ±¿®¼ò
Ê·-·¬ ¬¸» ݱ´´»¹» Þ±¿®¼ ±² ¬¸» É»¾æ ©©©ò½±´´»¹»¾±¿®¼ò±®¹ò
ßÐ Ý»²¬®¿´ ·- ¬¸» ±ºº·½·¿´ ±²´·²» ¸±³» º±® ¬¸» ßРЮ±¹®¿³æ ¿°½»²¬®¿´ò½±´´»¹»¾±¿®¼ò±®¹ò
ßÐr ÝßÔÝËÔËÍ ßÞñÝßÔÝËÔËÍ ÞÝ
îðïë ÍÝÑÎ×ÒÙ ÙË×ÜÛÔ×ÒÛÍ

Ï«»-¬·±² ï

t2
The rate at which rainwater flows into a drainpipe is modeled by the function R, where R t 20sin cubic
35
feet per hour, t is measured in hours, and 0 t 8. The pipe is partially blocked, allowing water to drain out the
other end of the pipe at a rate modeled by D t 0.04t 3 0.4t 2 0.96t cubic feet per hour, for 0 t 8.
There are 30 cubic feet of water in the pipe at time t 0.
(a) How many cubic feet of rainwater flow into the pipe during the 8-hour time interval 0 t 8?
(b) Is the amount of water in the pipe increasing or decreasing at time t 3 hours? Give a reason for your
answer.
(c) At what time t, 0 t 8, is the amount of water in the pipe at a minimum? Justify your answer.
(d) The pipe can hold 50 cubic feet of water before overflowing. For t 8, water continues to flow into and out
of the pipe at the given rates until the pipe begins to overflow. Write, but do not solve, an equation involving
one or more integrals that gives the time w when the pipe will begin to overflow.

8 1 : integrand
(a) R t dt 76.570 2:
0 1 : answer

(b) R 3 D 3 0.313632 0 1 : considers R 3 and D 3


2:
Since R 3 D 3 , the amount of water in the pipe is 1 : answer and reason
decreasing at time t 3 hours.

(c) The amount of water in the pipe at time t, 0 t 8, is 1 : considers R t D t 0


t 3: 1 : answer
30 R x D x dx.
0 1 : justification

R t D t 0 t 0, 3.271658

t Amount of water in the pipe


0 30
3.271658 27.964561
8 48.543686

The amount of water in the pipe is a minimum at time


t 3.272 (or 3.271) hours.

w 1 : integral
(d) 30 R t D t dt 50 2:
0 1 : equation

w îðïë ̸» ݱ´´»¹» Þ±¿®¼ò


Ê·-·¬ ¬¸» ݱ´´»¹» Þ±¿®¼ ±² ¬¸» É»¾æ ©©©ò½±´´»¹»¾±¿®¼ò±®¹ò
ßÐr ÝßÔÝËÔËÍ ÞÝ
îðïë ÍÝÑÎ×ÒÙ ÙË×ÜÛÔ×ÒÛÍ

Ï«»-¬·±² î

At time t 0, a particle moving along a curve in the xy-plane has position x t , y t with velocity vector
2 0.5t
vt cos t ,e . At t 1, the particle is at the point 3, 5 .

(a) Find the x-coordinate of the position of the particle at time t 2.


(b) For 0 t 1, there is a point on the curve at which the line tangent to the curve has a slope of 2.
At what time is the object at that point?
(c) Find the time at which the speed of the particle is 3.
(d) Find the total distance traveled by the particle from time t 0 to time t 1.

1 : integral
2 2
(a) x 2 3 cos t dt 2.557 (or 2.556) 3: 1 : uses initial condition
1
1 : answer

dy dy dt e0.5t
(b) 1 : slope in terms of t
dx dx dt cos t 2 2:
1 : answer
e0.5t
2
cos t 2

t 0.840

(c) Speed cos2 t 2 et 1 : speed in terms of t


2:
1 : answer
cos 2 t 2 et 3

t 2.196 (or 2.195)

1 1 : integral
(d) Distance cos 2 t 2 et dt 1.595 (or 1.594) 2:
0 1 : answer

w îðïë ̸» ݱ´´»¹» Þ±¿®¼ò


Ê·-·¬ ¬¸» ݱ´´»¹» Þ±¿®¼ ±² ¬¸» É»¾æ ©©©ò½±´´»¹»¾±¿®¼ò±®¹ò
ßÐr ÝßÔÝËÔËÍ ßÞñÝßÔÝËÔËÍ ÞÝ
îðïë ÍÝÑÎ×ÒÙ ÙË×ÜÛÔ×ÒÛÍ

Ï«»-¬·±² í

t
0 12 20 24 40
(minutes)
v t
0 200 240 –220 150
(meters per minute)

Johanna jogs along a straight path. For 0 t 40, Johanna’s velocity is given by a differentiable function v.
Selected values of v t , where t is measured in minutes and v t is measured in meters per minute, are given in
the table above.
(a) Use the data in the table to estimate the value of v 16 .
40
(b) Using correct units, explain the meaning of the definite integral v t dt in the context of the problem.
0
40
Approximate the value of v t dt using a right Riemann sum with the four subintervals indicated in the
0
table.
(c) Bob is riding his bicycle along the same path. For 0 t 10, Bob’s velocity is modeled by
3 2
B t t 6t 300, where t is measured in minutes and B t is measured in meters per minute.
Find Bob’s acceleration at time t 5.
(d) Based on the model B from part (c), find Bob’s average velocity during the interval 0 t 10.

240 200
(a) v 16 5 meters/min 2 1 : approximation
20 12
40
(b) v t dt is the total distance Johanna jogs, in meters, over the 1 : explanation
0
time interval 0 t 40 minutes. 3: 1 : right Riemann sum
1 : approximation
40
v t dt 12 i v 12 8 i v 20 4 i v 24 16 i v 40
0
12 i 200 8 i 240 4 i 220 16 i 150
2400 1920 880 2400
7600 meters

(c) Bob’s acceleration is B t 3t 2 12t. 1 : uses B t


2:
B 5 3 25 12 5 15 meters/min 2 1 : answer

1 10
(d) Avg vel t3 6t 2 300 dt 1 : integral
10 0
10 3: 1 : antiderivative
1 t4
2t 3 300t 1 : answer
10 4 0
1 10000
2000 3000 350 meters/min
10 4

w îðïë ̸» ݱ´´»¹» Þ±¿®¼ò


Ê·-·¬ ¬¸» ݱ´´»¹» Þ±¿®¼ ±² ¬¸» É»¾æ ©©©ò½±´´»¹»¾±¿®¼ò±®¹ò
ßÐr ÝßÔËÝÔËÍ ßÞñÝßÔÝËÔËÍ ÞÝ
îðïë ÍÝÑÎ×ÒÙ ÙË×ÜÛÔ×ÒÛÍ

Ï«»-¬·±² ì

dy
Consider the differential equation 2x y.
dx
(a) On the axes provided, sketch a slope field for the given differential equation at the six points indicated.
d2y
(b) Find in terms of x and y. Determine the concavity of all solution curves for the given differential
dx 2
equation in Quadrant II. Give a reason for your answer.
(c) Let y f x be the particular solution to the differential equation with the initial condition f 2 3.
Does f have a relative minimum, a relative maximum, or neither at x 2 ? Justify your answer.
(d) Find the values of the constants m and b for which y mx b is a solution to the differential equation.

(a) 1 : slopes where x 0


2:
1 : slopes where x 1

d2y dy d2y
(b) 2 2 (2 x y) 2 2x y 1:
dx 2 dx 2: dx 2
1 : concave up with reason
In Quadrant II, x 0 and y 0, so 2 2 x y 0.
Therefore, all solution curves are concave up in Quadrant II.

dy dy
(c) 2 2 3 1 0 1 : considers
dx x, y 2, 3 2: dx x, y 2, 3
Therefore, f has neither a relative minimum nor a relative 1 : conclusion with justification
maximum at x 2.

dy d d
(d) y mx b mx b m 1: mx b m
dx dx dx
2x y m 3: 1 : 2x y m
2x mx b m 1 : answer
2 m x m b 0
2 m 0 m 2
b m b 2

Therefore, m 2 and b 2.
w îðïë ̸» ݱ´´»¹» Þ±¿®¼ò
Ê·-·¬ ¬¸» ݱ´´»¹» Þ±¿®¼ ±² ¬¸» É»¾æ ©©©ò½±´´»¹»¾±¿®¼ò±®¹ò
ßÐr ÝßÔÝËÔËÍ ÞÝ
îðïë ÍÝÑÎ×ÒÙ ÙË×ÜÛÔ×ÒÛÍ

Ï«»-¬·±² ë

1
Consider the function f x , where k is a nonzero constant. The derivative of f is given by
x2 kx
k 2x
f x 2
.
2
x kx
1
(a) Let k 3, so that f x 2
. Write an equation for the line tangent to the graph of f at the point
x 3x
whose x-coordinate is 4.
1
(b) Let k 4, so that f x . Determine whether f has a relative minimum, a relative maximum, or
x2 4x
neither at x 2. Justify your answer.
(c) Find the value of k for which f has a critical point at x 5.
1
(d) Let k 6, so that f x . Find the partial fraction decomposition for the function f.
x2 6x
Find f x dx.

1 1 3 2i4 5
(a) f 4 f (4) 1 : slope
4 2
3i 4 4 2 16 2:
42 3i 4 1 : tangent line equation
An equation for the line tangent to the graph of f at the point
5 1
whose x-coordinate is 4 is y x 4 .
16 4

4 2x 4 2i2
(b) f x 2
f 2 2
0 1 : considers f 2
x2 4x 22 4i2 2:
1 : answer with justification
f x changes sign from positive to negative at x 2.
Therefore, f has a relative maximum at x 2.

k 2i 5
(c) f 5 2
0 k 10 1 : answer
2
5 ki 5

1 1 A B
(d) 1 A x 6 Bx 2 : partial fraction decomposition
x2 6x x x 6 x x 6 4:
2 : general antiderivative
1
x 0 1 Ai 6 A
6
1
x 6 1 Bi 6 B
6
1 1 6 1 6
x x 6 x x 6
1 6 1 6
f x dx dx
x x 6
1 1 1 x 6
ln x ln x 6 C ln C
6 6 6 x
w îðïë ̸» ݱ´´»¹» Þ±¿®¼ò
Ê·-·¬ ¬¸» ݱ´´»¹» Þ±¿®¼ ±² ¬¸» É»¾æ ©©©ò½±´´»¹»¾±¿®¼ò±®¹ò
ßÐr ÝßÔÝËÔËÍ ÞÝ
îðïë ÍÝÑÎ×ÒÙ ÙË×ÜÛÔ×ÒÛÍ

Ï«»-¬·±² ê

3n 1
3 2 3n 1
The Maclaurin series for a function f is given by xn x x 3x 3 xn and
n 1
n 2 n
converges to f x for x R, where R is the radius of convergence of the Maclaurin series.
(a) Use the ratio test to find R.
(b) Write the first four nonzero terms of the Maclaurin series for f , the derivative of f. Express f as a
rational function for x R.

(c) Write the first four nonzero terms of the Maclaurin series for e x . Use the Maclaurin series for e x to write
the third-degree Taylor polynomial for g x e x f x about x 0.

(a) Let an be the nth term of the Maclaurin series. 1 : sets up ratio
3: 1 : computes limit of ratio
an 1 3 n xn 1
n 3n 1 : determines radius of convergence
i ix
an n 1 3 n 1 n
x n 1

3n
lim ix 3x
n n 1
1
3x 1 x
3
1
The radius of convergence is R .
3

(b) The first four nonzero terms of the Maclaurin series for f are 2 : first four nonzero terms
3:
1 3x 9x 2
27 x . 3 1 : rational function
1 1
f x
1 3x 1 3x

(c) The first four nonzero terms of the Maclaurin series for e x are 1 : first four nonzero terms
x2 x3 3: of the Maclaurin series for e x
1 x .
2! 3!
2 : Taylor polynomial
The product of the Maclaurin series for e x and the Maclaurin
series for f is
x2 x3 3 2
1 x x x 3 x3
2! 3! 2
1 2
x x 2 x3
2
The third-degree Taylor polynomial for g x ex f x
1 2
about x 0 is T3 x x x 2 x3 .
2
w îðïë ̸» ݱ´´»¹» Þ±¿®¼ò
Ê·-·¬ ¬¸» ݱ´´»¹» Þ±¿®¼ ±² ¬¸» É»¾æ ©©©ò½±´´»¹»¾±¿®¼ò±®¹ò
2017

AP Calculus BC
Free-Response Questions

© 2017 The College Board. College Board, Advanced Placement Program, AP, AP Central, and the acorn logo
are registered trademarks of the College Board. Visit the College Board on the Web: www.collegeboard.org.
îðïé ßÐr ÝßÔÝËÔËÍ ÞÝ ÚÎÛÛóÎÛÍÐÑÒÍÛ ÏËÛÍÌ×ÑÒÍ

ÍÛÝÌ×ÑÒ ××ô

ß ÙÎßÐØ×ÒÙ ÝßÔÝËÔßÌÑÎ ×Í ÎÛÏË×ÎÛÜ ÚÑÎ ÌØÛÍÛ ÏËÛÍÌ×ÑÒÍò

¸ º»»¬ ·- ¹·ª»² ¾§
¬¸» º«²½¬·±² ßô ©¸»®» ßø¸÷ ·- ³»¿-«®»¼ ·² -¯«¿®» º»»¬ò ̸» º«²½¬·±² ß ·- ½±²¬·²«±«- ¿²¼ ¼»½®»¿-»- ¿- ¸
·²½®»¿-»-ò Í»´»½¬»¼ ª¿´«»- º±® ßø¸÷ ¿®» ¹·ª»² ·² ¬¸» ¬¿¾´» ¿¾±ª»ò

ø¿÷ Ë-» ¿ ´»º¬ η»³¿²² -«³ ©·¬¸ ¬¸» ¬¸®»» -«¾·²¬»®ª¿´- ·²¼·½¿¬»¼ ¾§ ¬¸» ¼¿¬¿ ·² ¬¸» ¬¿¾´» ¬± ¿°°®±¨·³¿¬» ¬¸»
ª±´«³» ±º ¬¸» ¬¿²µò ײ¼·½¿¬» «²·¬- ±º ³»¿-«®»ò

ø¾÷ ܱ»- ¬¸» ¿°°®±¨·³¿¬·±² ·² °¿®¬ ø¿÷ ±ª»®»-¬·³¿¬» ±® «²¼»®»-¬·³¿¬» ¬¸» ª±´«³» ±º ¬¸» ¬¿²µá Û¨°´¿·² §±«®
®»¿-±²·²¹ò

ø½÷ ̸» ¿®»¿ô ·² -¯«¿®» º»»¬ô ±º ¬¸» ¸±®·¦±²¬¿´ ½®±-- -»½¬·±² ¿¬ ¸»·¹¸¬ ¸ º»»¬ ·- ³±¼»´»¼ ¾§ ¬¸» º«²½¬·±² º ¹·ª»²
ëðòí
¾§ º ø¸÷ ðòî¸
ò Þ¿-»¼ ±² ¬¸·- ³±¼»´ô º·²¼ ¬¸» ª±´«³» ±º ¬¸» ¬¿²µò ײ¼·½¿¬» «²·¬- ±º ³»¿-«®»ò
» ¸

ø¼÷ É¿¬»® ·- °«³°»¼ ·²¬± ¬¸» ¬¿²µò ɸ»² ¬¸» ¸»·¹¸¬ ±º ¬¸» ©¿¬»® ·- ë º»»¬ô ¬¸» ¸»·¹¸¬ ·- ·²½®»¿-·²¹ ¿¬ ¬¸» ®¿¬»
±º ðòîê º±±¬ °»® ³·²«¬»ò Ë-·²¹ ¬¸» ³±¼»´ º®±³ °¿®¬ ø½÷ô º·²¼ ¬¸» ®¿¬» ¿¬ ©¸·½¸ ¬¸» ª±´«³» ±º ©¿¬»® ·-
½¸¿²¹·²¹ ©·¬¸ ®»-°»½¬ ¬± ¬·³» ©¸»² ¬¸» ¸»·¹¸¬ ±º ¬¸» ©¿¬»® ·- ë º»»¬ò ײ¼·½¿¬» «²·¬- ±º ³»¿-«®»ò

© 2017 The College Board.


Visit the College Board on the Web: www.collegeboard.org.

ÙÑ ÑÒ ÌÑ ÌØÛ ÒÛÈÌ ÐßÙÛò


óîó
îðïé ßÐr ÝßÔÝËÔËÍ ÞÝ ÚÎÛÛóÎÛÍÐÑÒÍÛ ÏËÛÍÌ×ÑÒÍ

îò ̸» º·¹«®» ¿¾±ª» -¸±©- ¬¸» °±´¿® ½«®ª»- ® º ø¯ ÷ ï -·² ¯ ½±- øî ¯ ÷ ¿²¼ ® ¹ø¯ ÷ î ½±- ¯ º±®
°
ð ¯ ò Ô»¬ Î ¾» ¬¸» ®»¹·±² ·² ¬¸» º·®-¬ ¯«¿¼®¿²¬ ¾±«²¼»¼ ¾§ ¬¸» ½«®ª» ® º ø¯ ÷ ¿²¼ ¬¸» ¨ó¿¨·-ò Ô»¬ Í ¾»

¬¸» ®»¹·±² ·² ¬¸» º·®-¬ ¯«¿¼®¿²¬ ¾±«²¼»¼ ¾§ ¬¸» ½«®ª» ® º ø¯ ÷ô ¬¸» ½«®ª» ® ¹ø¯ ÷ô ¿²¼ ¬¸» ¨ó¿¨·-ò

ø¿÷ Ú·²¼ ¬¸» ¿®»¿ ±º Îò

°
ø¾÷ ̸» ®¿§ ¯ µ ô ©¸»®» ð µ ô ¼·ª·¼»- Í ·²¬± ¬©± ®»¹·±²- ±º »¯«¿´ ¿®»¿ò É®·¬»ô ¾«¬ ¼± ²±¬ -±´ª»ô ¿²

»¯«¿¬·±² ·²ª±´ª·²¹ ±²» ±® ³±®» ·²¬»¹®¿´- ©¸±-» -±´«¬·±² ¹·ª»- ¬¸» ª¿´«» ±º µ

°
ø½÷ Ú±® »¿½¸ ¯ô ð ¯ ô ´»¬ ©ø¯÷ ¾» ¬¸» ¼·-¬¿²½» ¾»¬©»»² ¬¸» °±·²¬- ©·¬¸ °±´¿® ½±±®¼·²¿¬»- ø º ø¯ ÷ô ¯÷ ¿²¼
î
°
ø ¹ø¯ ÷ô ¯ ÷ò É®·¬» ¿² »¨°®»--·±² º±® ©ø¯÷ò Ú·²¼ ©ßô ¬¸» ¿ª»®¿¹» ª¿´«» ±º ©ø¯÷ ±ª»® ¬¸» ·²¬»®ª¿´ ð ¯ ò
î

ø¼÷ Ë-·²¹ ¬¸» ·²º±®³¿¬·±² º®±³ °¿®¬ ø½÷ô º·²¼ ¬¸» ª¿´«» ±º ¯ º±® ©¸·½¸ ©ø¯ ÷ ©ßò ×- ¬¸» º«²½¬·±² ©ø¯ ÷
·²½®»¿-·²¹ ±® ¼»½®»¿-·²¹ ¿¬ ¬¸¿¬ ª¿´«» ±º ¯ á Ù·ª» ¿ ®»¿-±² º±® §±«® ¿²-©»®ò

ÛÒÜ ÑÚ ÐßÎÌ ß ÑÚ ÍÛÝÌ×ÑÒ ×× ñ

© 2017 The College Board.


Visit the College Board on the Web: www.collegeboard.org.

ÙÑ ÑÒ ÌÑ ÌØÛ ÒÛÈÌ ÐßÙÛò


óíó
îðïé ßÐr ÝßÔÝËÔËÍ ÞÝ ÚÎÛÛóÎÛÍÐÑÒÍÛ ÏËÛÍÌ×ÑÒÍ

ÝßÔÝËÔËÍ ÞÝ
ÍÛÝÌ×ÑÒ ××ô ﮬ Þ
Ì·³»œ
Ò«³¾»® ±º ¯«»-¬·±²-œìñ

ÒÑ ÝßÔÝËÔßÌÑÎ ×Í ßÔÔÑÉÛÜ ÚÑÎ ÌØÛÍÛ ÏËÛÍÌ×ÑÒÍòñ

íò ̸» º«²½¬·±² º ·- ¼·ºº»®»²¬·¿¾´» ±² ¬¸» ½´±-»¼ ·²¬»®ª¿´ –êô ë ¿²¼ -¿¬·-º·»- º ø– î÷ éò ̸» ¹®¿°¸ ±º º ô ¬¸»
¼»®·ª¿¬·ª» ±º ºô ½±²-·-¬- ±º ¿ -»³·½·®½´» ¿²¼ ¬¸®»» ´·²» -»¹³»²¬-ô ¿- -¸±©² ·² ¬¸» º·¹«®» ¿¾±ª»ò

ø¿÷ Ú·²¼ ¬¸» ª¿´«»- ±º º ø– ê÷ ¿²¼ º øë÷ò

ø¾÷ Ѳ ©¸¿¬ ·²¬»®ª¿´- ·- º ·²½®»¿-·²¹á Ö«-¬·º§ §±«® ¿²-©»®ò

ø½÷ Ú·²¼ ¬¸» ¿¾-±´«¬» ³·²·³«³ ª¿´«» ±º º ±² ¬¸» ½´±-»¼ ·²¬»®ª¿´ –êô ë ò Ö«-¬·º§ §±«® ¿²-©»®ò

ø¼÷ Ú±® »¿½¸ ±º º ø– ë÷ ¿²¼ º øí÷ô º·²¼ ¬¸» ª¿´«» ±® »¨°´¿·² ©¸§ ·¬ ¼±»- ²±¬ »¨·-¬ò

© 2017 The College Board.


Visit the College Board on the Web: www.collegeboard.org.

ÙÑ ÑÒ ÌÑ ÌØÛ ÒÛÈÌ ÐßÙÛò


óìó
îðïé ßÐr ÝßÔÝËÔËÍ ÞÝ ÚÎÛÛóÎÛÍÐÑÒÍÛ ÏËÛÍÌ×ÑÒÍ

¬ ðô ¿ ¾±·´»¼ °±¬¿¬± ·- ¬¿µ»² º®±³ ¿ °±¬ ±² ¿ -¬±ª» ¿²¼ ´»º¬ ¬± ½±±´ ·² ¿ µ·¬½¸»²ò ̸» ·²¬»®²¿´

¬»³°»®¿¬«®» ±º ¬¸» °±¬¿¬± ·- çï ¼»¹®»»- Ý»´-·«- ø Ý÷ ¿¬ ¬·³» ¬ ðô ¿²¼ ¬¸» ·²¬»®²¿´ ¬»³°»®¿¬«®» ±º ¬¸» °±¬¿¬±

·- ¹®»¿¬»® ¬¸¿² îé Ý º±® ¿´´ ¬·³»- ¬ ðò ̸» ·²¬»®²¿´ ¬»³°»®¿¬«®» ±º ¬¸» °±¬¿¬± ¿¬ ¬·³» ¬ ³·²«¬»- ½¿² ¾»
¼Ø ï
³±¼»´»¼ ¾§ ¬¸» º«²½¬·±² Ø ¬¸¿¬ -¿¬·-º·»- ¬¸» ¼·ºº»®»²¬·¿´ »¯«¿¬·±² – øØ – îé÷ô ©¸»®» Ø ø¬÷ ·-
¼¬
³»¿-«®»¼ ·² ¼»¹®»»- Ý»´-·«- ¿²¼ Øøð÷ çï

ø¿÷ É®·¬» ¿² »¯«¿¬·±² º±® ¬¸» ´·²» ¬¿²¹»²¬ ¬± ¬¸» ¹®¿°¸ ±º Ø ¿¬ ¬ ðò Ë-» ¬¸·- »¯«¿¬·±² ¬± ¿°°®±¨·³¿¬» ¬¸»
·²¬»®²¿´ ¬»³°»®¿¬«®» ±º ¬¸» °±¬¿¬± ¿¬ ¬·³» ¬ íò

¼ îØ
ø¾÷ Ë-» ¬± ¼»¬»®³·²» ©¸»¬¸»® §±«® ¿²-©»® ·² °¿®¬ ø¿÷ ·- ¿² «²¼»®»-¬·³¿¬» ±® ¿² ±ª»®»-¬·³¿¬» ±º ¬¸»
¼¬ î

·²¬»®²¿´ ¬»³°»®¿¬«®» ±º ¬¸» °±¬¿¬± ¿¬ ¬·³» ¬ íò

ø½÷ Ú±® ¬ ïðô ¿² ¿´¬»®²¿¬» ³±¼»´ º±® ¬¸» ·²¬»®²¿´ ¬»³°»®¿¬«®» ±º ¬¸» °±¬¿¬± ¿¬ ¬·³» ¬ ³·²«¬»- ·- ¬¸» º«²½¬·±²
¼Ù
Ù ¬¸¿¬ -¿¬·-º·»- ¬¸» ¼·ºº»®»²¬·¿´ »¯«¿¬·±² –øÙ – îé÷î ñ íô ©¸»®» Ùø¬÷ ·- ³»¿-«®»¼ ·² ¼»¹®»»- Ý»´-·«-
¼¬
¿²¼ Ùøð÷ çïò Ú·²¼ ¿² »¨°®»--·±² º±® Ùø¬ ÷ò Þ¿-»¼ ±² ¬¸·- ³±¼»´ô ©¸¿¬ ·- ¬¸» ·²¬»®²¿´ ¬»³°»®¿¬«®» ±º ¬¸»

°±¬¿¬± ¿¬ ¬·³» ¬ íá

© 2017 The College Board.


Visit the College Board on the Web: www.collegeboard.org.

ÙÑ ÑÒ ÌÑ ÌØÛ ÒÛÈÌ ÐßÙÛò


óëó
îðïé ßÐr ÝßÔÝËÔËÍ ÞÝ ÚÎÛÛóÎÛÍÐÑÒÍÛ ÏËÛÍÌ×ÑÒÍ

í
ëò Ô»¬ º ¾» ¬¸» º«²½¬·±² ¼»º·²»¼ ¾§ º ø¨÷ î
ò
î¨ – é¨ ë

ø¿÷ Ú·²¼ ¬¸» -´±°» ±º ¬¸» ´·²» ¬¿²¹»²¬ ¬± ¬¸» ¹®¿°¸ ±º º ¿¬ ¨ íò

ø¾÷ Ú·²¼ ¬¸» ¨ó½±±®¼·²¿¬» ±º »¿½¸ ½®·¬·½¿´ °±·²¬ ±º º ·² ¬¸» ·²¬»®ª¿´ ï ¨ îòëò Ý´¿--·º§ »¿½¸ ½®·¬·½¿´ °±·²¬ ¿-
¬¸» ´±½¿¬·±² ±º ¿ ®»´¿¬·ª» ³·²·³«³ô ¿ ®»´¿¬·ª» ³¿¨·³«³ô ±® ²»·¬¸»®ò Ö«-¬·º§ §±«® ¿²-©»®-ò

í î ï
ø½÷ Ë-·²¹ ¬¸» ·¼»²¬·¬§ ¬¸¿¬ î
– ô »ª¿´«¿¬» º ø¨÷ ¼¨ ±® -¸±© ¬¸¿¬ ¬¸» ·²¬»¹®¿´
î¨ – é¨ ë î¨ – ë ¨–ï ë

¼·ª»®¹»-ò

í
ø¼÷ Ü»¬»®³·²» ©¸»¬¸»® ¬¸» -»®·»- î
½±²ª»®¹»- ±® ¼·ª»®¹»-ò ͬ¿¬» ¬¸» ½±²¼·¬·±²- ±º ¬¸» ¬»-¬
² î² – é² ë

«-»¼ º±® ¼»¬»®³·²·²¹ ½±²ª»®¹»²½» ±® ¼·

© 2017 The College Board.


Visit the College Board on the Web: www.collegeboard.org.

ÙÑ ÑÒ ÌÑ ÌØÛ ÒÛÈÌ ÐßÙÛò


óêó
îðïé ßÐr ÝßÔÝËÔËÍ ÞÝ ÚÎÛÛóÎÛÍÐÑÒÍÛ ÏËÛÍÌ×ÑÒÍ

º øð÷ ð
º øð÷ ï
º ø² ï÷
øð ÷ – ² º ø²÷ øð÷ º±® ¿´´ ² ï

êò ß º«²½¬·±² º ¸¿- ¼»®·ª¿¬·ª»- ±º ¿´´ ±®¼»®- º±® –ï ¨ ïò ̸» ¼»®·ª¿¬·ª»- ±º º -¿¬·-º§ ¬¸» ½±²¼·¬·±²- ¿¾±ª»ò
̸» Ó¿½´¿«®·² -»®·»- º±® º ½±²ª»®¹»- ¬± º ø¨÷ º±® ¨ ïò

¨î ¨í ¨ì
ø¿÷ ͸±© ¬¸¿¬ ¬¸» º·®-¬ º±«® ²±²¦»®± ¬»®³- ±º ¬¸» Ó¿½´¿«®·² -»®·»- º±® º ¿®» ¨ – – ô ¿²¼ ©®·¬» ¬¸»
î í ì

¹»²»®¿´ ¬»®³ ±º ¬¸» Ó¿½´¿«®·² -»®·»- º±® ºò

ø¾÷ Ü»¬»®³·²» ©¸»¬¸»® ¬¸» Ó¿½´¿«®·² -»®·»- ¼»-½®·¾»¼ ·² °¿®¬ ø¿÷ ½±²ª»®
½±²¼·¬·±²¿´´§ô ±® ¼·ª»®¹»- ¿¬ ¨ ï

¨
ø½÷ É®·¬» ¬¸» º·®-¬ º±«® ²±²¦»®± ¬»®³- ¿²¼ ¬¸» ¹»²»®¿´ ¬»®³ ±º ¬¸» Ó¿½´¿«®·² -»®·»- º±® ¹ø¨÷ º ø¬ ÷ ¼ ¬ ò
ð

ï ï
ø¼÷ Ô»¬ в ®»°®»-»²¬ ¬¸» ²¬¸ó¼»¹®»» Ì¿§´±® °±´§²±³·¿´ º±® ¹ ¿¾±«¬ ¨ 𠻪¿´«¿¬»¼ ¿¬ ¨ ô ©¸»®» ¹ ·-
î î

¬¸» º«²½¬·±² ¼»º·²»¼ ·² °¿®¬ ø½÷ò Ë-» ¬¸» ¿´¬»®²¿¬·²¹ -»®·»- »®®±® ¾±«²¼ ¬± -¸±© ¬¸¿¬

ï ï ï
Ðì –¹ ò
î î ëðð

© 2017 The College Board.


Visit the College Board on the Web: www.collegeboard.org.

óéóñ
2017

AP Calculus BC
Scoring Guidelines

© 2017 The College Board. College Board, Advanced Placement Program, AP, AP Central, and the acorn logo
are registered trademarks of the College Board. Visit the College Board on the Web: www.collegeboard.org.
ßÐr ÝßÔÝËÔËÍ ßÞñÝßÔÝËÔËÍ ÞÝ
îðïé ÍÝÑÎ×ÒÙ ÙË×ÜÛÔ×ÒÛÍ

Ï«»-¬·±² ï

1 : units in parts (a), (c), and (d)

10
(a) Volume A h dh 1 : left Riemann sum
0 2:
2 0 A0 5 2 A2 10 5 A5 1 : approximation
2 50.3 3 14.4 5 6.5
176.3 cubic feet

(b) The approximation in part (a) is an overestimate because a left Riemann 1 : overestimate with reason
sum is used and A is decreasing.

10
(c) f h dh 101.325338 1 : integral
0 2:
1 : answer
The volume is 101.325 cubic feet.

h dV
(d) Using the model, V h f x dx. 2:
0 3: dt
1 : answer
dV dV dh
dt h 5 dh dt h 5
dh
f h
dt h 5
f 5 0.26 1.694419

When h 5, the volume of water is changing at a rate of


1.694 cubic feet per minute.

w îðïé ̸» ݱ´´»¹» Þ±¿®¼ò


Ê·-·¬ ¬¸» ݱ´´»¹» Þ±¿®¼ ±² ¬¸» É»¾æ ©©©ò½±´´»¹»¾±¿®¼ò±®¹ò
ßÐr ÝßÔÝËÔËÍ ÞÝ
îðïé ÍÝÑÎ×ÒÙ ÙË×ÜÛÔ×ÒÛÍ

Ï«»-¬·±² î

1 2 2
1 : integral
(a) f d 0.648414
2 0 2:
1 : answer
The area of R is 0.648.

k 2 2 1 2 2 2
(b) g f d g f d 1 : integral expression
0 2 0
2: for one region
— OR — 1 : equation

k 2 2 2 2 2
g f d g f d
0 k

(c) w g f 1:w
3: 1 : integral
2
w d 1 : average value
0
wA 0.485446
0
2

The average value of w on the interval 0, is 0.485.


2

(d) w wA for 0 0.517688 1 : solves w wA


2 2:
1 : answer with reason
w wA at 0.518 (or 0.517).

w 0.518 0 w is decreasing at 0.518.

w îðïé ̸» ݱ´´»¹» Þ±¿®¼ò


Ê·-·¬ ¬¸» ݱ´´»¹» Þ±¿®¼ ±² ¬¸» É»¾æ ©©©ò½±´´»¹»¾±¿®¼ò±®¹ò
ßÐr ÝßÔÝËÔËÍ ßÞñÝßÔÝËÔËÍ ÞÝ
îðïé ÍÝÑÎ×ÒÙ ÙË×ÜÛÔ×ÒÛÍ

Ï«»-¬·±² í

6 2
(a) f 6 f 2 f x dx 7 f x dx 7 4 3 1 : uses initial condition
2 6
5 3: 1: f 6
f 5 f 2 f x dx 7 2 3 10 2 1: f 5
2

(b) f x 0 on the intervals 6, 2 and 2, 5 . 2 : answer with justification


Therefore, f is increasing on the intervals 6, 2 and 2, 5 .

(c) The absolute minimum will occur at a critical point where f x 0 1 : considers x 2
2:
or at an endpoint. 1 : answer with justification

f x 0 x 2, x 2

x f x
6 3
2 7
2 7 2
5 10 2

The absolute minimum value is f 2 7 2 .

2 0 1
(d) f 5 1: f 5
6 2 2
2: 1 : f 3 does not exist,
with explanation
f x f 3 f x f 3
lim 2 and lim 1
x 3 x 3 x 3 x 3

f 3 does not exist because


f x f 3 f x f 3
lim lim .
x 3 x 3 x 3 x 3

w îðïé ̸» ݱ´´»¹» Þ±¿®¼ò


Ê·-·¬ ¬¸» ݱ´´»¹» Þ±¿®¼ ±² ¬¸» É»¾æ ©©©ò½±´´»¹»¾±¿®¼ò±®¹ò
ßÐr ÝßÔÝËÔËÍ ßÞñÝßÔÝËÔËÍ ÞÝ
îðïé ÍÝÑÎ×ÒÙ ÙË×ÜÛÔ×ÒÛÍ

Ï«»-¬·±² ì

1
(a) H 0 91 27 16 1 : slope
4
H 0 91 3: 1 : tangent line
1 : approximation
An equation for the tangent line is y 91 16t .

The internal temperature of the potato at time t 3 minutes is


approximately 91 16 3 43 degrees Celsius.

2
(b) d H 1 dH 1 1
H 27
1
H 27 1 : underestimate with reason
dt 2 4 dt 4 4 16

d 2H 1
H 27 for t 0 H 27 0 for t 0
dt 2 16
Therefore, the graph of H is concave up for t 0. Thus, the
answer in part (a) is an underestimate.

(c) dG
2 3
dt 1 : separation of variables
G 27 1 : antiderivatives
dG 1 : constant of integration and
2 3
1 dt 5:
G 27 uses initial condition
1 3
3 G 27 t C 1 : equation involving G and t
3 91 27 1 3
0 C C 12 1 : G t and G 3
1 3
3 G 27 12 t
Note: max 2 5 [1-1-0-0-0] if no constant
3
12 t of integration
G t 27 for 0 t 10
3
Note: 0 5 if no separation of variables
The internal temperature of the potato at time t 3 minutes is
3
12 3
27 54 degrees Celsius.
3

w îðïé ̸» ݱ´´»¹» Þ±¿®¼ò


Ê·-·¬ ¬¸» ݱ´´»¹» Þ±¿®¼ ±² ¬¸» É»¾æ ©©©ò½±´´»¹»¾±¿®¼ò±®¹ò
ßÐr ÝßÔÝËÔËÍ ÞÝ
îðïé ÍÝÑÎ×ÒÙ ÙË×ÜÛÔ×ÒÛÍ

Ï«»-¬·±² ë

3 4x 7
(a) f x 2
2: f 3
2
2x 7x 5
3 5 15
f 3
18 21 5 2 4

3 4x 7 7
(b) f x 0 x 1 : x-coordinate
2 2 4
2x 7x 5 2: 1 : relative maximum
7 with justification
The only critical point in the interval 1 x 2.5 has x-coordinate .
4
7
f changes sign from positive to negative at x .
4
7
Therefore, f has a relative maximum at x .
4

b b
3 2 1
(c) f x dx lim dx lim dx 1 : antiderivative
5 b 5 2x 2
7x 5 b 5 2x 5 x 1
b
3: 1 : limit expression
b 2x 5
lim ln 2 x 5 ln x 1 lim ln 1 : answer
b 5 b x 1 5
2b 5 5 5 8
lim ln ln ln 2 ln ln
b b 1 4 4 5

(d) f is continuous, positive, and decreasing on 5, . 2 : answer with conditions

3
The series converges by the integral test since dx
5 2 x2 7x 5
converges.

— OR —

3 1
2
0 and 0 for n 5.
2n 7n 5 n2

3
2 3 1
Since lim 2n 7n 5 and the series converges,
n 1 2 n 5 n 2

n2
3
the series 2
converges by the limit comparison test.
n 5 2n 7n 5

w îðïé ̸» ݱ´´»¹» Þ±¿®¼ò


Ê·-·¬ ¬¸» ݱ´´»¹» Þ±¿®¼ ±² ¬¸» É»¾æ ©©©ò½±´´»¹»¾±¿®¼ò±®¹ò
ßÐr ÝßÔÝËÔËÍ ÞÝ
îðïé ÍÝÑÎ×ÒÙ ÙË×ÜÛÔ×ÒÛÍ

Ï«»-¬·±² ê

4
(a) f 0 0 1 : f 0 , f 0 , and f 0
f 0 1 3: 1 : verify terms
f 0 11 1 1 : general term
f 0 2 1 2
4
f 0 3 2 6

The first four nonzero terms are


1 2 2 3 6 4 x2 x3 x4
0 1x x x x x .
2! 3! 4! 2 3 4

( 1) n 1 x n
The general term is .
n

1n 1
(b) For x 1, the Maclaurin series becomes . 2 : converges conditionally
n 1
n
with reason
The series does not converge absolutely because the harmonic series
diverges.

The series alternates with terms that decrease in magnitude to 0, and


therefore the series converges conditionally.

x n 1 n
(c)
x t2 t3 t4 1 t
1 : two terms
f t dt t dt
0 2 3 4 n
0 3: 1 : remaining terms
t x
1 n tn 1
1 1 : general term
t2 t3 t4 t5
2 3 2 4 3 5 4 n 1 n
t 0
2 3 4 5 n 1 n 1
x x x x 1 x
2 6 12 20 n 1 n

(d) The terms alternate in sign and decrease in magnitude to 0. By the 1 : error bound
alternating series error bound, the error P4 1 g
1 is bounded
2 2
5
1 2
by the magnitude of the first unused term, .
20
5
1 1 1 2 1 1
Thus, P4 g .
2 2 20 32 20 500

w îðïé ̸» ݱ´´»¹» Þ±¿®¼ò


Ê·-·¬ ¬¸» ݱ´´»¹» Þ±¿®¼ ±² ¬¸» É»¾æ ©©©ò½±´´»¹»¾±¿®¼ò±®¹ò

You might also like